Med Surg II Exam 1

Pataasin ang iyong marka sa homework at exams ngayon gamit ang Quizwiz!

Nursing activities for a client with ALS and family include helping themThe nurThe Nurse cautions clients with ALS and their families to be aware that (Select all that apply)

...activities should be spaced throughout the day. muscle weakness may cause a risk for injury. Safety is a prime concern with ALS (and with any degenerative neurologic disorder). Muscle weakness is progressive, leading to increased risk of falls. Some interventions to prevent this include spacing activities throughout the day, conserving energy, avoiding extremes of hot and cold, and using assistive devices such as canes or wheelchairs. Clients with ALS usually do not experience incontinence and cognition remains intact for the duration of the disorder.

Health promotion activities the nurse could suggest to a community group for Huntington's disease include

...genetic screening for high-risk individuals.

A patient with Parkinson's disease has decreased tongue mobility and an inability to move the facial muscles. The nurse recognizes that these impairments commonly contribute to the nursing diagnosis of

...impaired verbal communication related to difficulty articulating. Rationale: The inability to use the tongue and facial muscles decreases the patient's ability to socialize or communicate needs. Disuse syndrome is not an appropriate nursing diagnosis because the patient is continuing to use the muscles as much as possible. There is no indication in the stem that the patient has a self-care deficit, bradykinesia, or rigidity. The oral mucous membranes will continue to be moist and should not be impaired by the patient's difficulty swallowing.

A patiA patient with Guillain-Barré syndrome asks the nurse what has caused the disease. In responding to the patient, the nurse explains that Guillain-Barré syndrome

...is due to an immune reaction that attacks the covering of the peripheral nerves.

When teaching a patient with myasthenia gravis (MG) about management of the disease, the nurse advises the patient to

...perform necessary physically demanding activities in the morning. Rationale: Muscles are generally strongest in the morning, and activities involving muscle activity should be scheduled then. Plasmapheresis is not routinely scheduled but is used for myasthenia crisis or for in situations where corticosteroid therapy should be discontinued. There is no decrease in sensation with MG, and muscle atrophy does not occur because muscles are used during part of the day.

A 24A 29 -year-old patient is hospitalized with the onset of Guillain-Barré syndrome. During this phase of the patient's illness, the most essential assessment for the nurse to carry out is

...performing constant evaluation of respiratory function.

The nurse reminds a group of students about the major component of pathophysiology in multiple sclerosis (MS), which is

...plaques occur anywhere in the white matter of the central nervous system (CNS). Although plaques may occur anywhere in the white matter of the CNS, the areas most commonly involved are the optic nerves, cerebrum, and cervical spinal cord.

A client with advanced ALS is admitted to the hospital. Because of manifestations that are common in clients with ALS, the nurse should

...provide the client with small, frequent feedings. The course of the disease is relentlessly progressive. Cognition, as well as bowel and bladder sphincters, remains intact. The client may be malnourished because of dysphagia. Encourage small, frequent, high-nutrient feedings. The nurse should assess for aspiration and choking. A feeding tube may be considered during the course of the illness.

When is dementia usually diagnosed? A. Two brain functions are impaired. B. Memory is affected. C. Positive result is obtained on a computed tomography (CT) study. D. The patient fails the Benton Visual Form Discrimination Test (BVFD).

A Dementia is usually diagnosed when two or more brain functions, such as memory loss or language skills, are significantly impaired. The Mini-Mental State Examination (MMSE) is used to assess cognitive effect. Although tests help to make the diagnosis, no single clinical test can be used to diagnose dementia, and it is primarily a diagnosis of exclusion.

The nurse is caring for a hospitalized client with Alzheimer's disease who has a history of agitation. Which intervention does the nurse implement to help prevent agitation and aggressive behavior in this client? a. Provide undisturbed sleep. b. Orient the client to reality. c. Leave the television turned on. d. Administer hypnotic drugs as needed.

A Fatigue from disturbed sleep increases confusion and behavioral manifestations, such as aggression and agitation. Reality orientation is inappropriate for clients in a later stage of the disease. Constant noise from the TV most likely would agitate the client. Sedation should be used as a last resort.

Which modifiable risk factors for stroke are most important for you to include when planning a community education program? A. Hypertension B. Hyperlipidemia C. Alcohol consumption D. Oral contraceptive use

A Hypertension is the single most important modifiable risk factor, but it is still often undetected and inadequately treated.

8. For mild-to-moderate migraines you would anticipate the use of which medication? A. Aspirin or acetaminophen B. Ibuprofen C. Sumatriptan (Imitrex) D. Codeine

A Many people with mild or moderate migraine can obtain relief with aspirin or acetaminophen. Reference: 1488

You know the pathologic process of Huntington's disease (HD) involves a deficiency of A. acetylcholine. B. dopamine. C. serotonin. D. endorphins.

A The pathologic process of HD involves the basal ganglia and the extrapyramidal motor system. However, instead of a deficiency of dopamine, HD involves a deficiency of the neurotransmitters acetylcholine and γ-aminobutyric acid (GABA). Reference: 1515

A hospitalized client with late-stage Alzheimer's disease says that breakfast has not been served. The nurse witnessed the client eating breakfast earlier. Which statement made to this client is an example of validation therapy? a. "I see you are still hungry. I will get you some toast." b. "You are confused about mealtimes this morning." c. "You ate your breakfast 30 minutes ago." d. "You look tired. Maybe a nap will help."

A Use of validation therapy involves acknowledgment of the client's feelings and concerns. This technique has proved more effective in later stages of the disease, when using reality orientation only increases agitation. Telling the client that he or she already ate breakfast may agitate the client. The other statements do not validate the client's concerns.

You are called to the patient's room and find the patient in a clonic reaction. Your priority action is to A. record the time sequence of all of the patient's movements and responses as they occur. B. turn the patient to the side. C. call the health care provider. D. start oxygen by mask at 6 L/miN

A When a seizure occurs, you should carefully observe and record details of the event because the diagnosis and subsequent treatment often rest solely on the seizure description. All aspects of the seizure should be noted. What events preceded the seizure? When did the seizure occur? How long did each phase (aural [if any], ictal, postictal) last? Reference: 1499

A patient with a suspected closed head injury has bloody nasal drainage. You suspect that this patient has a cerebrospinal fluid (CSF) leak when observing which of the following? A. A halo sign on the nasal drip pad B. Decreased blood pressure and urinary output C. A positive reading for glucose on a Test-tape strip D. Clear nasal drainage along with the bloody discharge

A When drainage containing CSF and blood is allowed to drip onto a white pad, the blood coalesces into the center within a few minutes, and a yellowish ring of CSF encircles the blood, giving a halo effect. The presence of glucose is unreliable for determining the presence of CSF because blood also contains glucose.

You would correctly identify that the most common early symptom(s) of myasthenia gravis are A. weakness, fatigue, and ptosis. B. significant unilateral weakness. C. nausea, dizziness, and dysphagia. D. numbness and tingling of the extremities.

A he primary feature of MG is fluctuating weakness of skeletal muscle. Strength is usually restored after a period of rest. The muscles most often involved are those used for moving the eyes and eyelids, chewing, swallowing, speaking, and breathing. Reference: 1512

1. A patient with septic shock has a urine output of 20 mL/hr for the past 3 hours. The pulse rate is 120 and the central venous pressure and pulmonary artery wedge pressure are low. Which of these orders by the health care provider will the nurse question? a. Give furosemide (Lasix) 40 mg IV. b. Increase normal saline infusion to 150 mL/hr. c. Administer hydrocortisone (SoluCortef) 100 mg IV. d. Prepare to give drotrecogin alpha (Xigris) 24 mcg/kg/hr.

ANS: A Furosemide will lower the filling pressures and renal perfusion further for the patient with septic shock. The other orders are appropriate. DIF: Cognitive Level: Application REF: 1724-1726 | 1731 | 1733 TOP: Nursing Process: Implementation MSC: NCLEX: Physiological Integrity

Manifestations of left brain damage

Aphasia, inability to remember words

Which action should the nurse take when the low pressure alarm sounds for a patient who has an arterial line in the right radial artery?

Assess for cardiac dysrhythmias. rationale: The low pressure alarm indicates a drop in the patient's blood pressure, which may be caused by cardiac dysrhythmias. There is no indication to rezero the equipment. Pallor of the right hand would be caused by occlusion of the radial artery by the arterial catheter, not by low pressure. There is no indication of a need for flushing the line.

15.The client suspected to have myasthenia gravis is about to undergo the Tensilon (edrophonium chloride) test. Which drug will the nurse have available for complications of this test?

Atropine sulfate Tensilon increases cholinergic responses and can slow the heart rate down so that ectopic beats dominate, causing cardiac fibrillation or arrest. Atropine sulfate is an anticholinergic drug.

A female patient has left-sided hemiplegia after an ischemic stroke that occurred 2 weeks earlier. How should you best promote the integrity of the patient's skin? A. Position the patient on her weak side most of the time. B. Alternate the patient between supine and side-lying positions. C. Avoid the use of pillows to promote independence in positioning. D. Establish a schedule for the massage of areas where skin breakdown emerges.

B A position change schedule should be established for stroke patients. An example is side-backside positioning, with a maximum duration of 2 hours for any position. The patient should be positioned on the weak or paralyzed side for only 30 minutes. Pillows may be used to facilitate positioning. Areas of skin breakdown should never be massaged.

The early stage of AD is characterized by A. no noticeable change in behavior. B. memory problems and mild confusion. C. increased time spent sleeping or in bed. D. incontinence, agitation, and wandering behavior.

B An initial sign of AD is a subtle deterioration in memory.

Which response can be expected in a patient with low oxygen concentration and acidosis? A. Decreased cerebral fluid flow with decreased cerebral pressure B. Vasodilation with increased cerebral pressure C. Systemic hypotension with decreased cerebral pressure D. Cerebral tissue hypertrophy with increased cerebral pressure

B Low concentration of oxygen ions and high concentration of hydrogen ions cause vasodilation, which can result in increased ICP if autoregulation has failed. The other options are not possible

You would correctly identify the pathophysiologic etiology of myasthenia gravis as a deficit of A. dopamine. B. acetylcholine. C. myelin. D. albumin.

B MG is caused by an autoimmune process in which antibodies attack acetylcholine (ACh) receptors, resulting in a decreased number of ACh receptor (AChR) sites at the neuromuscular junction. This prevents ACh molecules from attaching and stimulating muscle contraction. Reference: 1510

A client who experienced a spinal cord injury 1 hour ago is brought to the emergency department. Which prescribed medication does the nurse prepare to administer to this client? a. Intrathecal baclofen (Lioresal) b. Methylprednisolone (Medrol) c. Atropine sulfate d. Epinephrine (Adrenalin)

B Methylprednisolone (Medrol) should be given within 8 hours of the injury. Clients who receive this therapy usually show improvement in motor and sensory function. The other medications are inappropriate for the client.

What is the treatment of choice for normal pressure hydrocephalus? A. Donepezil (Aricept) B. Shunt C. Furosemide (Lasix) D. Aspiration

B Normal pressure hydrocephalus results from an obstruction in the flow of cerebrospinal fluid (CSF), which causes a buildup of CSF fluid in the brain. Manifestations of the condition include dementia, urinary incontinence, and difficulty walking. Meningitis, encephalitis, or head injury may cause the condition. If diagnosed early, it is treated by surgically inserting a shunt to divert the fluid.

A patient has ptosis secondary to myasthenia gravis. Which assessment finding would you expect to see in this patient? A. Redness and swelling of the conjunctiva B. Drooping of the upper lid margin in one or both eyes C. Redness, swelling, and crusting along the lid margins D. Small, superficial white nodules along the lid margin

B Ptosis is the term used to describe drooping of the upper lid margin, which may be either unilateral or bilateral. Ptosis can be a result of mechanical causes, such as an eyelid tumor or excess skin, or from myogenic causes such as myasthenia gravis. Reference: 1512, 1513

The patient is seen in the clinic due to an increase in the frequency of seizure activity. In addition to a thorough health history you should draw blood for A. anemia. B. serum drug levels. C. arterial blood gases. D. electrolytes.

B Serum levels of the drug should be monitored if seizures continue to occur, if seizure frequency increases, or if drug compliance is questioned. The therapeutic range for each drug indicates the serum level above which most patients experience toxic side effects and below which most continue to have seizures. Reference: 1495

After a major head trauma, the patient's respiratory and cardiac functions are affected. Which area of the brain is damaged? A. Temporal lobe of the cerebrum B. Brainstem C. Cerebellum D. Spinal nerves

B The brainstem includes the midbrain, pons, and medulla. The vital centers concerned with respiratory, vasomotor, and cardiac function are located in the medulla. Integration of somatic, visual, and auditory data occurs in the temporal lobe. The cerebellum coordinates voluntary movement, trunk stability, and equilibrium. Motor and spinal nerves serve particular areas of the body.

The nurse is caring for a client with a history of epilepsy who suddenly begins to experience a tonic-clonic seizure and loses consciousness. What is the nurse's priority action? a. Restrain the client's extremities. b. Turn the client's head to the side. c. Take the client's blood pressure. d. Place an airway into the client's mouth.

B The nurse should turn the client's head to the side to prevent aspiration and allow drainage of secretions. The client should not be restrained nor an airway placed in his or her mouth during the seizure because these actions increase seizure activity and can harm the client. Vital signs are measured in the postictal phase of the seizure.

he nursing student would be correct in identifying that the pain of a cluster headache is characterized as A. dull and heavy. B. sharp and stabbing. C. preceded by an aura. D. worsened by light.

B The pain of cluster headache is sharp and stabbing, which is in contrast to the pulsing pain of the migraine headache. Reference: 1487

The patient asks you, "How can I be certain I have Parkinson's disease?" Your response is based on the knowledge that the absolute confirmation of the diagnosis is A. decreased serum dopamine level. B. positive response to medication administration. C. nerve biopsy. D. electromyography (EMG).

B The ultimate confirmation of PD is a positive response to antiparkinsonian drugs. Reference: 1508

Vasogenic cerebral edema increases ICP by A. shifting fluid in the gray matter. B. altering the endothelial lining of cerebral capillaries. C. leaking molecules from the intracellular fluid to the capillaries. D. altering the osmotic gradient flow into the intravascular component.

B Vasogenic cerebral edema occurs mainly in the white matter and is caused by changes in the endothelial lining of cerebral capillaries.

The nurse notes thick, white respiratory secretions for a patient who is receiving mechanical ventilation. Which intervention will be most effective in resolving this problem?

Add additional water to the patient's enteral feedings. rationale: Because the patient's secretions are thick, better hydration is indicated. Suctioning every hour without any specific evidence for the need will increase the incidence of mucosal trauma and would not address the etiology of the ineffective airway clearance. Instillation of saline does not liquefy secretions and may decrease the SpO2. Repositioning the patient is appropriate but will not decrease the thickness of secretions.

16. When the charge nurse is evaluating the skills of a new RN, which action by the new RN indicates a need for more education in the care of patients with shock? a. Placing the pulse oximeter on the ear for a patient with septic shock b. Keeping the head of the bed flat for a patient with hypovolemic shock c. Decreasing the room temperature to 68° F for a patient with neurogenic shock d. Increasing the nitroprusside (Nipride) infusion rate for a patient with a high SVR

ANS: C Patients with neurogenic shock may have poikilothermia. The room temperature should be kept warm to avoid hypothermia. The other actions by the new RN are appropriate. DIF: Cognitive Level: Application REF: 1721-1722 | 1724 OBJ: Special Questions: Delegation TOP: Nursing Process: Evaluation MSC: NCLEX: Safe and Effective Care Environment

Benzodiazepines are indicated in the treatment of cases of delirium that have which cause? A. Polypharmacy B. Cerebral hypoxia C. Alcohol withdrawal D. Electrolyte imbalances

C Benzodiazepines can be used to treat delirium associated with sedative and alcohol withdrawal. However, these drugs may worsen delirium caused by other factors and must be used cautiously.

The earliest signs of increased ICP the nurse should assess for include a. Cushing's triad b. unexpected vomiting c. decreasing level of consciousness (LOC) d. dilated pupil with sluggish response to light

C. One of the most sensitive signs of increased intracranial pressure (ICP) is a decreasing LOC. A decrease in LOC will occur before changes in vital signs, ocular signs, and projectile vomiting occur

21.A client with myasthenia gravis is preparing for discharge. Which instructions will be included in the education of the client's family members or caregiver?

Cardiopulmonary resuscitation (CPR) Respiratory compromise is a common occurrence with myasthenia gravis. The client's family members are encouraged to learn CPR and to have resuscitation equipment available in the home.

A patient with cardiomyopathy is demonstrating signs of cardiogenic shock. The nurse realizes that this type of shock is due to: 1. Reduced cardiac output 2. Increased stroke volume 3. Reduced blood volume 4. Blood flow blocked in the pulmonary circulation

Correct Answer: 1 Rationale 1: In cardiogenic shock, cardiac output is reduced, leading to poor tissue perfusion. Rationale 2: In cardiogenic shock, stroke volume is decreased. Rationale 3: There is not a reduction of blood volume in cardiogenic shock. Rationale 4: There is not a blockage of blood flow through the pulmonary circulation in cardiogenic shock.

A patient is brought to the emergency department with hypotension, tachycardia, reduced capillary refill, and oliguria. During the assessment, the nurse determines the patient is experiencing cardiogenic shock because of which additional finding? 1. Jugular vein distention 2. Dry mucous membranes 3. Poor skin turgor 4. Thirst

Correct Answer: 1 Rationale 1: Jugular vein distention is a manifestation of cardiogenic shock. Rationale 2: The mucous membranes are not dry in cardiogenic shock. Rationale 3: The skin turgor is not poor in cardiogenic shock. Rationale 4: Thirst is not a manifestation of cardiogenic shock.

The nurse, caring for a patient recovering from an acute myocardial infarction, is planning interventions to reduce the risk of which type of shock? 1. Cardiogenic 2. Hypovolemic 3. Distributive 4. Obstructive

Correct Answer: 1 Rationale 1: One etiology of cardiogenic shock is a myocardial infarction. Rationale 2: Acute myocardial infarction does not cause hypovolemic shock. Rationale 3: Acute myocardial infarction does not cause distributive shock. Rationale 4: Acute myocardial infarction does not cause obstructive shock.

Manifestations of right brain damage

Left homonymous hemianopsia, agnosia, quick impulsive behavior, neglect of the left side of the body

The nurse notes that a patient's endotracheal tube (ET), which was at the 21-cm mark, is now at the 24-cm mark and the patient appears anxious and restless. Which action should the nurse take first?

Listen to the patient's lungs. rationale: The nurse should first determine whether the ET tube has been displaced into the right mainstem bronchus by listening for unilateral breath sounds. If so, assistance will be needed to reposition the tube immediately. The other actions also are appropriate, but detection and correction of tube malposition are the most critical actions.

Treat for hemorrhagic stroke

Lower BP

A nurse is caring for a client with Guillain-Barré syndrome who has been admitted to the intensive care unit. During the last 2 hours, the nurse notes that the client's vital capacity has declined to 12 mL/kg, and the client is having difficulty clearing secretions. Which is the nurse's priority action?

Preparing the client for elective intubation Deterioration in vital capacity to less than 15 mL/kg and the inability to clear secretions are indications for elective intubation.

3.The nurse correlates which clinical manifestation of Guillain-Barré syndrome as the most common?

Progressive, ascending weakness and paresthesia The most common clinical pattern of Guillain-Barré syndrome is the ascending variety. Weakness and paresthesia begin in the lower extremities and progress upward.

When caring for a patient with pulmonary hypertension, which parameter will the nurse monitor to evaluate whether treatment has been effective?

Pulmonary vascular resistance (PVR) rationale: PVR is a major contributor to pulmonary hypertension, and a decrease would indicate that pulmonary hypertension was improving. The other parameters also may be monitored, but do not directly assess for pulmonary hypertension.

Which conditions or factors in a middle-aged woman diagnosed with Guillain-Barré syndrome are most likely to have contributed to this problem?

She had a viral infection about 2 weeks ago. The client with GBS often relates a history of acute illness, trauma, surgery, or immunization 1 to 3 weeks before the onset of neurologic symptoms.

A patient with respiratory failure has hemodynamic monitoring and is receiving mechanical ventilation with peak end-expiratory pressure (PEEP) of 10 cm H2O. Which information indicates that a change in the ventilator settings may be required?

The arterial line shows a blood pressure of 90/46. rationale: The hypotension indicates that the high intrathoracic pressure caused by the PEEP may be decreasing venous return and cardiac output (CO). The other assessment data would not be caused by mechanical ventilation.

Which assessment information obtained by the nurse when caring for a patient receiving mechanical ventilation indicates the need for suctioning?

The respiratory rate is 32 breaths/min. rationale: The increase in respiratory rate indicates that the patient may have decreased airway clearance and requires suctioning. Suctioning is done when patient assessment data indicate that it is needed, not on a scheduled basis. Occasional expiratory wheezes do not indicate poor airway clearance, and suctioning the patient may induce bronchospasm and increase wheezing. An SpO2 of 93% is acceptable and does not suggest that immediate suctioning is needed.

While caring for a patient who has been admitted with a pulmonary embolism, the nurse notes a change in the patient's oxygen saturation from 94% to 88%. Which action should the nurse take next? a. increase oxygen flow rate b. suction the patient's oropharynx c. instruct the patient to cough and deep breathe d. help patient sit in a more upright position

a. increase oxygen flow rate

A nurse is caring for an obese patient with right lower lobe pneumonia. Which position will be best to improve gas exchange? a. On the left side b. On the right side c. In the tripod position d. In the high-Fowler's position

a. on the left side

When evaluating outcomes of a glycerol rhizotomy for a patient with trigeminal neuralgia, the nurse will a. assess whether the patient is doing daily facial exercises. b. question whether the patient is using an eye shield at night. c. ask the patient about social activities with family and friends. d. remind the patient to chew on the unaffected side of the mouth.

c. ask the patient about social activities

Which nursing action will the home health nurse include in the plan of care for a patient with paraplegia at the T4 level in order to prevent autonomic dysreflexia? a. Support selection of a high-protein diet. b. Discuss options for sexuality and fertility. c. Assist in planning a prescribed bowel program. d. Use quad coughing to strengthen cough efforts.

c. assist in planning a prescribed bowel program

When a brain-injured patient responds to nail bed pressure with internal rotation, adduction, and flexion of the arms, the nurse reports the response as a. flexion withdrawal. b. localization of pain. c. decorticate posturing. d. decerebrate posturing.

c. decorticate posturing

A patient develops increasing dyspnea and hypoxemia 2 days after heart surgery. To determine whether the patient has acute respiratory distress syndrome (ARDS) or pulmonary edema caused by heart failure, the nurse will plan to assist with a. obtaining a ventilation-perfusion scan. b. drawing blood for arterial blood gases. c. insertion of a pulmonary artery catheter. d. positioning the patient for a chest x-ray.

c. insertion of a pulmonary artery catheter

A 33-year-old patient with a T4 spinal cord injury asks the nurse whether he will be able to be sexually active. Which initial response by the nurse is best? a. Reflex erections frequently occur, but orgasm may not be possible. b. Sildenafil (Viagra) is used by many patients with spinal cord injury. c. Multiple options are available to maintain sexuality after spinal cord injury. d. Penile injection, prostheses, or vacuum suction devices are possible options.

c. multiple options are available to maintain sexuality after spinal cord injury

A 27-year-old patient is hospitalized with new onset of Guillain-Barré syndrome. The most essential assessment for the nurse to carry out is a. determining level of consciousness. b. checking strength of the extremities. c. observing respiratory rate and effort. d. monitoring the cardiac rate and rhythm

c. observing respiratory rate and effort

The nurse documents the vital signs for a patient admitted 2 days ago with gram-negative sepsis: temperature 101.2° F, blood pressure 90/56 mm Hg, pulse 92, respirations 34. Which action should the nurse take next? a. Give the scheduled IV antibiotic. b. Give the PRN acetaminophen (Tylenol). c. Obtain oxygen saturation using pulse oximetry. d. Notify the health care provider of the patient's vital signs.

c. obtain O2Sat using pulse oximetry

Problem with IV-TPA

only 1-3% of patients arrive in time to receive it

The intensive care unit (ICU) charge nurse will determine that teaching about hemodynamic monitoring for a new staff nurse has been effective when the new nurse ______________

positions the zero-reference stopcock line level with the phlebostatic axis. rationale: For accurate measurement of pressures, the zero-reference level should be at the phlebostatic axis. There is no need to rebalance and recalibrate monitoring equipment hourly. Accurate hemodynamic readings are possible with the patient's head raised to 45 degrees or in the prone position. The anatomic position of the phlebostatic axis does not change when patients are repositioned.

A patient with multiple sclerosis (MS) has a nursing diagnosis of urinary retention related to sensorimotor deficits. An appropriate nursing intervention for this problem is to

teach the patient how to use the Credé method. Rationale: The Credé method can be used to improve bladder emptying. Decreasing fluid intake will not improve bladder emptying and may increase risk for urinary tract infection (UTI) and dehydration. The use of incontinence briefs and frequent toileting will not improve bladder emptying.

...34.A nurse is preparing a client newly diagnosed with multiple sclerosis for discharge home from a rehabilitation center. The client has been prescribed cyclophosphamide (Cytoxan) and methylprednisolone (Medrol). Which instruction will the nurse include in a teaching plan for the client?

"Avoid people with colds." The client should be taught to avoid individuals with any type of upper respiratory illness because these medications are immunosuppressive.

Which statement from the client with Guillain-Barré syndrome indicates that teaching about disease progression was effective?

"I may need a ventilator until the paralysis goes away." GBS is characterized by ascending paralysis that is not permanent. During the acute phase, if the paralysis affects the intercostal muscles, the client may require mechanical ventilation until the paralysis begins to descend. The client will not require a mechanical ventilator or wheelchair after recovery from the acute phase of the disease. There is no relationship of exacerbation of GBS to being around other people

Which statement indicates that the client has a correct understanding about recovery from Guillain-Barré syndrome?

"I will have to take things slowly for several months after I leave the hospital." Most clients make a full recovery from GBS. Recovery can take as long as 6 months to 2 years. Fatigue is a major lingering symptom for most of those diagnosed with this disorder. Clients are not permanently paralyzed. They are in an acute care environment during the acute phase of this disorder.

a.A nurse has instructed the client with myasthenia gravis to take drugs on time and to eat meals 45 to 60 minutes after taking the anticholinesterase drugs. The client asks why the timing of meals is so important. Which is the nurse's best response?

"This timing allows the drug to have maximum effect, so it is easier for you to chew, swallow, and not choke." The skeletal muscle weakness extends to the ability to chew and swallow. Clients who have myasthenia gravis are at risk for aspiration during meals. Timing the medication so that most of the meal is eaten when the drugs have produced their peak effect enables the client to chew and swallow more easily.

...

...

Huntington's disease is inherited in an autosomal-dominant pattern. Genetic testing is available to families in which a member has Huntington's disease. The availability of the testing has created some ethical conflicts.

...

Which statement by the client indicates understanding of treatment for pain related to Guillain-Barré syndrome?

..."A combination of morphine and distraction seems to help bring me relief right now." Typical pain from GBS is often not relieved by medication other than opiates. Distraction, repositioning, massage, heat, cold, and guided imagery may enhance the opiate effects.

A nurse is performing an assessment on a client who is suspected of having MG. The complaint made by the client that reflects a manifestation commonly seen in clients with this disease is

..."By the end of the day, my eyelids usually are drooping." The primary feature of MG is increasing weakness with sustained muscle contraction. After a period of rest the muscles regain their strength. Muscle weakness is greatest after exertion or at the end of the day. Ocular manifestations are most common, with ptosis or diplopia occurring in a majority of clients.

The client diagnosed with the Huntington gene but who has no symptoms asks for options related to family planning. Which is the nurse's best response?

..."Tell me more specifically what information you need about family planning so that I can direct you to the right information or health care provider." The presence of the Huntington gene means that the trait will be passed on to all offspring of the affected individual. Understanding options for contraception and conception (e.g., surrogate mother options) and implications for children may require the expertise of a genetic counselor or reproductive specialist.

Which statement by the client with a family history of Huntington's disease indicates that teaching about this disease was effective?

..."The disease progresses differently if inherited from the father." Huntington's disease is a hereditary disorder with an autosomal dominant pattern of transmission. The client who inherits the mutation from his or her father has an earlier onset and shorter life expectancy than the client who inherits the disease from the mother.

The client with relapsing-remitting multiple sclerosis asks why continuous treatment with interferon beta-1a (Avonex) is necessary. Which is the nurse's best response?

..."This medication will help decrease the number and severity of relapses." Interferon beta-1a is a biologic response modifier that is given IM once weekly to decrease the number and severity of relapses.

In discussing advanced directives, a client with ALS states that he does not want to be placed on a mechanical ventilator. Which is the nurse's best response?

..."What would you like to be done if you begin to have difficulty breathing?" ALS is an adult-onset upper and lower motor neuron disease, characterized by progressive weakness, muscle wasting, and spasticity, eventually leading to paralysis. Once muscles of breathing are involved, the client must include in the advanced directives what is to be done when breathing is no longer possible without intervention.

For which side effects in the client with Parkinson's disease who has been taking a combination carbidopa-levodopa drug (Sinemet) for 3 years will the nurse monitor?

...Abnormal movements Following 3 or more years of treatment, about one third of clients develop involuntary movements that are thought to be treatment-related.

The nurse assesses for which clinical manifestation in the client with MS of the relapsing-remitting type?

...Attacks becoming increasingly frequent The classic picture of relapsing-remitting MS is characterized by increasingly frequent attacks.

ImportaImportant self-care measures a nurse can teach a client with Parkinson's disease in order to prevent contractures and improve mobility include which of the following? (Select all that apply.)

...Bend over with your head over your toes to get out of chairs. Exercise first thing in the morning. Look up when you walk, not down at the floor. Clients with PD need to maintain mobility and prevent contractures. Options a, b, and d are important self-help measures. The client should use a wide-based gait. If it is too hard to get on the floor to exercise, the client should do exercises in bed.

A client with multiple sclerosis has been treated for 6 months with mitoxantrone (Novantrone). Which clinical manifestation alerts the nurse to an adverse effect of this medication?

...Crackles in the lungs Mitoxantrone (Novantrone) is an antineoplastic agent that can cause cardiotoxicity when used for long periods. Adverse effects are congestive heart failure and dysrhythmias.

The nurse correlates which pathophysiologic process to the client with a diagnosis of multiple sclerosis (MS)?

...Damage to the myelin sheath causes an inflammatory response. In MS, the myelin sheath is damaged, leading to an inflammatory response.

Which physical assessment finding does the nurse expect to observe in a client with myasthenia gravis?

...Difficulty or inability to perform the six cardinal positions of gaze The most common assessment finding in more than 90% of clients with myasthenia gravis is involvement of the extraocular muscles. The nurse observes for inability or difficulty with tests of extraocular function, such as the cardinal positions of gaze. Ptosis and incomplete eye closure also may be observed.

In evaluating laboratory data, the nurse correlates which results with the diagnosis of myasthenia gravis?

...Elevated acetylcholine receptor antibody levels Testing for acetylcholine receptor (AChR) antibodies is important, because 80% to 90% of clients with the disease have elevated AChR antibody levels.

Which nursing intervention will assist in preventing respiratory complications in the client with Parkinson's disease?

...Elevation of the back rest will help prevent aspiration. Maintaining the back rest elevation at greater than 30 degrees

Which teaching intervention is most appropriate for the client with Parkinson's disease?

...Fall precautions Rigidity in movement increases the risk of falls.

In planning discharge for the client with Parkinson's disease, the nurse collaborates with the physical therapist for which outcome?

...Maintaining physical strength and mobility Early in the disease process, collaborate with physical and occupational therapists to plan and implement a program to keep the client mobile and flexible by incorporating active and passive range-of-motion (ROM) exercises, muscle stretching, and activity.

A client presents with an acute exacerbation of multiple sclerosis. Which drug will the nurse be prepared to administer?

...Methylprednisolone (Medrol) Methylprednisolone is the drug of choice for acute exacerbations of the disease.

Which clinical manifestations would serve to alert the nurse to the early onset of MS?

...Nystagmus and ataxia Early signs and symptoms of MS include changes in motor skills, vision, and sensation.

For which motor changes in the client with Huntington's disease will the nurse monitor?

...Rapid hand movements with no purpose An imbalance between excitatory and inhibitory neurotransmitters leads to uninhibited motor movements, such as brisk, jerky, purposeless movements of the hands, face, tongue, and legs.

In providing discharge teaching to the client with Parkinson's disease who is taking monamine oxidase type B (MAO-B) inhibitors (MAOIs), the nurse instructs the client to avoid which foods or beverages? (Select all that apply.)

...Smoked ham Sausage Beer Red wine Teach clients taking MAOIs about the need to avoid foods, beverages, and drugs that contain tyramine, including aged, smoked, or cured foods and sausage. Remind them also to avoid red wine and beer to prevent severe headache and life-threatening hypertension.

The client with myasthenia gravis develops a sudden increase in weakness, accompanied by an increase in heart rate from 76 to 100 beats/min and an increase in blood pressure from 122/72 to 152/82 mm Hg. Which conclusion will the nurse reach from these findings?

...The client is experiencing myasthenic crisis. The client in myasthenic crisis experiences a rise in heart rate and blood pressure as well as an increase in muscle weakness.

The nurse recognizes which pathophysiologic feature as a hallmark of Guillain-Barré syndrome?

...The immune system destroys the myelin sheath.

A patient has a new prescription for levodopa (L-dopa) to control symptoms of Parkinson's disease. Which assessment data obtained by the nurse may indicate a need for a decrease in the dose?

...The patient's blood pressure is 90/46 mm Hg. Rationale: Hypotension is an adverse effect of L-dopa, and the nurse should check with the health care provider before giving the medication. Diarrhea, cough, and deep vein thrombosis are not associated with L-dopa use.

The nurse recognizes which pathophysiologic change in the client diagnosed with myasthenia gravis?

...There is a defect in the transmission of nerve impulses to the skeletal muscles. The major pathologic defect in myasthenia gravis is that nerve impulses are not transmitted to skeletal muscles at the neuromuscular junction

The nurse explains that the pathology of Huntington's disease involves

...an excess of the neurotransmitter dopamine. The degeneration of the caudate nucleus leads to a reduction in several neurotransmitters, including gamma-aminobutyric acid, acetylcholine, substance P, and metenkephalin, and their synthetic enzymes. This change leaves relatively higher concentrations of the other neurotransmitters, dopamine and norepinephrine

A client with MG began to experience a sudden worsening of her condition with difficulty in breathing. The nurse explains that this complication of MG is usually initially treated with

...an increased dose of anticholinesterase drugs. With myasthenic crisis, if an increase in the dosage of the anticholinesterase drug does not improve the weakness, endotracheal intubation and mechanical ventilation may be required. None of the other

The nurse identifies the nursing diagnosis of impaired physical mobility related to bradykinesia for a patient with Parkinson's disease. To assist the patient to ambulate safely, the nurse should

...instruct the patient to rock from side to side to initiate leg movement. Rationale: Rocking the body from side to side stimulates balance and improves mobility. The patient should initially be ambulated with assistance but might not require continual assistance with ambulation. The patient should maintain a wide base of support to help with balance. The patient should lift the feet and avoid a shuffling gait.

To prevent complications caused by a common problem of Huntington's disease, the nurse should

...pad wheelchairs and beds. Excessive movements and falling can cause injury in the client with Huntington's disease. Interventions include padding wheelchairs and beds, providing shin guards, and using gait belts for ambulation. Communication does become difficult and alternative forms of communication are appropriate before the client becomes completely demented, but this does not take priority over safety precautions. The client does not need an exercise regimen as the client is already hyperactive, and seizures do not occur.

When a client is admitted to the hospital with Guillain-Barré syndrome (GBS), the most important assessment the nurse should make is for

...respiratory difficulty. The two most dangerous features of GBS are respiratory muscle weakness and autonomic neuropathy involving both the sympathetic and the parasympathetic systems.

A hospitalized patient with myasthenia gravis (MG) has a nursing diagnosis of imbalanced nutrition: less than body requirements related to impaired swallowing. To promote nutrition, the nurse suggests that before meals the patient should avoid

...talking on the phone. Rationale: The same muscles are used for talking and swallowing, so the patient should avoid fatiguing the muscles of the mouth and throat before meals. The other activities will not affect the muscles used for chewing and swallowing.

To assist the client with Parkinson's disease to reduce tremor, the nurse suggests that the client

...tightly hold change in the pocket. Clasping change tightly in the pocket, using both hands to complete tasks, and sleeping on the tremorous side will help lessen the tremor.

A patient who has been taking bromocriptine (Parlodel) and benztropine (Cogentin) for Parkinson's disease is experiencing a worsening of symptoms. The nurse will anticipate that patient may benefit from

...use of levodopa (L-dopa)-carbidopa (Sinemet). Rationale: After the dopamine receptor agonists begin to fail to relieve symptoms, the addition of L-dopa with carbidopa can be added to the regimen. Complete drug withdrawal will result in worsening of symptoms. Anticholinergic therapy should be continued to help maintain the balance between the actions of dopamine and acetylcholine. Increasing the dose of bromocriptine will increase the risk for toxic effects.

A patient is being admitted for meningitis. The nurse is aware that the presenting signs and symptoms for bacterial and viral meningitis are the same, so the nurse must use laboratory data to differentiate between these acute processes. Which of the following is true concerning bacterial meningitis? In the spinal fluid, protein levels will be high and glucose levels will be low In the spinal fluid, the color will be clear In the spinal fluid, the protein and glucose levels will be low The spinal fluid will be bloody

1. In the spinal fluid, protein levels will be high and glucose levels will be low. Rationale: 1. With bacterial meningitis, the spinal fluid protein levels will be high and glucose levels will be low. 2. The spinal fluid will be clear in viral meningitis. 3. Protein levels will be high. 4. The spinal fluid will be bloody if there has been a bleed within the cranium. If bleeding occurred as a result of the puncture, the fluid would clear.

When planning educational opportunities for patient related to the psychosocial impact of spinal cord injury, the nurse understands that: The patients family will need ongoing education to help deal with the stress of any role changes associated with the injury The patient may exhibit any number of negative emotions about physical challenges, including elation and jubilation. The patient should be encouraged to discuss his feelings openly and to support the decisions made for him by the health care provider The patient is usually so preoccupied with the physical challenges faced after a spinal cord injury that psycho social problems rarely occur

1. The patient's family will need ongoing education to help them deal with the stress of any role changes associated with the injury. Rationale: 1. Role changes within the family are a very real possibility following spinal cord injury, and the family will require ongoing support and education to help deal with this change. 2. Elation and jubilation are not negative emotions; however, patients who experience spinal cord injury often do have negative feelings. They may exhibit denial, anger, grief, hopelessness, and depression. 3. Patients should be encouraged to openly discuss their feelings, but they should also be allowed the opportunity to make their own health care decisions when possible. 4. It is because of the physical challenges caused by the spinal cord injury that the patient usually faces significant psychosocial challenges as well.

Which of the following statements would the nurse recognize as best describing appropriate prophylaxis for some of the most common complications that a patient may experience following a spinal cord injury? (select all that apply) Use of an abdominal binder and compression stocking to decrease the peripheral venous pooling that can cause neurogenic shock Use of a Duplex Doppler ultrasound to dissolve deep vein thromboembolisms and reduce the likelihood of pulmonary embolus Use of a reclining wheelchair to gradually progress the patient into an upright position so as not to cause orthostatic hypotension Use of sound nutritional measures such as an increase caloric, protein, and micronutriant intake to promote bowel evacuation Learn the technique of straight catheterization to reduce the risk of urinary retention and decrease potential for urinary tract infections

1. Use of an abdominal binder and compression stockings to decrease the peripheral venous pooling that can cause neurogenic shock; 3. Use of a reclining wheelchair to gradually progress the patient into an upright position so as not to cause orthostatic hypotension; 4. Use of sound nutritional measures such as an increased caloric, protein, and micronutrient intake to promote bowel evacuation; 5. Learn the technique of straight catheterization to reduce the risk of urinary retention and decrease potential for urinary tract infections. Rationale: 1. The use of an abdominal binder and compression stockings does decrease blood pooling in the peripheral veins, but these interventions are used to help improve venous return to the heart and to prevent orthostatic hypotension as well. 2. A duplex Doppler ultrasound is a highly sensitive screening tool for detecting deep venous thrombosis. It does not dissolve clots. 3. A reclining wheelchair or tilt table gradually places the patient into an upright position, decreasing the development of orthostatic hypotension. 4. Good nutrition is important is aiding bowel evacuation, but increased caloric, protein, and micronutrient intake is used to aid in wound healing and the promotion of good skin integrity. Increased fiber and fluid intake would help prevent constipation. 5. Urinary retention and the potential for UTI is a major cause for concern. Teaching the skill of intermittent or straight catheterization is an appropriate intervention.

the clients diagnosed with a gunshot wound to the head assumes decorticate pOsturing when the nurse applies painful stimuli. Which assessment data obtained 3 hours later would indicate the client is improving? 1. Purposeless movement in response to painful stimuli. 2. flaccid paralysis in all four extremities. 3. Decerebrate posturing when painful stimuli are applied. 4. Pupils that are 6 millimeters in size and nonreactive to painful stimuli.

1. purposeless movement indicates that the clients cerebral edema is decreasing. The best motor responses purposeful movement, but purpose less movement indicates an improvement over the Decorticate movement, which, in turn, is an improvement over decerebrate movement or flaccidity.

Secondary prevention: Risk factor modification (4)

1. smoking cessation 2. diabetes control 3. aggressive cholesterol lowering 4. hypertension control

The nurse is teaching a class in the community about stroke prevention and is planning to include information about the pathophysiology of stroke. Which of the following would the nurse include as a process that can cause a stroke? (select all that apply) Plaque formation that alters the internal diameter of a cerebral artery Embolus that separates from a deep venous thrombosis in the left leg Thrombus formation as a result of plaque formation Lipohyalinosis, a vascular abnormality, caused by hypertension Cardiac dysrhythmia such as atrial fibrillation

1.Plaque formation that alters the internal diameter of a cerebral artery; 3.Thrombus formation as a result of plaque formation; 4.Lipohyalinosis, a vascular abnormality, caused by hypertension; 5. Cardiac dysrhythmia such as atrial fibrillation Rationale: 1. Plaque formation that alters the internal diameter in a cerebral artery is a common cause of ischemic stroke. 2. The fragmentation of a clot in the venous system of the leg cannot enter the arterial circulation of the brain unless there is a defect in the heart. 3. The plaque disrupts the integrity of the arterial lining, causing blood to enter the clot and ultimately forming a thrombus. 4. Lipohyalinosis is a process that results in a vascular abnormality in small vessels and is normally associated with hypertension. 5. Embolisms from cardiac sources are referred to as cardiogenic embolisms. The most common cause is atrial fibrillation, which accounts for almost 10% of all ischemic strokes irrespective of whether the atrial fibrillation is constant in nature or paroxysmal.

A patient is being seen in the neurology clinic and his wife report that he has a recent history of being unable to identify common objects by name, such as keys and glasses. The nurse realizes this impairment is characteristic of which of the following disorders? Multiple sclerosis Alzheimer's disease Parkinson's Disease Myasthenia Gravis

2. Alzheimer's disease Rationale: 1. Cognitive impairment is not typical with multiple sclerosis. 2. The scenario describes agnosia. A person with Alzheimer's disease will demonstrate multiple cognitive deficits, including aphasia, apraxia, agnosia, and disturbances with executive functioning. 3. Specific cognitive deficits are not typically seen in Parkinson's disease. 4. Cognitive impairment is not typical with myasthenia gravis.

the client diagnosed with a closed head injury is admitted to the rehabilitation department. Which medication order with the nurse question? 1. A subcutaneous anti coagulant. 2. An intravenous osmotic diuretics. 3. An oral anticonvulsant. 4. An oral proton pump inhibitor.

2. An Osmotic diuretic is ordered in the acute phase to help decrease cerebral edema, but this medication would not be expected to be ordered in a rehabilitation unit.

The nurse is caring for apatient with cerebral vasospasm associated with subarachnoid hemorrhage who is being treated with triple-H therapy. The nurse must continuousy assess for the complications by including? Assessing renal output to detect the onset of cerebral salt washing Assessing lung and heart sounds to detect fluid overload Detection of the nidus, which can warn of impending seizure onset Monitoring for focal seizure onset, as this warns of a dire complication

2. Assessing lung and heart sounds to detect fluid overload Rationale: 1. Cerebral salt wasting is a complication of subarachnoid hemorrhage not associated with vasospasm. 2. With hypervolemic therapy, plasma expanders may cause pulmonary compromise or congestive heart failure. 3. The nidus is a concentration of abnormal vessels located at the center of an arteriovenous malformation, not an assessment parameter. 4. Seizures are a common finding in clients with cerebral vasospasm and may be generalized or focal depending on the area of the brain involved.

The patient was involved in a motor vehicle collision with rear impact and is now experiencing severe neck pain and decreased upper extremity motor function, but there is normal motor function in the lower extremities. The patient complains of parasthesias in both the upper and lower extremities but greater in the arms. Based on these findings, the nurse suspects the patient may have which of the following types of injuries? Anterior cord syndrome Central cord syndrome Cauda equina syndrome Brown-Sequard syndrome

2. Central cord syndrome Rationale: 1. Anterior cord syndrome is caused by direct insult to the anterior portion of the spinal cord. Compression or hyperextension injury may produce this problem, but paralysis and loss of pain and temperature sensations are evident below the level of the injury. Also, light touch, vibration, and proprioception are preserved. 2. Central cord syndrome is often caused by hyperextension injuries, such as a rear-impact motor vehicle collision. This syndrome is caused by central spinal cord damage, which, in turn, causes damage to the more centrally located fibers leading to the upper extremities. The fibers leading to the lower extremities are more laterally located, producing less motor and sensory problems in the lower versus upper extremities. 3. Cauda equina syndrome is caused by compression of the lumbar nerve roots below the first lumbar vertebra. The associated signs and symptoms can vary based on the specific nerve root involved, but they often involve motor and sensory loss to the pelvic organs and lower extremities. 4. Brown-Séquard syndrome is usually the result of penetrating trauma that causes hemisection of the spinal cord. The ipsilateral side experiences paralysis and proprioception loss, and the contralateral side experiences pain and temperature sensation loss.

When describing the ischemic cascade to a group of students, the nurse educator uses which understanding when explaining the phenomenon? Tissues that surround the necrotic area have themselves undergone anoxia Ischemic cascade further extends the area of infarction This tissue may be reperfused, but its ability to function has been permanently impaired The penumbra of a stroke is the necrotic core

2. Ischemic cascade further extends the area of infarction. Rationale: 1. Tissues surrounding the necrotic area have undergone tissue hypoxia and may be viable if perfusion is restored before anoxia occurs. 2. The ischemic cascade further propagates cerebral edema, cerebral ischemia, cerebral infarction, and cell death. 3. Reperfusion injury results in further injury to already damaged tissue that is compromised but viable, and it can present as conversion to hemorrhagic stroke (Leibeskind, 2010). 4. The penumbra is the area surrounding the necrotic core.

the resident in a long term care facility Fell during the previous shift and has a laceration in the occiptal area that has been closed with steri strips. Which signs or symptoms would warrant transferring the resident to the emergency department?1. A 4 centimeters area of bright red drainage on the dressing. 2. A weak pulse, shallow respirations, and cool pale skin. 3. pupils that are equal, react to light, and accommodate. 4. Complaints of a headache that's resolved with medication.

2. These signs and symptoms indicate increased intracranial pressure from cerebral edema secondary to the fall, and they require immediate attention.

The nursing management for the patient with increased intracranial pressure involves assessment of respiratory, cardiovascular, metabolic, and neurological processes. Desired parameters to avoid complications include which of the following (select all that apply) Pulse oximetry should be used with supplemental oxygen to maintain saturations of 90% or greater Normocapnia levels (PaCO2 levels of 35 to 40 mmHg) Euglycemic levels of 80 to 120 mg/dL Normothermia and interventions for body temperatures >100.6 F rectally Hypothermia with intervention to maintain temperatures <95 F rectally

2.Normocapnia levels (PaCO2 levels of 35-40 mmHg); 3.Euglycemic levels of 80- 120 mg/dL; 4.Normothermia and interventions for body temperatures >100.6°F rectally Rationale: 1. Pulse oximetry and supplemental oxygen should be used to maintain saturations of 95% or greater. 2. Normocapnia levels (PaCO2 levels of 35-40 mmHg) are best for the brain. 3. It is optimal to maintain euglycemic levels of 80-120 mg/dL, because the brain is adversely impacted by both hypoglycemia and hyperglycemia, 4. It is optimal to maintain normothermia and to intervene for body temperatures >99.5°F rectally.

A patient is admitted with a gradual increase in intracranial pressure due to an enlarging tumor. Early signs of increased intracranial pressure may include which of the following? Chances in the optic nerve (CNVIII) that may result in ringing in the ears Decorticate movement observable in the arms Complains of a "different" headache and projectile vomiting Cushing's triad

3. Complaints of a "different" headache and projectile vomiting Rationale: 1. Changes in cranial nerve III, the oculomotor nerve, result in changes in papillary response and are early signs of ICP. 2. Decorticate movements are a late sign. 3. Complaints of a "different" headache and projectile vomiting may be an early indication of increasing ICP. 4. Cushing's triad is a late sign of ICP.

The nurse is teaching the family of a patient with a ruptured cerebral aneurysm. When deciding about treatment options of endovascular coiling or neurosurgical clipping, the choice is based upon which of the following factors? Endovascular coiling is associated with a longer length of hospital stay Neurosurgical clipping is associated with a higher survival and lower morbidity rate one year after the procedure Endovascular coiling is associated with improved outcomes with better potential that the patient may be able to return to their activities of normal living and work earlier than patients who had undergone neurosurgical clipping There is little difference in either treatment option other than the availability of an interventional neuroradiologist

3. Endovascular coiling is associated with the patient being able to return to his activities of normal living and work earlier than patients who had undergone neurosurgical clipping. Rationale: 1. Endovascular coiling is associated with a shorter, not longer length of hospital stay. 2. Endovascular coiling, not neurosurgical clipping, is associated with a higher survival and lower morbidity rates 1 year after the procedure. 3. Improved outcomes are achieved with endovascular coiling as compared to surgery. 4. Although the availability of an interventional neuroradiologist is important, if the patient is a candidate for endovascular coiling, this is the option of choice.

A student nurse asks the nurse to explain a physician note that refers to the patient showing evidence of demyelinzation of nerve fibers and plaque formation. The nurse explains to the students that this patient is likely experiencing: Alzheimer's disease Myasthenia gravis multiple sclerosis Parkinson's disease

3. Multiple sclerosis. Rationale: 1. The brain of a patient with Alzheimer's disease shows a profusion of amyloid plaques and neurofibrillary tangles, which form in the hippocampus and other parts of the brain critical to memory. 2. Myasthenia gravis is a disease of muscle weakness as the result of dysfunction at the neuromuscular junction where the transmission of nerve impulses is blocked. 3. Multiple sclerosis is a neuroimmunologic disease that affects myelin, the protective sheath surrounding nerve fibers. Demyelinating lesions or plaques form along nerve fibers in the brain and spinal cord, producing symptoms related to the location of damage. 4. Parkinson's disease is a disease of poor dopamine production, which leads to difficulty with movement, tremor, rigidity, and difficulty maintaining posture.

the nurse is caring for the following clients. Which client what the nurse assess first after receiving the shift report? 1. The 22 year old male client diagnosed with a concussion who is complaining someone is waking him up every 2 hours. 2. The 36 year old female client admitted with complaints of left sided weakness who is scheduled for an MRI scan. 3. The 45-year-old client admitted with blunt trauma to the head after a motorcycle accident who has a Glasgow Coma Scale score of 6. 4. The 62-year-old client diagnosed with CVA who has expressive aphasia.

3. The Glasgow Coma Scale is used to determine a client's response to stimuli such asEye opening response, best verbal response, and best motor response secondary to a neurological problem scores range from 3 which is a deep coma to 15 which is intact neurological function. A client with a score of 6 should be assessed first.

the client with a closed head injury has clear fluid draining from the nose. Which action should the nurse implement first? 1. Notify the health-care provider immediately. 2. Prepare to administer an antihistamine. 3. Test the drainage for presence of glucose. 4. Place 2x2 Gauze under the nose to collect drainage.

3. The presence of glucose in drainage from the nose or ears indicates cerebrospinal fluid and the HCP should be notified immediately.

The nurse is caring for a client with a diagnosis of ischemic stroke. Which of the following statements should the nurse include when discussing the patient's care with a student nurse assigned to the patient to enhance the student's understanding of the patient's diagnosis? Ischemic strokes are hemorrhagic in nature The signs and symptoms of ischemic stroke resolve without permanent disability Ischemic strokes are commonly caused by atherosclerosis Patients experiencing ischemic stroke are usually younger

3. The strokes are commonly caused by atherosclerosis. Rationale: 1. Most ischemic strokes are not hemorrhagic in nature. 2. The symptoms of ischemic stokes may result in permanent disability. 3. Eighty percent of all strokes are ischemic in nature and most are caused by atherosclerosis. Hemorrhagic strokes are commonly associated with hypertension. 4. Patients experiencing ischemic strokes are generally older than those experiencing hemorrhagic strokes. Cognitive level: Application

A patient with Parkinson's disease tells the nurse that she's worried about her 80 year old husband's ability to care for her as she gets older. Which of the following would be an appropriate response for the nurse to make to this patient? "That's not something that you should be worried about." "You are so healthy now that I'm sure you won't be a burden to your husband." "Would you like to talk about things that can help you and your husband?" "I'll get the doctor for you so you can talk with him about your concerns."

3. Would you like to talk about things that can help you and your husband? Rationale: 1. This response does not address the patient's concerns. 2. This response dismisses the patient's very realistic concern. 3. People can live with Parkinson's disease for many years. The chronic and progressive nature of the disease can significantly impact older, spousal caregivers, who may not have the physical strength to handle the weight of a patient with limited mobility. Caregiver stress and burden have been shown to increase as the disease progresses. The nurse should encourage the patient to talk about things that can help both the patient and the husband. 4. Suggesting the patient discuss the concern with her physician is also not an appropriate response. The nurse can discuss this concern with the patient.

The nurse is teaching a group of nursing students about the causes of spinal cord injury. Which of the following statements made by one of the students indicates a clear understanding of the topic? "Forward falls in which the chin or forehead strikes the ground might produce compression spinal cord injuries." "When axial pressure is loaded on the spine, the resultant rotational injuries might include fracture of the vertebral body." "Hyperextension injuries can cause anterior vertebral body fractures and posterior spinal column facet fracture and dislocation". "A sudden deceleration injury, such as a head-on motor vehicle collision, is frequently the cause of hyperflexion injuries."

4. "A sudden deceleration injury, such as a head-on motor vehicle collision, is frequently the cause of hyperflexion injuries." Rationale: 1. Hyperextension injuries are caused by a sudden forceful extension of the head and neck, for instance, during a forward fall where the chin or forehead strikes the ground. 2. Axial pressure loaded onto the spine may cause compression injuries, which include vertebral body fractures and spinal cord compression. 3. Rotational injuries are associated with lateral flexion or rotation of the spine, causing posterior ligaments to rupture and facets to dislocate and fracture. 4. Sudden deceleration injury, such as a head-on motor vehicle collision, is often the cause of hyperflexion injuries. These injuries can cause anterior vertebral body fractures and posterior spinal column facet fracture and dislocation.

4. A female client with Guillain-Barré syndrome has paralysis affecting the respiratory muscles and requires mechanical ventilation. When the client asks the nurse about the paralysis, how should the nurse respond? a. "You may have difficulty believing this, but the paralysis caused by this disease is temporary." b. "You'll have to accept the fact that you're permanently paralyzed. However, you won't have any sensory loss." c. "It must be hard to accept the permanency of your paralysis." d. "You'll first regain use of your legs and then your arms."

4. Answer A. The nurse should inform the client that the paralysis that accompanies Guillain-Barré syndrome is only temporary. Return of motor function begins proximally and extends distally in the legs.

A patient's vital signs are pulse 80, respirations 24, and BP of 124/60 mm Hg and cardiac output is 4.8 L/min. What is the patient's stroke volume? ____________________

60 mL rationale: Stroke volume = cardiac output/heart rate

The patient had an acute ischemic stroke 4 hours ago and has an elevated blood pressure. What action should you take? A. Document the findings because the increased pressure is needed to perfuse the brain. B. Administer an antihypertensive medication to prevent additional damage. C. Hyperventilate the patient to cause vasodilatation. D. Teach patient about a low sodium diet.

A After a stroke, temporary hypertension is needed to perfuse the area of swelling. No treatment is done unless the pressure is above 220/110 mm Hg in the first few hours. Aggressive lowering of blood pressure is not done, because if the pressure drops, it can prevent regional perfusion and lead to local tissue damage. Hyperventilation is done if hypercapnia is identified, but it is not prophylactic.

A client who has a herniated disk is being discharged after a percutaneous endoscopic discectomy. Which postprocedure instructions does the nurse provide before discharge? a. "You should begin an exercise routine which includes walking every day." b. "You must sleep in a supine position until the bandage is removed." c. "You may feel numbness or tingling in the legs for 24 hours." d. "You will need to wear a lumbar brace for 1 week."

A After this minimally invasive surgery, clients typically go home the same day or the day after surgery. Clients should be taught to begin the prescribed exercise program immediately after discharge, which includes walking every day. The client should not be restricted to one sleeping position. Clients generally have less pain with this procedure and do not experience numbness or tingling. The client may have a clear or gauze dressing but will not need to wear a lumbar brace.

During admission of a patient with a severe head injury to the emergency department, you place the highest priority on assessment of A. patency of airway. B. presence of a neck injury. C. neurologic status with the Glasgow Coma Scale. D. cerebrospinal fluid leakage from the ears or nose.

A An initial priority in the emergency management of a patient with a severe head injury is for you to ensure that the patient has a patent airway.

When assessing a patient with a traumatic brain injury, you notice uncoordinated movement of the extremities. How would you document this? A. Ataxia B. Apraxia C. Anisocoria D. Anosognosia

A Ataxia is a lack of coordination of movement, possibly caused by lesions of sensory or motor pathways, cerebellar disorders, or certain medications.

The patient with AD is significantly more agitated and restless today. What action should you take first? A. Look at urine characteristics. B. Assess room temperature. C. Reassure the patient that she is safe. D. Allow the patient an area to pace in.

A Behavioral problems are often the patient's way of responding to a precipitating factor. Initially, the physical status should be evaluated. Urinary problems and pneumonia are the most common causes. Assessing the environment and reassuring the patient should then take place. The behavior should not be allowed to continue without trying to assess a cause for it. Reference: 1529

You approach the patient with AD to provide her bath. The patient states, "Go away! I'm not taking a bath." What is your initial response? A. Leave and reapproach in a few minutes. B. Ask the patient why she feels that way. C. Inform the patient that the physician will be notified. D. Obtain additional help and proceed with the bath.

A Behavioral problems occur in about 90% of patients with AD. They can respond to redirection, reapproach, distraction, and reassurance. Persons with AD have limited verbal skills and are not able to respond to "why" questions. You should not threaten with restraints or to call the physician. It is always preferable to try a nonthreatening approach first, especially for something that is not absolutely essential.

The aim of biofeedback in the patient who experiences headaches is to A. relax muscles and increase peripheral blood flow. B. raise the blood pressure. C. decrease the pulse rate. D. relax the diaphragm and increase the oxygen saturation.

A Biofeedback involves the use of physiologic monitoring equipment to give the patient information regarding muscle tension and peripheral blood flow (e.g., skin temperature of the fingers). The patient is trained to relax the muscles and raise the finger temperature, and is given reinforcement (operant conditioning) in accomplishing these changes. Reference: 1488

The nurse is assessing a client with a cluster headache. Which clinical manifestation does the nurse expect to find? a. Ipsilateral tearing of the eye b. Exophthalmos c. Abrupt loss of consciousness d. Neck and shoulder tenderness

A Cluster headache is usually accompanied by ipsilateral tearing, rhinorrhea or nasal congestion, ptosis, eyelid edema, facial sweating, and miosis. The other manifestations are not associated with cluster headaches.

A 50-year-old man complains of recurring headaches. He describes these as sharp, stabbing, and located around his left eye. He also reports that his left eye seems to swell and get teary when these headaches occur. Based on this history, you suspect that he has A. cluster headaches. B. tension headaches. C. migraine headaches. D. medication overuse headaches.

A Cluster headaches involve repeated headaches that can occur for weeks to months at a time, followed by periods of remission. The pain of cluster headache is sharp and stabbing; the intense pain lasts from a few minutes to 3 hours. Headaches can occur every other day and as often as 8 times a day. The clusters occur with regularity, usually occurring at the same time each day, during the same seasons of the year. Typically a cluster lasts 2 weeks to 3 months, and then the patient goes into remission for months to years. The pain is generally located around the eye, radiating to the temple, forehead, cheek, nose, or gums. Other manifestations may include swelling around the eye, lacrimation (tearing), facial flushing or pallor, nasal congestion, and constriction of the pupil. During the headache, the patient is often agitated and restless, unable to sit still or relax. Reference: 1487-1488

An important nursing diagnosis for the patient with amyotrophic lateral (ALS) is A. impaired gas exchange related to paralysis of respiratory muscles. B. hypothermia related to impaired regulation by the hypothalamus. C. impaired memory related to cognitive changes. D. dysreflexia related to loss of sympathetic nervous tone

A Death usually results from respiratory infection secondary to compromised respiratory function. Reference: 1515

The patient reports falling when he his foot got "stuck" on a crack in the sidewalk, hitting his head when he fell, and "passing out". The paramedics found the patient walking at the scene and talking before transporting the patient to the hospital. In the emergency department, the patient starts to lose consciousness. This is a classic scenario for which complication? A. Epidural hematoma B. Subdural hematoma C. Subarachnoid bleed D. Diffuse axial inju

A Epidural hematoma often results from a linear fracture crossing a major artery in the dura. The classic sign is an initial period of unconsciousness at the scene and a brief lucid interval followed by a decrease in LOC. A subdural hematoma often results from injury to the brain and veins and develops more slowly. The classic sign or symptom of subarachnoid hemorrhage is a patient describing "the worst headache of my life." Diffuse axonal injury is widespread axonal damage occurring after a traumatic brain injury.

The nurse is providing health education at a community center. Which instruction does the nurse include as part of client education for the prevention of low back pain? a. "Participate in a regular exercise program." b. "Purchase a soft mattress for sleeping comfort." c. "Wear high-heeled shoes only for special occasions." d. "Keep your weight within 20% of your ideal body weight."

A Exercise can strengthen back muscles, reducing the incidence of low back pain. The other options will not prevent low back pain.

Which nursing action should be implemented in the care of a patient who is experiencing increased ICP? A. Monitor fluid and electrolyte status astutely. B. Position the patient in a high-Fowler's position. C. Administer vasoconstrictors to maintain cerebral perfusion. D. Maintain physical restraints to prevent episodes of agitation.

A Fluid and electrolyte disturbances can have an adverse effect on ICP and must be vigilantly monitored. The head of the patient's bed should be kept at 30 degrees in most circumstances, and physical restraints are not applied unless absolutely necessary. Vasoconstrictors are not typically administered in the treatment of ICP.

Which measure should you prioritize when providing care for a patient with a diagnosis of multiple sclerosis (MS)? A. Vigilant infection control and adherence to standard precautions B. Careful monitoring of neurologic vital signs and frequent reorientation C. Maintenance of a calorie count and hourly assessment of intake and output D. Assessment of blood pressure and monitoring for signs of orthostatic hypotension

A Infection control is a priority in the care of patients with MS since infection is the most common precipitator of an exacerbation of the disease. Decreases in cognitive function are less likely and MS does not typically result in hypotension or fluid volume excess or deficit. Reference: 1504

The pathophysiology of multiple sclerosis (MS) is related to an attack on the A. myelin sheaths on various neurons. B. goblet cells of the muscles. C. lining of the blood vessels. D. lack of acetylcholine at the synaptic junction.

A Initially the myelin sheaths of the neurons in the brain and spinal cord are attacked. Early in the disease the myelin sheath is damaged. Reference: 1501

Computed tomography of a 68-year-old patient's head reveals that he has experienced a hemorrhagic stroke. Which option is a nursing priority intervention in the emergency department? A. Maintenance of the patient's airway B. Positioning to promote cerebral perfusion C. Control of fluid and electrolyte imbalances D. Administration of tissue plasminogen activator (tPA)

A Maintenance of a patent airway is the priority in the acute care of a patient with a hemorrhagic stroke, and it supersedes the importance of fluid and electrolyte imbalance and positioning. Use of tPA is contraindicated in hemorrhagic stroke.

Why are the data regarding mobility, strength, coordination, and activity tolerance important for you to obtain? A. Many neurologic diseases affect one or more of these areas. B. Patients are less able to identify other neurologic impairments. C. These are the first functions to be affected by neurologic disease. D. Aspects of movement are the most important function of the nervous system.

A Many neurologic disorders can cause problems in the patient's mobility, strength, and coordination. These problems can result in changes in the patient's usual activity and exercise patterns.

The nurse is taking the health history of a client suspected of having bacterial meningitis. Which question is most important for the nurse to ask? a. "Do you live in a crowded residence?" b. "When was your last tetanus vaccination?" c. "Have you had any viral infections recently?" d. "Have you traveled out of the country in the last month?"

A Meningococcal meningitis tends to occur in outbreaks. It is most likely to occur in areas of high-density population, such as college dormitories, prisons, and military barracks. The other questions do not identify risk factors for bacterial meningitis.

A patient's eyes jerk while the patient looks to the left. How do you record this finding? A. Nystagmus B. Cranial nerve VI palsy C. Oculocephalia D. Ophthalmic dyskinesia

A Nystagmus is fine, rapid jerking movements of the eyes.

The nurse is teaching a client who has a spinal cord injury how to prevent respiratory problems at home. Which statement indicates that the client correctly understands the teaching? a. "I will use my incentive spirometer every 2 hours while I'm awake." b. "I will not drink thick fluids to prevent choking." c. "I will take cough medicine to prevent excessive coughing." d. "I will position myself on my right side so I don't aspirate."

A Often, the person with a spinal cord injury will have weak intercostal muscles and is at higher risk for developing atelectasis and stasis pneumonia. Using an incentive spirometer every 2 hours helps the client expand her or his lungs more fully and prevents atelectasis. Clients should drink fluids that they can tolerate; usually thick fluids are easy to tolerate. The client should be encouraged to cough and clear secretions. Clients should be placed in high Fowler's position to prevent aspiration.

For which patient should you prioritize an assessment for depression? A. A patient in the early stages of AD B. A patient who is in the final stages of AD C. A patient experiencing delirium resulting from dehydration D. A patient who has become delirious after an atypical drug response

A Patients in the early stages of AD are particularly susceptible to depression, because the patient is aware of his or her cognitive changes and the expected disease trajectory. Delirium is typically a short-term health problem that does not typically pose a heightened risk of depression. Reference: 1527

After you administer a dose of risperidone (Risperdal) to a patient with delirium, which intended effect of the medication do you assess for? A. Lying quietly in bed B. Alleviation of depression C. Reduction in blood pressure D. Disappearance of confusion

A Risperidone is an antipsychotic drug that reduces agitation and produces a restful state in patients with delirium. However, it should be used with caution

The priority nursing intervention for the patient with myasthenia gravis who is receiving pyridostigmine (Mestinon) includes A. timing drug administration so that chewing and swallowing are enhanced. B. withholding the drug if muscle function improves. C. assessing for constipation and paralytic ileus. D. monitoring renal and hepatic function.

A Scheduling doses of drugs so that peak action is reached at mealtime may make eating less difficult. Reference: 1513

You would correctly identify the advantage of Sinemet in the treatment of Parkinson's disease is its ability to A. restore deficient dopamine. B. increase the effect of levodopa. C. block the breakdown of dopamine. D. block uptake of catecholamines.

A Sinemet is the preferred drug because it also contains carbidopa, an agent that inhibits the enzyme dopa-decarboxylase in the peripheral tissues. Dopa-decarboxylase breaks down levodopa before it reaches the brain. The net result of the combination of levodopa and carbidopa is that more levodopa reaches the brain, and therefore less drug is needed. Reference: 1508

ou recognize that status epilepticus is a medical emergency because A. seizures continue without a return of consciousness. B. fractures of a limb may occur. C. urinary fecal incontinence may occur. D. heart rate becomes bradycardic.

A Status epilepticus is a state of continuous seizure activity or a condition in which seizures recur in rapid succession without return to consciousness between seizures. It is the most serious complication of epilepsy and is a neurologic emergency. Status epilepticus can occur due to any type of seizure. During repeated seizures the brain uses more energy than can be supplied.

Magnetic resonance imaging (MRI) has revealed a brain tumor in a patient. You recognize the patient's likely need for which treatment modality? A. Surgery B. Chemotherapy C. Radiation therapy D. Pharmacologic treatment

A Surgical removal is the preferred treatment for brain tumors.

The patient is diagnosed with a brain tumor. Which option is the correct understanding of the preferred treatment? A. Surgical removal is preferred, even if the tumor is not malignant. B. Chemotherapy is a common and effective treatment. C. Stereotactic radiosurgery is the preferred treatment. D. A large dose of intravenous steroid therapy is preferred.

A Surgical removal is the preferred treatment. It can reduce tumor mass (decreasing intracranial pressure [ICP]), provides relief of symptoms, and extend survival time. Even a benign mass has a malignant effect by taking up space. Traditional chemotherapy effectiveness is limited because of the blood-brain barrier, tumor cell heterogeneity, and tumor cell drug resistance. Stereotactic radiosurgery delivers a high, concentrated dose of radiation precisely directed and is used when conventional surgery has failed or is not an option. Corticosteroids are not an integral part of therapy, but are used to control complications of radiation therapy.

When administering a Tensilon test to a patient with a possible diagnosis of myasthenia gravis, you would realize the test is positive if the patient A. reports improved muscle strength. B. demonstrates improved cognition. C. experiences a surge of energy. D. exhibits enhanced vision.

A Tensilon test in a patient with MG reveals improved muscle contractility after intravenous injection of the anticholinesterase agent edrophonium chloride (Tensilon). (Anticholinesterase blocks the enzyme acetylcholinesterase.) Reference: 1512

What is the standard to evaluate the degree of impaired consciousness for a patient with an acute head trauma? A. Best eye opening, verbal response, and motor response B. National Institutes of Health (NIH) Stroke Scale C. Romberg test D. Widening pulse pressure, bradycardia, and respirations

A The Glasgow Coma Scale (GCS) is a standardized tool used to assess the degree of impaired consciousness, and it consists of three components. The NIH stroke scale is used for a suspected stroke and includes other components of cranial nerve assessment, motor testing, and sensory testing. The Romberg test measures balance and is used for suspected cerebellar dysfunction. The components in the last option are Cushing's triad and an indication of increased ICP, not LOC.

What is a major goal of treatment for the patient with AD? A. To maintain patient safety B. To maintain or increase body weight C. To return to a higher level of self-care D. To enhance functional ability over time

A The overall goals are that the patient with AD will (1) maintain functional ability for as long as possible, (2) be maintained in a safe environment with a minimum of injuries, (3) have personal care needs met, and (4) have dignity maintained. You should emphasize patient safety while planning and providing nursing care. Reference: 1527

What is the cause of the clinical manifestations of Parkinson's disease? A. Decreased levels of dopamine B. Decreased levels of acetylcholine C. Increased levels of angiotensinogen D. Increased levels of relaxin

A The pathologic process of PD involves degeneration of the dopamine-producing neurons in the substantia nigra of the midbrain. Reference: 1506

What is a key distinction of delirium compared with dementia? A. Sudden cognitive impairment B. Confusion related to time C. Distorted perceptions D. Worse in the morning

A The person with delirium is more likely to exhibit sudden cognitive impairment, disorientation, or clouded sensorium. Dementia is usually insidious. Delirium is rapid, often occurring at night. Confusion related to time and distorted perceptions are true for both. Depression is characterized by worse symptoms in the morning. Reference: 1533-1534

The patient relates to you that the health care provider suggested the patient may be diagnosed with restless legs syndrome (RLS). The patient asks you about the initial symptoms of RLS. You would identify which of the following as initial manifestation(s) of RLS? A. Numbness and tingling in the legs B. Periodic inability to control leg movements C. Cramp-like pain in the thighs D. Redness of the lower legs.

A The severity of RLS sensory symptoms ranges from infrequent minor discomfort (paresthesias, including numbness, tingling, and "pins and needles" sensation) to severe pain. Sensory symptoms often appear first and are manifested as an annoying and uncomfortable (but usually not painful) sensation in the legs. The sensation is often compared with the sensation of bugs creeping or crawling on the legs. Reference: 1514

Which statement is true for a patient who has pathology in Wernicke's area of the cerebrum? A. Receptive speech is affected. B. The parietal lobe is involved. C. Sight processing is abnormal. D. An abnormal Romberg test is present.

A The temporal, not parietal, lobe contains the Wernicke area, which is responsible for receptive speech and integration of somatic, visual, and auditory data. Sight processing occurs in the occipital lobe. The Romberg test is used to assess the position sense of the lower extremities.

A classic sign of an absence (petit mal) seizure is A. blank stare. B. frequent smacking of lips. C. urinary incontinence. D. asking the same question over and over.

A The typical clinical manifestation is a brief staring spell that lasts only a few seconds, so it often occurs unnoticed. There may be an extremely brief loss of consciousness. Reference: 1493

Which statement by the wife of a patient with AD demonstrates an accurate understanding of her husband's medication regimen? A. "I'm really hoping his medications will slow down his mental losses." B. "We're both holding out hope that this medication will cure his disease." C. "I know that this won't cure him, but we learned that it may prevent a bodily decline while he declines mentally." D. "I learned that if we are vigilant about his medication schedule, he may not experience the physical effects of his disease."

A There is no cure for AD, and drug therapy aims at improving or controlling the decline in cognition. Medications do not directly address the physical manifestations of AD. Reference: 1525

The patient says, "The smoodle pinkered, and I want him square, and I want to plunthery him." What impairment do you suspect? A. Wernicke's aphasia B. Broca's aphasia C. Dysarthria D. Aphasia

A Wernicke's aphasia is damage to the left temporal lobe, although it can result in damage to the right lobe. The patient may speak in long sentences that have no meaning, add unnecessary words, or even create words. In Broca's aphasia, the patient is able to understand but speaks only in short phrases that are difficult to produce. Dysarthria is a disturbance in the muscular control of speech. Impairments may involve pronunciation, articulation, and phonation. This condition does not affect meaning or comprehension of language but does affect the mechanics of speech. Aphasia is a total loss of comprehension and use of language or total inability to communicate.

The nurse is planning care for a client with epilepsy. Which precautions does the nurse implement to ensure the safety of the client while in the hospital? (Select all that apply.) a. Have suction equipment at the bedside. b. Place a padded tongue at the bedside. c. Permit only clear oral fluids. d. Keep bed rails up at all times. e. Maintain the client on strict bedrest. f. Ensure that the client has IV access.

A, D, F

The patient taking topiramate (Topamax) for prophylactic treatment of migraine headache should be instructed to (select all that apply) A. not abruptly discontinue the drug. B. maintain adequate fluid intake. C. take the medication with food. D. avoid physical activities that cause exertion. E. discontinue the drug if it is ineffective after two weeks.

A,B Instruct patient to not abruptly discontinue as this may cause seizures and take adequate fluid intake to decrease risk of renal stone development. Topiramate must be used for 2 to 3 months to determine its effectiveness.

A young woman has been diagnosed with relapsing-remitting multiple sclerosis and interferon β-1b (Betaseron) has been prescribed. You determine that teaching about the drug has been successful when the patient says (select all that apply) A. "I should avoid direct sunlight and use sunscreen and protective clothing when out of doors." B. "I will need to rotate injection sites with each dose I inject." C. "I should report any depression or suicidal thoughts that develop." D. "Because this drug is a corticosteroid, I should reduce my sodium intake to prevent edema." E. "Flulike symptoms are indicative of a significant side effect and should be reported immediately."

A,B,C Interferon is an immunomodulator, not a corticosteroid. The patient should rotate injection sites with each dose and the patient should watch for depression and suicidal ideation. The patient should wear sunscreen and protective clothing while exposed to sun. The patient should also be aware that flu-like symptoms are common following initiation of therapy. Reference: 1502-1503

You are caring for a patient admitted for evaluation and surgical removal of a brain tumor. You plan interventions for this patient based on knowledge that brain tumors can lead to which complications (select all that apply)? A. Vision loss B. Cerebral edema C. Pituitary dysfunction D. Parathyroid dysfunction E. Focal neurologic deficits

A,B,C,E Depending on the location, brain tumors can have a wide variety of clinical manifestations, including vision loss and focal neurologic deficits. Tumors that put pressure on the pituitary can lead to dysfunction of the gland. As the tumor grows, clinical manifestations of increased ICP and cerebral edema can appear. Parathyroid glands are not regulated by the cerebral cortex or the pituitary.

he classic symptoms of Parkinson's disease include (select all that apply) A. tremor. B. rigidity. C. loss of balance. D. bradykinesia. E. nystagmus.

A,B,D The classic manifestations of PD often include tremor, rigidity, and bradykinesia, which are often called the triad of PD. Reference: 150

You are discharging a patient admitted with a transient ischemic attack (TIA). For which medications do you expect to provide discharge instructions (select all that apply)? A. Clopidogrel (Plavix) B. Enoxaparin (Lovenox) C. Dipyridamole (Persantine) D. Enteric-coated aspirin (Ecotrin) E. Tissue plasminogen activator (tPA)

A,C,D Aspirin is the most frequently used antiplatelet agent. Other drugs used to prevent clot formation include clopidogrel (Plavix), ticlopidine (Ticlid), dipyridamole (Persantine), combined dipyridamole and aspirin (Aggrenox), and anticoagulant drugs, such as oral warfarin (Coumadin). The tPA is a fibrinolytic medication used to treat acute ischemic stroke, not prevent TIAs.

You are providing care for a patient who has been admitted to the hospital with a head injury who requires regular neurologic vital signs. Which assessments are components of the patient's score on the Glasgow Coma Scale (select all that apply)? A. Eye opening B. Abstract reasoning C. Best verbal response D. Best motor response E. Cranial nerve function

A,C,D The three dimensions of the Glasgow Coma Scale are eye opening, best verbal response, and best motor response.

What are the common psychosocial reactions of the patient to the stroke (select all that apply)? A. Depression B. Disassociation C. Intellectualization D. Sleep disturbances E. Denial of the severity of the stroke

A,D,E The patient with a stroke may experience many losses, including sensory, intellectual, communicative, functional, role behavior, emotional, social, and vocational losses. Some patients experience long-term depression and symptoms such as anxiety, weight loss, fatigue, poor appetite, and sleep disturbances. The time and energy required to perform previously simple tasks can result in anger and frustration. Frustration and depression are common in the first year after a stroke. A stroke is usually a sudden, extremely stressful event for the patient, caregiver, family, and significant others. The family is often affected emotionally, socially, and financially, as well as changing roles and responsibilities. Reactions vary considerably but may involve fear, apprehension, denial of the severity of the stroke, depression, anger, and sorrow.

A patient has ICP monitoring with an intraventricular catheter. A priority nursing intervention for the patient is a. aseptic technique to prevent infection b. constant monitoring of ICP waveforms c. removal of CSF to maintain normal ICP d. sampling CSF to determine abnormalities

A. Aseptic technique to prevent infection- An intraventricular catheter is a fluid coupled system that can provide direct access for microorganisms to enter the ventricles of the brain, and aseptic technique is a very high nursing priority to decrease the risk for infection. Constant monitoring of ICP waveforms is not usually necessary, and removal of CSF for sampling or to maintain normal ICP is done only when specifically ordered

A patient has a nursing diagnosis of risk for ineffective cerebral tissue perfusion related to cerebral edema. An appropriate nursing intervention for the patient is a. avoiding positioning the patient with neck and hip flexion b. maintaining hyperventilation to a PaCO2 of 15 to 20 mm Hg c. clustering nursing activities to provide periods of uninterrupted rest d. routine suctioning to prevent accumulation of respiratory secretions

A. Avoiding positioning the patient with neck and hip flexion- Nursing care activities that increase ICP include hip and neck flexion, suctioning, clustering care activities, and noxious stimuli; they should be avoided or performed as little as possible in the patient with increased ICP. Lowering the PaCO2 below 20 mm Hg can cause ischemia and worsening of ICP; the PaCO2 should be maintained at 30 to 35 mm Hg.

A patient is admitted to the hospital with a left hemiplegia. To determine the size and location and to ascertain whether a stroke is ischemic or hemorrhagic, the nurse anticipates that the health care provider will request a a. CT scan b. lumbar puncture c. cerebral arteriogram d. positron emission tomography (PET)

A: CT scan- A CT scan is the most commonly used diagnostic test to determine the size and location of the lesion and to differentiate a thrombotic stroke from a hemorrhagic stroke. Positron emission tomography (PET) will show the metabolic activity of the brain and provide a depiction of the extent of tissue damage after a stroke. Lumbar punctures are not performed routinely because of the chance of increased intracranial pressure causing herniation. Cerebral arteriograms are invasive and may dislodge an embolism or cause further hemorrhage; they are performed only when no other test can provide the needed information.

During the acute phase of a stroke, the nurse assesses the patient's vital signs and neurologic status every 4 hours. A cardiovascular sign that the nurse would see as the body attempts to increase cerebral blood flow is a. hypertension b. fluid overload c. cardiac dysrhythmias d. S3 and S4 heart sounds

A: Hypertension- The body responds to the vasopasm and a decreased circulation to the brain that occurs with a stroke by increasing the BP, frequently resulting in hypertension. The other options are important cardiovascular factors to assess, but they do not result from impaired cerebral blood flow.

A newly admitted patient who has suffered a right sided brain stroke has a nursing diagnosis of disturbed visual sensory perception related to homonymous hemianopsia. Early in the care of the patient, the nurse should a. place objects on the right side within the patient's field of vision b. approach the patient from the left side to encourage the patient to turn the head c. place objects on the patient's left side to assess the patient's ability to compensate d. patch the affected eye to encourage the patient to turn the head to scan the environment

A: Place objects on the right side within the patient's field of vision- the presence of homonymous hemianopia in a patient with right-hemisphere brain damage causes a loss of vision in the left field. Early in the care of the patient, objects should be placed on the right side of the patient in the field of vision, and the nurse should approach the patient from the right side. Later in treatment, patients should be taught to turn the head and scan the environment and should be approached from the affected side to encourage head turning. Eye patches are used if patients have diplopia (double vision).

Four days following a stroke, a patient is to start oral fluids and feedings. Before feeding the patient, the nurse should first a. check the patient's gag reflex b. order a soft diet for the patient c. raise the head of the bed to sitting position d. evaluate the patient's ability to swallow small sips of ice water

A: check the patient's gag reflex- the first step in providing oral feedings for a patient with a stroke is ensuring that the patient has an intact gag reflex because oral feedings will not be provided if gag reflex is impaired. The nurse should then evaluate the patient's ability to swallow ice chips or ice water after placing the patient in an upright position

Social effects of a chronic neurologic disease include (select all that apply) A. divorce. B. job loss. C. depression. D. role changes. E. loss of self-esteem.

ALL OF THEM Social problems related to chronic neurologic disease may include changes in roles and relationships (such as divorce, job loss, and role changes); other psychologic problems (such depression and loss of self-esteem) may also have social effects. Reference: 1510

1. The health care provider prescribes these actions for a patient who has possible septic shock with a BP of 70/42 mm Hg and oxygen saturation of 90%. In which order will the nurse implement the actions? Put a comma and space between each answer choice (a, b, c, d, etc.) ____________________ a. Obtain blood and urine cultures. b. Give vancomycin (Vancocin) 1 g IV. c. Infuse vasopressin (Pitressin) 0.01 units/min. d. Administer normal saline 1000 mL over 30 minutes. e. Titrate oxygen administration to keep O2 saturation >95%.

ANS: E, D, C, A, B The initial action for this hypotensive and hypoxemic patient should be to improve the oxygen saturation, followed by infusion of IV fluids and vasopressors to improve perfusion. Cultures should be obtained before administration of antibiotics.

While caring for a patient who has been admitted with a pulmonary embolism, the nurse notes a change in the patient's oxygen saturation (SpO2) from 94% to 88%. The nurse will a. increase the oxygen flow rate. b. suction the patient's oropharynx. c. assist the patient to cough and deep breathe. d. help the patient to sit in a more upright position.

ANS: A Increasing oxygen flow rate usually will improve oxygen saturation in patients with ventilation-perfusion mismatch, as occurs with pulmonary embolism. Because the problem is with perfusion, actions that improve ventilation, such as deep-breathing and coughing, sitting upright, and suctioning, are not likely to improve oxygenation. DIF: Cognitive Level: Application REF: 1747-1749 | 1754 TOP: Nursing Process: Implementation MSC: NCLEX: Physiological Integrity

When admitting a patient in possible respiratory failure with a high PaCO2, which assessment information will be of most concern to the nurse? a. The patient is somnolent. b. The patient's SpO2 is 90%. c. The patient complains of weakness. d. The patient's blood pressure is 162/94.

ANS: A Increasing somnolence will decrease the patient's respiratory rate and further increase the PaCO2 and respiratory failure. Rapid action is needed to prevent respiratory arrest. An SpO2 of 90%, weakness, and elevated blood pressure all require ongoing monitoring but are not indicators of possible impending respiratory arrest. DIF: Cognitive Level: Application REF: 1751 TOP: Nursing Process: Assessment MSC: NCLEX: Physiological Integrity

A patient with ARDS who is receiving mechanical ventilation using synchronized intermittent mandatory ventilation (SIMV) has settings of fraction of inspired oxygen (FIO2) 80%, tidal volume 500, rate 18, and positive end-expiratory pressure (PEEP) 5 cm. Which assessment finding is most important for the nurse to report to the health care provider? a. Oxygen saturation 99% b. Patient respiratory rate 22 breaths/min c. Crackles audible at lung bases d. Apical pulse rate 104 beats/min

ANS: A The FIO2 of 80% increases the risk for oxygen toxicity. Since the patient's O2 saturation is 99%, a decrease in FIO2 is indicated to avoid toxicity. The other patient data would be typical for a patient with ARDS and would not need to be urgently reported to the health care provider. DIF: Cognitive Level: Analysis REF: 1760 | 1761-1762 OBJ: Special Questions: Prioritization TOP: Nursing Process: Assessment MSC: NCLEX: Physiological Integrity

21. The patient with neurogenic shock is receiving a phenylephrine (Neo-Synephrine) infusion through a left forearm IV. Which assessment information obtained by the nurse indicates a need for immediate action? a. The patient's IV infusion site is cool and pale. b. The patient has warm, dry skin on the extremities. c. The patient has an apical pulse rate of 58 beats/min. d. The patient's urine output has been 28 mL over the last hour.

ANS: A The coldness and pallor at the infusion site suggest extravasation of the phenylephrine. The nurse should discontinue the IV and, if possible, infuse the medication into a central line. An apical pulse of 58 is typical for neurogenic shock but does not indicate an immediate need for nursing intervention. A 28 mL urinary output over 1 hour would require the nurse to monitor the output over the next hour, but an immediate change in therapy is not indicated. Warm, dry skin is consistent with early neurogenic shock, but it does not indicate a need for a change in therapy or immediate action. DIF: Cognitive Level: Application REF: 1733-1734 OBJ: Special Questions: Prioritization TOP: Nursing Process: Assessment MSC: NCLEX: Physiological Integrity

14. Which information obtained by the nurse when caring for a patient who has cardiogenic shock indicates that the patient may be developing multiple organ dysfunction syndrome (MODS)? a. The patient's serum creatinine level is elevated. b. The patient complains of intermittent chest pressure. c. The patient has crackles throughout both lung fields. d. The patient's extremities are cool and pulses are weak.

ANS: A The elevated serum creatinine level indicates that the patient has renal failure as well as heart failure. The crackles, chest pressure, and cool extremities are all consistent with the patient's diagnosis of cardiogenic shock. DIF: Cognitive Level: Application REF: 1740-1741 TOP: Nursing Process: Assessment MSC: NCLEX: Physiological Integrity

18. A patient is treated in the emergency department (ED) for shock of unknown etiology. The first action by the nurse should be to a. administer oxygen. b. attach a cardiac monitor. c. obtain the blood pressure. d. check the level of consciousness.

ANS: A The initial actions of the nurse are focused on the ABCs—airway, breathing, circulation—and administration of oxygen should be done first. The other actions should be accomplished as rapidly as possible after oxygen administration. DIF: Cognitive Level: Application REF: 1729-1731 | 1732 | 1733 | 1736-1737 OBJ: Special Questions: Prioritization TOP: Nursing Process: Implementation MSC: NCLEX: Physiological Integrity

2. A patient with shock of unknown etiology whose hemodynamic monitoring indicates BP 92/54, pulse 64, and an elevated pulmonary artery wedge pressure has the following collaborative interventions prescribed. Which intervention will the nurse question? a. Infuse normal saline at 250 mL/hr. b. Keep head of bed elevated to 30 degrees. c. Give nitroprusside (Nipride) unless systolic BP <90 mm Hg. d. Administer dobutamine (Dobutrex) to keep systolic BP >90 mm Hg.

ANS: A The patient's elevated pulmonary artery wedge pressure indicates volume excess. A normal saline infusion at 250 mL/hr will exacerbate this. The other actions are appropriate for the patient.

When prone positioning is used in the care of a patient with acute respiratory distress syndrome (ARDS), which information obtained by the nurse indicates that the positioning is effective? a. The patient's PaO2 is 90 mm Hg, and the SaO2 is 92%. b. Endotracheal suctioning results in minimal mucous return. c. Sputum and blood cultures show no growth after 24 hours. d. The skin on the patient's back is intact and without redness.

ANS: A The purpose of prone positioning is to improve the patient's oxygenation as indicated by the PaO2 and SaO2. The other information will be collected but does not indicate whether prone positioning has been effective. DIF: Cognitive Level: Application REF: 1762-1763 TOP: Nursing Process: Evaluation MSC: NCLEX: Physiological Integrity

Which assessment finding by the nurse when caring for a patient with ARDS who is being treated with mechanical ventilation and high levels of positive end-expiratory pressure (PEEP) indicates that the PEEP may need to be decreased? a. The patient has subcutaneous emphysema. b. The patient has a sinus bradycardia with a rate of 52. c. The patient's PaO2 is 50 mm Hg and the SaO2 is 88%. d. The patient has bronchial breath sounds in both the lung fields.

ANS: A The subcutaneous emphysema indicates barotrauma caused by positive pressure ventilation and PEEP. Bradycardia, hypoxemia, and bronchial breath sounds are all concerns and will need to be addressed, but they are not indications that PEEP should be reduced. DIF: Cognitive Level: Application REF: 1760-1761 TOP: Nursing Process: Assessment MSC: NCLEX: Physiological Integrity

23. Which information about a patient who is receiving vasopressin (Pitressin) to treat septic shock is most important for the nurse to communicate to the heath care provider? a. The patient's heart rate is 108 beats/min. b. The patient is complaining of chest pain. c. The patient's peripheral pulses are weak. d. The patient's urine output is 15 mL/hr.

ANS: B Because vasopressin is a potent vasoconstrictor, it may decrease coronary artery perfusion. The other information is consistent with the patient's diagnosis and should be reported to the health care provider but does not indicate a need for a change in therapy. DIF: Cognitive Level: Application REF: 1735-1736 OBJ: Special Questions: Prioritization TOP: Nursing Process: Assessment MSC: NCLEX: Physiological Integrity

22. The following therapies are prescribed by the health care provider for a patient who has respiratory distress and syncope after a bee sting. Which will the nurse administer first? a. normal saline infusion b. epinephrine (Adrenalin) c. dexamethasone (Decadron) d. diphenhydramine (Benadryl)

ANS: B Epinephrine rapidly causes peripheral vasoconstriction, dilates the bronchi, and blocks the effects of histamine and reverses the vasodilation, bronchoconstriction, and histamine release that cause the symptoms of anaphylaxis. The other interventions also are appropriate but would not be the first ones administered. DIF: Cognitive Level: Application REF: 1736-1737 OBJ: Special Questions: Prioritization TOP: Nursing Process: Implementation MSC: NCLEX: Physiological Integrity

A patient has a nursing diagnosis of ineffective airway clearance related to thick, secretions. Which action will be best for the nurse to include in the plan of care? a. Encourage use of the incentive spirometer. b. Offer the patient fluids at frequent intervals. c. Teach the patient the importance of coughing. d. Increase oxygen level to keep O2 saturation >95%.

ANS: B Since the reason for the poor airway clearance is the thick secretions, the best action will be to encourage the patient to improve oral fluid intake. The use of the incentive spirometer should be more frequent in order to facilitate the clearance of the secretions. The other actions also may be helpful in improving the patient's gas exchange, but they do not address the thick secretions that are causing the poor airway clearance. DIF: Cognitive Level: Application REF: 1755 TOP: Nursing Process: Planning MSC: NCLEX: Physiological Integrity

7. A patient with cardiogenic shock has the following vital signs: BP 86/50, pulse 126, respirations 30. The PAWP is increased and cardiac output is low. The nurse will anticipate a. infusion of 5% human albumin. b. administration of furosemide (Lasix) IV. c. titration of an epinephrine (Adrenalin) drip. d. administration of hydrocortisone (SoluCortef).

ANS: B The PAWP indicates that the patient's preload is elevated and furosemide is indicated to reduce the preload and improve cardiac output. Epinephrine would further increase heart rate and myocardial oxygen demand. Normal saline infusion would increase the PAWP further. Hydrocortisone might be used for septic or anaphylactic shock. DIF: Cognitive Level: Application REF: 1735 | 1736 TOP: Nursing Process: Planning MSC: NCLEX: Physiological Integrity

A client with advanced ALS is admitted to the hospital. Because of manifestations that are common in clients with ALS, the nurse should a. attempt to institute bowel-training activities. b. provide the client with small, frequent feedings. c. obtain an order for intermittent catheterization. d. orient the client to his or her surroundings frequently.

ANS: B The course of the disease is relentlessly progressive. Cognition, as well as bowel and bladder sphincters, remains intact. The client may be malnourished because of dysphagia. Encourage small, frequent, high-nutrient feedings. The nurse should assess for aspiration and choking. A feeding tube may be considered during the course of the illness.

The nurse explains that the pathology of Huntington's disease involves a. a decrease in the neurotransmitter norepinephrine. b. an excess of the neurotransmitter dopamine. c. destruction of white matter in the brain. d. formation of neurofibrillary tangles and plaques.

ANS: B The degeneration of the caudate nucleus leads to a reduction in several neurotransmitters, including gamma-aminobutyric acid, acetylcholine, substance P, and metenkephalin, and their synthetic enzymes. This change leaves relatively higher concentrations of the other neurotransmitters, dopamine and norepinephrine.

When the nurse is caring for an obese patient with left lower lobe pneumonia, gas exchange will be best when the patient is positioned a. on the left side. b. on the right side. c. in the tripod position. d. in the high-Fowler's position.

ANS: B The patient should be positioned with the "good" lung in the dependent position to improve the match between ventilation and perfusion. The obese patient's abdomen will limit respiratory excursion when sitting in the high-Fowler's or tripod positions. DIF: Cognitive Level: Application REF: 1754-1755 TOP: Nursing Process: Implementation MSC: NCLEX: Physiological Integrity

. To evaluate the effectiveness of prescribed therapies for a patient with ventilatory failure, which diagnostic test will be most useful to the nurse? a. Chest x-rays b. Pulse oximetry c. Arterial blood gas (ABG) analysis d. Pulmonary artery pressure monitoring

ANS: C ABG analysis is most useful in this setting because ventilatory failure causes problems with CO2 retention, and ABGs provide information about the PaCO2 and pH. The other tests also may be done to help in assessing oxygenation or determining the cause of the patient's ventilatory failure. DIF: Cognitive Level: Application REF: 1752-1754 TOP: Nursing Process: Assessment MSC: NCLEX: Physiological Integrity

To decrease the risk for ventilator-associated pneumonia, which action will the nurse include in the plan of care for a patient who requires intubation and mechanical ventilation? a. Avoid use of positive end-expiratory pressure (PEEP). b. Suction every 2 hours. c. Elevate head of bed to 30 to 45 degrees. d. Give enteral feedings at no more than 10 mL/hr.

ANS: C Elevation of the head decreases the risk for aspiration. PEEP is frequently needed to improve oxygenation in patients receiving mechanical ventilation. Suctioning should be done only when the patient assessment indicates that it is necessary. Enteral feedings should provide adequate calories for the patient's high energy needs. DIF: Cognitive Level: Application REF: 1760-1761 TOP: Nursing Process: Planning MSC: NCLEX: Physiological Integrity

6. To evaluate the effectiveness of the omeprazole (Prilosec) being administered to a patient with systemic inflammatory response syndrome (SIRS), which assessment will the nurse make? a. Auscultate bowel sounds. b. Ask the patient about nausea. c. Monitor stools for occult blood. d. Check for abdominal distention.

ANS: C Proton pump inhibitors are given to decrease the risk for stress ulcers in critically ill patients. The other assessments also will be done, but these will not help in determining the effectiveness of the omeprazole administration. DIF: Cognitive Level: Application REF: 1735-1737 | 1742-1743 TOP: Nursing Process: Evaluation MSC: NCLEX: Physiological Integrity

10. Which intervention will the nurse include in the plan of care for a patient who has cardiogenic shock? a. Avoid elevating head of bed. b. Check temperature every 2 hours. c. Monitor breath sounds frequently. d. Assess skin for flushing and itching.

ANS: C Since pulmonary congestion and dyspnea are characteristics of cardiogenic shock, the nurse should assess the breath sounds frequently. The head of the bed is usually elevated to decrease dyspnea. Elevated temperature and flushing or itching of the skin are not typical of cardiogenic shock. DIF: Cognitive Level: Application REF: 1721 TOP: Nursing Process: Implementation MSC: NCLEX: Physiological Integrity

The nurse is caring for a 22-year-old patient who came to the emergency department with acute respiratory distress. Which information about the patient requires the most rapid action by the nurse? a. Respiratory rate is 32 breaths/min. b. Pattern of breathing is shallow. c. The patient's PaO2 is 45 mm Hg. d. The patient's PaCO2 is 34 mm Hg.

ANS: C The PaO2 indicates severe hypoxemia and respiratory failure. Rapid action is needed to prevent further deterioration of the patient. Although the shallow breathing, rapid respiratory rate, and low PaCO2 also need to be addressed, the most urgent problem is the patient's poor oxygenation. DIF: Cognitive Level: Application REF: 1746-1747 OBJ: Special Questions: Prioritization TOP: Nursing Process: Assessment

5. After receiving 1000 mL of normal saline, the central venous pressure for a patient who has septic shock is 10 mm Hg, but the blood pressure is still 82/40 mm Hg. The nurse will anticipate the administration of a. nitroglycerine (Tridil). b. drotrecogin alpha (Xigris). c. norepinephrine (Levophed). d. sodium nitroprusside (Nipride).

ANS: C When fluid resuscitation is unsuccessful, vasopressor drugs are administered to increase the systemic vascular resistance (SVR) and improve tissue perfusion. Nitroglycerin would decrease the preload and further drop cardiac output and BP. Drotrecogin alpha may decrease inappropriate inflammation and help prevent systemic inflammatory response syndrome, but it will not directly improve blood pressure. Nitroprusside is an arterial vasodilator and would further decrease SVR. DIF: Cognitive Level: Application REF: 1731 | 1733-1735 TOP: Nursing Process: Planning MSC: NCLEX: Physiological Integrity

Which statement by the nurse when explaining the purpose of positive end-expiratory pressure (PEEP) to the family members of a patient with ARDS is correct? a. "PEEP will prevent fibrosis of the lung from occurring." b. "PEEP will push more air into the lungs during inhalation." c. "PEEP allows the ventilator to deliver 100% oxygen to the lungs." d. "PEEP prevents the lung air sacs from collapsing during exhalation."

ANS: D By preventing alveolar collapse during expiration, PEEP improves gas exchange and oxygenation. PEEP will not prevent the fibrotic changes that occur with ARDS, push more air into the lungs, or change the fraction of inspired oxygen (FIO2) delivered to the patient. DIF: Cognitive Level: Comprehension REF: 1761-1762 TOP: Nursing Process: Implementation MSC: NCLEX: Physiological Integrity

4. A patient with cardiogenic shock is cool and clammy and hemodynamic monitoring indicates a high systemic vascular resistance (SVR). Which action will the nurse anticipate taking? a. Increase the rate for the prescribed dopamine (Intropin) infusion. b. Decrease the rate for the prescribed nitroglycerin (Tridil) infusion. c. Decrease the rate for the prescribed 5% dextrose in water (D5W) infusion. d. Increase the rate for the prescribed sodium nitroprusside (Nipride) infusion.

ANS: D Nitroprusside is an arterial vasodilator and will decrease the SVR and afterload, which will improve cardiac output. Changes in the D5W and nitroglycerin infusions will not directly increase SVR. Increasing the dopamine will tend to increase SVR. DIF: Cognitive Level: Application REF: 1733-1734 TOP: Nursing Process: Planning MSC: NCLEX: Physiological Integrity

When assessing a patient with chronic lung disease, the nurse finds a sudden onset of agitation and confusion. Which action should the nurse take first? a. Check pupil reaction to light. b. Notify the health care provider. c. Attempt to calm and reassure the patient. d. Assess oxygenation using pulse oximetry.

ANS: D Since agitation and confusion are frequently the initial indicators of hypoxemia, the nurse's initial action should be to assess oxygen saturation. The other actions also are appropriate, but assessment of oxygenation takes priority over other assessments and notification of the health care provider. DIF: Cognitive Level: Application REF: 1750-1751 OBJ: Special Questions: Prioritization TOP: Nursing Process: Implementation MSC: NCLEX: Physiological Integrity

19. During change-of-shift report, the nurse learns that a patient has been admitted with dehydration and hypotension after having vomiting and diarrhea for 3 days. Which finding is most important for the nurse to report to the health care provider? a. Decreased bowel sounds b. Apical pulse 110 beats/min c. Pale, cool, and dry extremities d. New onset of confusion and agitation

ANS: D The changes in mental status are indicative that the patient is in the progressive stage of shock and that rapid intervention is needed to prevent further deterioration. The other information is consistent with compensatory shock. DIF: Cognitive Level: Application REF: 1728-1729 OBJ: Special Questions: Prioritization TOP: Nursing Process: Assessment MSC: NCLEX: Physiological Integrity

20. A patient who has been involved in a motor vehicle crash is admitted to the emergency department (ED) with cool, clammy skin; tachycardia; and hypotension. Which of these prescribed interventions should the nurse implement first? a. Place the patient on continuous cardiac monitor. b. Draw blood to type and crossmatch for transfusions. c. Insert two 14-gauge IV catheters in antecubital space. d. Administer oxygen at 100% per non-rebreather mask

ANS: D The first priority in the initial management of shock is maintenance of the airway and ventilation. Cardiac monitoring, insertion of IV catheters, and obtaining blood for transfusions also should be rapidly accomplished, but only after actions to maximize oxygen delivery have been implemented. DIF: Cognitive Level: Application REF: 1732 OBJ: Special Questions: Prioritization TOP: Nursing Process: Implementation

The nurse is caring for a patient who was hospitalized 2 days earlier with aspiration pneumonia. Which assessment information is most important to communicate to the health care provider? a. Cough that is productive of blood-tinged sputum b. Scattered crackles throughout the posterior lung bases c. Temperature of 101.5° F (38.6° C) after 2 days of IV antibiotic therapy d. Oxygen saturation (SpO2) has dropped to 90% with administration of 100% O2 by non-rebreather mask.

ANS: D The patient's low SpO2 despite receiving a high fraction of inspired oxygen (FIO2) indicates the possibility of acute respiratory distress syndrome (ARDS). The patient's blood-tinged sputum and scattered crackles are not unusual in a patient with pneumonia, although they do require continued monitoring. The continued temperature elevation indicates a possible need to change antibiotics, but this is not as urgent a concern as the progression toward hypoxemia despite an increase in O2 flow rate. DIF: Cognitive Level: Application REF: 1760 OBJ: Special Questions: Prioritization TOP: Nursing Process: Assessment MSC: NCLEX: Physiological Integrity

After receiving change-of-shift report, which patient will the nurse assess first? a. A patient with cystic fibrosis who has thick, green-colored sputum b. A patient with pneumonia who has coarse crackles in both lung bases c. A patient with emphysema who has an oxygen saturation of 91% to 92% d. A patient with septicemia who has intercostal and suprasternal retractions

ANS: D This patient's history of septicemia and labored breathing suggest the onset of ARDS, which will require rapid interventions such as administration of oxygen and use of positive pressure ventilation. The other patients also should be assessed as quickly as possible, but their assessment data are typical of their disease processes and do not suggest deterioration in their status. DIF: Cognitive Level: Analysis REF: 1758-1760 OBJ: Special Questions: Multiple Patients TOP: Nursing Process: Assessment MSC: NCLEX: Safe and Effective Care Environment

You are alerted to a possible acute subdural hematoma in the patient who A. has a linear skull fracture crossing a major artery. B. has focal symptoms of brain damage with no recollection of a head injury. C. develops decreasing LOC and a headache within 48 hours of a head injury. D. has an immediate loss of consciousness with a brief lucid interval followed by decreasing LOC.

An acute subdural hematoma manifests within 24 to 48 hours of the injury. The signs and symptoms are similar to those associated with brain tissue compression by increased intracranial pressure (ICP) and include decreasing LOC and headache.

To assist a patient in reducing risk for stroke, the nurse may advise the patient to (select all that apply) Keep blood pressure below 140/90 Investigate family history of stroke Limit smoking to < 1 pack per week Limit alcohol intake Engage in physical daily activity

Answers: 1.keep blood pressure below 140/90; 4. limit alcohol intake; 5. Engage in daily physical activity. Rationale: 1. Hypertension is a risk for stroke so maintaining blood pressure in a healthy range will limit risk. 2. Investigation of family history will not lessen risk, but may make the client more aware of personal stroke risk.3. Smoking should be eliminated. 4. Alcohol intake should be limited. 5. A sedentary lifestyle is a risk which exercise on a regular basis will help to address.

While family members are visiting, a patient has a cardiac arrest and is being resuscitated. Which action by the nurse is best?

Ask family members if they wish to remain in the room during the resuscitation. rationale: Research indicates that family members want the option of remaining in the room during procedures such as CPR and that this decreases anxiety and facilitates grieving. The other options may be appropriate if the family decides not to remain with the patient.

16.Which nursing intervention is aimed at reducing muscle weakness in the client with myasthenia gravis?

Assisting the client with activities of daily living (ADLs) The hallmark of myasthenia gravis is muscle weakness that increases with fatigue. The nurse provides assistance with ADLs to prevent fatigue. The nurse also collaborates with the physical therapist in teaching the client energy conservation techniques.

A patient with amyotrophic lateral sclerosis (ALS) is hospitalized with pneumonia. Which nursing action will be included in the plan of care?

Assisting the patient with active range of motion (ROM)

The patient has an order for phenytoin (Dilantin) 100 mg q8hr IV. Available is a phenytoin injection containing 50 mg/mL. How many milliliters of solution should you draw up for the dose? A. 0.5 B. 2 C. 5 D. 20

B 100 mg ÷ 50 mg/mL = 2 mL.

The patient has rhinorrhea after a head injury. What action should you take? A. Pack the nares with sterile gauze. B. A loose collection pad may be placed under the nose. C. Suction the drainage with an inline suction catheter. D. Obtain a sample for culture.

B A loose collection pad may be placed under the nose. Do not place a dressing in the nasal cavity, and nothing should be placed inside the nostril. There is no need to culture the drainage. The concern is whether it is spinal fluid, which is determined by a test for glucose or the halo or ring sign.

The patient has West Nile virus encephalitis and seizures for which phenytoin (Dilantin) is prescribed. What is essential for you do regarding administration? A. Dilute the intravenous (IV) drug in 5% dextrose. B. Assist the patient to maintain good oral hygiene. C. Notify the health care provider if the patient becomes drowsy during IV administration. D. Verify that the levels are between 40 and 60 μg/mL before administration.

B A side effect of Dilantin is gingival hyperplasia, and good oral care minimizes this side effect. The IV drug is diluted in normal saline and will precipitate in D5W. Drowsiness is an expected side effect of IV administration of the drug. Therapeutic levels are between 10 and 20 μg/mL.

You are obtaining a history and physical assessment for a patient with a suspected brain tumor. Which option would be a correct interpretation of findings? A. Balance issues indicate a tumor related in the occipital lobe. B. Personality changes can be caused by a tumor in the frontal lobe. C. Impulsivity is related to a tumor in the temporal lobe. D. Vision is affected by a tumor in the parietal lobe.

B A tumor of the frontal lobe can cause behavioral and personality changes. Balance is related to the parietal lobe or cerebellum. Memory loss and impulsivity are related to alterations in the frontal lobe. Vision is affected by the occipital lobe, and parietal lobe function affects movement.

You would expect the health care provider to order which of the following as the initial treatment for acute cluster headache? A. Ice cap for 30 minutes, four times a day B. 100% oxygen delivered at a rate of 6-8 L/min for 10 minutes C. Aspirin D. Indomethacin

B Acute treatment of cluster headache is inhalation of 100% oxygen delivered at a rate of 6-8 L/min for 10 minutes, which may relieve headache by causing vasoconstriction and increasing synthesis of serotonin in the central nervous system. It can be repeated after a 5-minute rest. Reference: 1490

A client is prescribed levetiracetam (Keppra). Which laboratory tests does the nurse monitor for potential adverse effects of this medication? a. Serum electrolyte levels b. Kidney function tests c. Complete blood cell count d. Antinuclear antibodies

B Adverse effects of levetiracetam (Keppra) include coordination problems and renal toxicity. The other laboratory tests are not affected by levetiracetam.

What would you include in your teaching about a dietary trigger of a cluster headache? A. Aged cheese B. Alcohol C. Caffeine D. Grapefruit

B Alcohol is the only dietary trigger of cluster headaches.

Which statements accurately describe MCI (select all that apply)? A. Always progresses to AD B. Caused by variety of factors and may progress to AD C. Should be aggressively treated with acetylcholinesterase drugs D. Caused by vascular infarcts, which if treated, can delay progression to AD E. Patient is usually not aware that there is a problem with his or her memory

B Although some individuals with MCI revert to normal cognitive function or do not go on to develop AD, those with MCI are at high risk for AD. No drugs have been approved for the treatment of MCI. Research is being conducted to determine whether patients with MCI can benefit from the medications used in AD (e.g., acetylcholinesterase inhibitors). A person MCI is often aware of a significant change in memory.

The nurse is assessing a client with Huntington's disease. Which motor changes does the nurse monitor for in this client? a. Shuffling gait b. Jerky hand movements c. Continuous chewing motions d. Tremors of the hands during fine motor tasks

B An imbalance between excitatory and inhibitory neurotransmitters leads to uninhibited motor movements, such as brisk, jerky, purposeless movements of the hands, face, tongue, and legs. Shuffling gait, continuous chewing motions, and tremors are associated with Parkinson's disease.

An elderly patient fell at home. Which information from the patient's history makes this patient at high risk for an intracerebral bleed? A. History of a heart condition B. Taking warfarin (Coumadin) C. Has lost consciousness for 5 seconds D. History of migraine headaches

B Anticoagulant use is associated with increased hemorrhage and more severe head injury. A heart condition may have caused the syncope that caused the fall, but it was not solely responsible for increased bleeding. Concussions are usually minor injuries that resolve, and the typical signs include a brief disruption in level of consciousness (LOC). If the loss of consciousness is less than 5 minutes, patients are usually discharged. Headache by itself does not indicate a risk for intracerebral bleeding.

. The patient with migraines asks you why "everyone always asks if anyone in the family has a history of migraines." Your priority response is A. "It is standard practice to assess a person's history to identify potential health risks" B. "Around 70% of individuals with migraines have a relative who also has migraines." C. "Assessing your history identifies the treatment plan." D. "This assessment provides a baseline for future assessments."

B Approximately 70% of those with migraine have a first-degree relative who also had migraine headaches. Reference: 1487

ne major goal of treatment for the patient with Huntington's disease is A. disease cure. B. symptomatic relief. C. maintaining employment. D. improving muscle strength.

B Because there is no cure for Huntington's disease (HD), collaborative care is palliative and based on symptom relief. The goal of nursing management is to provide the most comfortable environment possible for the patient and the caregiver by maintaining physical safety, treating the physical symptoms, and providing emotional and psychological support. Reference: 1515-1516

You would question the use of sumatriptan (Imitrex) in which patient? A. A patient with a history of gastric ulcer disease B. A patient with a history of heart disease C. A person allergic to yeast D. A person allergic to cheese

B Because these drugs cause vasoconstriction of coronary arteries, they need to be avoided in patients with heart disease. Reference: 1489

The nurse is caring for a client experiencing migraine headaches who is receiving a beta blocker to help manage this disorder. When preparing a teaching plan, which instruction does the nurse plan to provide? a. "Take this drug only when you have prodromal symptoms indicating the onset of a migraine headache." b. "Take this drug as ordered, even when feeling well, to prevent vascular changes associated with migraine headaches." c. "This drug will relieve the pain during the aura phase soon after a headache has started." d. "This medication will have no effect on your heart rate or blood pressure because you are taking it for migraines."

B Beta blockers are prescribed as prophylactic treatment to prevent the vascular changes that initiate migraine headaches. Heart rate and blood pressure will also be affected, and the client should monitor these side effects. The other responses do not discuss an appropriate use of the medication.

You are caring for a patient admitted with a subdural hematoma after a motor vehicle accident. Which change in vital signs would you interpret as a manifestation of increased intracranial pressure? A. Tachypnea B. Bradycardia C. Hypotension D. Narrowing pulse pressure

B Changes in vital signs indicative of increased ICP are known as Cushing's triad, which consists of increasing systolic pressure with a widening pulse pressure, bradycardia with a full and bounding pulse, and irregular respirations.

You would expect to find which clinical manifestation in a patient admitted with a left-brain stroke? A. Impulsivity B. Impaired speech C. Left-sided neglect D. Short attention span

B Clinical manifestations of left-sided brain damage include right hemiplegia, impaired speech and language aphasias, impaired right and left discrimination, and slow and cautious performance. The other options are all manifestations of right-sided brain damage.

During the nursing assessment the patient states that he has been experiencing headaches that last for weeks to months at a time and then go into remission. These signs and symptoms are consistent with which type of headache? A. Sinus B. Cluster C. Tension D. Migraine

B Cluster headaches are a rare form of headache, affecting less than 0.1% of the population. Cluster headaches involve repeated headaches that can occur for weeks to months at a time, followed by periods of remission.

What characteristic differentiates delirium from dementia? A. Answers orientation questions with "I don't know" B. Altered alertness (hypovigilant or hypervigilant) C. Has a consistent course D. Is treated with donepezil (Aricept)

B Delirium is characterized by an acute onset, a fluctuating course, inattention, disorganized thinking, and altered level of consciousness. Dementia is characterized by a normal level or alertness but confusion. Answers to orientation questions such as "I don't know" are elicited from persons with depression. Dementia has a consistent course; delirium fluctuates. Dementia is treated with Aricept. Treatment of delirium focuses on the treatment of precipitating factors.

The patient has been receiving scheduled doses of phenytoin (Dilantin) and begins to experience diplopia. You immediately assess the patient for A. an aura. B. nystagmus or confusion. C. abdominal pain or cramping. D. irregular pulse or palpitations

B Diplopia is a sign of phenytoin toxicity. You should assess for other signs of toxicity, which include neurologic changes such as nystagmus, ataxia, confusion, dizziness, or slurred speech. Reference: 1496

For a patient with a suspected stroke, which important piece of information should you obtain? A. Time of the patient's last meal B. Time at which stroke symptoms first appeared C. Patient's hypertension history and management D. Family history of stroke and other cardiovascular diseases

B During initial evaluation, the single most important point in the patient's history is the time of onset of stroke symptoms. If the stroke is ischemic, recombinant tissue plasminogen activator (tPA) must be administered within 3 to 4.5 hours of the onset of clinical signs; tPA reestablishes blood flow through a blocked artery and prevents brain cell death in patients with an acute onset of symptoms.

The patient with Parkinson's disease is beginning therapy with carbidopa/levodopa (Sinemet).What teaching should you provide to this patient? A. The medication should be taken every other day. B. It may take several weeks for effects to be observed. C. Side effects are limited to rash and photophobia. D. Constipation may occur.

B Effects of carbidopa/levodopa (Sinemet) may be delayed for several weeks to months. Reference: 1508

You are caring for a patient with peripheral neuropathy who is going to have electromyographic (EMG) studies tomorrow morning. What should you do to prepare the patient? A. Ensure the patient has an empty bladder. B. Instruct the patient that there is no risk of electric shock. C. Ensure the patient has no metallic jewelry or metal fragments. D. Instruct the patient that he or she may experience pain during the study.

B Electromyography (EMG) assesses electrical activity associated with nerves and skeletal muscles. Needle electrodes are inserted to detect muscle and peripheral nerve disease. You should inform the patient that pain and discomfort are associated with insertion of needles. There is no risk of electric shock with this procedure.

The patient is newly diagnosed with encephalitis caused by herpes simplex virus (HSV) infection. What is essential for you to do? A. Administer penicillin. B. Administer acyclovir. C. Perform a Glasgow Coma Scale assessment. D. Facilitate a magnetic resonance imaging (MRI) study.

B Encephalitis is usually viral, and it is treated with antiviral acyclovir (Zovirax). For maximum benefit, the drug should be started in a timely manner. Penicillin is used for bacterial meningitis. Because the diagnosis is known, treatment of the cause is more important than additional assessment.

What nursing measure is indicated to reduce the potential for seizures and increased ICP in a patient with bacterial meningitis? A. Administering codeine for relief of head and neck pain B. Controlling fever with prescribed drugs and cooling techniques C. Keeping the room darkened and quiet to minimize environmental stimulation D. Maintaining the patient on strict bed rest with the head of the bed slightly elevated

B Fever must be vigorously managed because it increases cerebral edema and the frequency of seizures. Neurologic damage may result from an extremely high temperature over a prolonged period. Acetaminophen or aspirin may be used to reduce fever; other measures, such as a cooling blanket or sponge baths with tepid water, may be effective in lowering the temperature.

What is the difference between viral and bacterial meningitis? A. Viral meningitis is treated with sulfa drugs, and bacterial meningitis is treated with penicillin. B. Full recovery is expected with the viral form, but more serious residual effects are seen with the bacterial form. C. Viral meningitis is considered more ominous because serious brain involvement is experienced. D. Typical signs of viral meningitis are diplopia and fasciculations, whereas signs of bacterial meningitis are fever and nuchal rigidity.

B Full recovery is expected from viral meningitis, and any residual effects are rare and minor compared with those of bacterial meningitis. The bacterial form can cause dementia, seizures, deafness, hemiplegia, and hydrocephalus. Viral meningitis is managed symptomatically because the disease is self-limiting, a full recovery is expected, and there usually is no brain involvement. Antibiotics are not used for viruses. Both forms manifest with the symptoms of headache, fever, photophobia, and stiff neck.

You are counseling the family of a patient with Huntington's disease (HD) about the genetics involved in the disease. You would be correct in informing the family that the genetic risk for manifestation of the disease in genetic transmission is A. 1 in every 4 pregnancies. B. 1 in every 2 pregnancies. C. only evident in male children. D. impossible to predict.

B HD is a genetically transmitted, autosomal dominant disorder that affects both men and women of all races. The offspring of a person with this disease have a 50% risk of inheriting it. Reference: 1515

The patient had an ischemic stroke and is undergoing rehabilitation. He is diagnosed with homonymous hemianopsia. What should you do? A. Better arrange the environment to suit the patient's needs. B. Teach the patient to turn his head to scan the environment. C. Obtain prescriptive glasses for the patient. D. Have the patient wear an eye patch.

B Homonymous hemianopsia (blindness in the same half of each visual field) is a common problem. Persistent disregard of objects in part of the visual field should alert you to this possibility. In rehabilitation, the patient should learn to compensate by consciously attending to or by scanning the neglected side. Early immediate intervention involves arranging the environment within the patient's perceptual field. Glasses do not help the problem. Diplopia (double vision) can be a problem and is treated with an eye patch.

During the postictal period of a seizure, you would expect the patient to A. demonstrate minor jerking and eye fluttering. B. sleep for several hours. C. be incontinent of urine and feces. D. require ventilator assistance.

B In the postictal phase the patient usually has muscle soreness, is very tired, and may sleep for several hours. Some patients may not feel normal for several hours or days after a seizure. The patient has no memory of the seizure. Reference: 1493

A client is actively experiencing status epilepticus. Which prescribed medication does the nurse prepare to administer? a. Atropine b. Lorazepam (Ativan) c. Phenytoin (Dilantin) d. Morphine sulfate

B Initially, intravenous lorazepam is administered to stop motor movements. This is followed by the administration of phenytoin. Atropine and morphine are not administered for seizure activity.

Along with dementia, what additional symptom is characteristic of Lewy body dementia? A. Myasthenia gravis B. Extrapyramidal signs C. Ataxia D. Multiple sclerosis

B Lewy body dementia has Lewy bodies in the brainstem and cortex. In addition to dementia, it is characterized by at least two of the following: extrapyramidal signs, fluctuating cognitive ability, and hallucinations.

Which statement is true regarding the prevalence of restless legs syndrome? A. Rates are great than 50%. B. Rates are lowest in the Asian population. C. Rates are higher in males than females. D. Rates are highest in the age group of 20 to 30 year olds.

B Restless legs syndrome (RLS) is a relatively common condition characterized by unpleasant sensory (paresthesias) and motor abnormalities of one or both legs. Prevalence rates vary from 5% to 15%. However, the prevalence may be higher because the condition is underdiagnosed. The prevalence of RLS is lower in Asian populations, and RLS is more common in older adults. It is more common in women than men, and women may have an earlier age of onset. Reference: 1514

The caregiver of a client with advanced Alzheimer's disease states, "She is always wandering off. What can I do to manage this restless behavior?" How does the nurse respond? a. "Allow for a 45-minute daytime nap." b. "Take the client for frequent walks throughout the day." c. "Using a Geri-chair may decrease agitation." d. "Give a mild sedative during periods of restlessness."

B Several strategies may be used to cope with restlessness and wandering. Taking the client for frequent walks may decrease restless behavior. Another strategy is to engage the client in structured activities. The other options would not be as helpful.

How should you most accurately assess the position sense of a patient with a recent traumatic brain injury? A. Ask the patient to close his or her eyes and slowly bring the tips of the index fingers together. B. Ask the patient to maintain balance while standing with his or her feet together and eyes closed. C. Ask the patient to close his or her eyes and identify the presence of a common object on the forearm. D. Place the two points of a calibrated compass on the tips of the fingers and toes, and ask the patient to discriminate the points.

B The Romberg test is an assessment of position sense in which the patient stands with the feet together and then closes his or her eyes while attempting to maintain balance. The other tests of neurologic function do not directly assess position sense.

The nurse assesses for which clinical manifestation in a client with multiple sclerosis (MS) of the relapsing type? a. Absence of periods of remission b. Attacks becoming increasingly frequent c. Absence of active disease manifestations d. Gradual neurologic symptoms without remission

B The classic picture of relapsing-remitting MS is characterized by increasingly frequent attacks. The other manifestations do not correlate with a relapsing type of MS.

The nurse is preparing a client who has multiple sclerosis (MS) for discharge home from a rehabilitation center. The client has been prescribed cyclophosphamide (Cytoxan) and methylprednisolone (Medrol). Which instruction does the nurse include in the teaching plan for the client? a. "Take warm baths to promote muscle relaxation." b. "Avoid crowds and people with colds." c. "Use physical aids such as walkers as little as possible." d. "Stop using these medications when your symptoms improve."

B The client should be taught to avoid people with any type of upper respiratory illness because these medications are immunosuppressive. Warm baths will exacerbate the MS symptoms, assistive devices may be required for safe ambulation, and medication should not be stopped.

When working with the family of a patient with amyotrophic lateral sclerosis (ALS) in the later stages of the disease, what teaching should you reinforce? A. Perform frequent passive range of motion to maintain joint function. B. Communicate with patient normally because cognition remains intact. C. Provide foods high in fiber to prevent constipation. D. Speak in a loud clear voice to facilitate the patient's hearing.

B The illness trajectory for ALS is devastating because the patient remains cognitively intact while wasting away. Active range of motion is needed to maintain muscle function. Foods high in fiber may be difficult to chew. Patients with ALS usually do not have a hearing impairment. Reference: 1514

What is important when obtaining a history of a patient with a neurologic problem? A. Have patient agree or disagree with suggested symptoms to obtain a thorough history. B. Mode of onset and course of illness are essential aspects. C. Check out neurologic problems caused by nutrition by asking about sodium. D. Assess for dementia using the Confusion Assessment Method (CAM).

B The mode of onset and the course of the illness are especially important aspects of the history. The nature of a neurologic disease process often can be described by these facts alone. Avoid suggesting certain symptoms or using leading questions. Nutritional deficits of B vitamins are most likely to cause neurologic problems. CAM is used to assess for delirium.

When presenting information to a nursing student regarding migraine headaches, you would identify which age group as most affected by migraine headaches? A. Adolescents B. 20 to 30 year olds C. 40 to 50 year olds D. Older adults aged 60 and greater

B The most common age for onset of migraine is between the ages of 20 and 30 years. Migraine affects as many as 17% of females and 6% of males in the United States. Reference: 1487

. Which characteristic of a patient's recent seizure indicates a partial seizure? A. The patient lost consciousness during the seizure. B. The seizure involved lipsmacking and repetitive movements. C. The patient fell to the ground and became stiff for 20 seconds. D. The etiology of the seizure involved both sides of the patient's brain.

B The most common complex partial seizure involves lip smacking and automatisms (repetitive movements that may not be appropriate). Loss of consciousness, bilateral brain involvement, and a tonic phase are associated with generalized seizure activity. Reference: 1494

Preventing which problem is a priority nursing goal for a patient who had cranial surgery today? A. Pain B. Increased ICP C. Infection D. Malnutrition

B The primary goal of care after cranial surgery is prevention of increased ICP. Other priorities are monitoring neurologic function, fluid and electrolyte levels, and serum osmolality. The brain does not have pain receptors, although the patient can have a headache. However, increased ICP remains a priority. Infection is not a priority the day of surgery, and nutrition is important, but increased ICP is the priority.

How do you assess the accessory nerve? A. Assess the gag reflex by stroking the posterior pharynx. B. Ask the patient to shrug the shoulders against resistance. C. Ask the patient to push the tongue to either side against resistance. D. Have the patient say "ah" while visualizing elevation of the soft palate

B The spinal accessory nerve is tested by asking the patient to shrug the shoulders against resistance and to turn the head to either side against resistance. The other options are used to test the glossopharyngeal and vagus nerves.

What are the two most common causes of dementia? A. Diabetes mellitus and hypercholesterolemia B. Neurodegenerative conditions and vascular disorders C. Effects of smoking and coronary artery disease D. Metabolic syndrome and systemic diseases

B The two most common causes of dementia are neurodegenerative conditions (e.g., Alzheimer's disease) and vascular disorders. Risk factors include advanced age, family history, history of smoking, cardiac dysrhythmias, hypertension, hypercholesterolemia, diabetes mellitus, and coronary artery disease, and metabolic syndrome.

Which measure is most effective in preventing the spread of viral meningitis? A. Have close personnel wear surgical masks. B. Avoid touching respiratory secretions. C. Obtain yearly vaccinations. D. Gargle daily with salt wate

B Viral meningitis is usually acquired through direct contact with respiratory secretions. Bacterial meningitis has respiratory precautions until the cultures are negative, and precautions require a particulate respirator mask rather than surgical mask. There is no yearly vaccination for viral meningitis. The use of Haemophilus influenzae vaccine has decreased the incidence of bacterial meningitis caused by this organism, and the vaccine against Neisseria meningitides provides protection against that organism that causes bacterial meningitis. Gargling with salt water is a helpful home measure, but not touching infectious secretions is more effective.

You plan care for the patient with increased ICP with the knowledge that the best way to position the patient is to A. keep the head of the bed flat. B. elevate the head of the bed to 30 degrees. C. maintain patient on the left side with the head supported on a pillow. D. use a continuous-rotation bed to continuously change patient position.

B You should maintain the patient with increased ICP in the head-up position. Elevation of the head of the bed to 30 degrees enhances respiratory exchange and aids in decreasing cerebral edema. You should position the patient to prevent extreme neck flexion, which can cause venous obstruction and contribute to elevated ICP. Elevation of the head of the bed reduces sagittal sinus pressure, promotes drainage from the head through the valveless venous system in the jugular veins, and decreases the vascular congestion that can produce cerebral edema. However, raising the head of the bed above 30 degrees may decrease the cerebral perfusion pressure (CPP) by lowering systemic blood pressure. Careful evaluation of the effects of elevation of the head of the bed on the ICP and the CPP is required.

The nurse is teaching a client with a spinal cord tumor about the treatment plan. Which statements indicate that the client correctly understands the teaching? (Select all that apply.) a. "Because my symptoms occurred so quickly, I am likely to be cured quickly by surgery." b. "Radiation therapy can shrink the tumor but radiation can cause more problems, too." c. "I am glad you are here to turn me. Lying in one position for a long time makes my pain worse, even if turning is uncomfortable." d. "I have put my affairs in order and purchased a burial plot because this type of cancer is almost always fatal." e. "My family is making some changes at home for me, including moving my bedroom downstairs."

B, C, E

The nurse is teaching a male client with a spinal cord injury at T4 (thoracic) about the sexual effects of this injury. Which statement by the client indicates correct understanding of the teaching? (Select all that apply.) a. "I will not be able to have an erection because of my injury." b. "Ejaculation may not be as predictable as before." c. "I will explore other ways besides intercourse to please my partner." d. "I may urinate with ejaculation but this will not cause an infection." e. "I should be able to have an erection with stimulation."

B, D, E

When providing community health care teaching regarding the early warning signs of AD, which signs would you advise family members to report (select all that apply)? A. Misplacing car keys B. Loses the sense of time C. Difficulty performing familiar tasks D. Problems with performing basic calculations E. Becoming lost in a usually familiar environment

B,C,D,E Difficulty performing familiar tasks, problems with performing basic calculations, and becoming lost in a usually familiar environment are early warning signs of AD. Misplacing car keys is a normal frustrating event for many people.

A patient with an intracranial problem does not open his eyes to any stimulus, has no verbal response except moaning and muttering when stimulated, and flexes his arm in response to painful stimuli. The nurse records the patients GCS score as a. 6 b. 7 c. 9 d. 11

B. 7- no opening of eyes = 1; incomprehensible words= 2, flexion withdrawal = 4 Total = 7

The nurse recognizes the presence of Cushing's triad in the patient with a. Increased pulse, irregular respiration, increased BP b. decreased pulse, irregular respiration, increased pulse pressure c. increased pulse, decreased respiration, increased pulse pressure d. decreased pulse, increased respiration, decreased systolic BP

B. Cushing's triad consists of three vital sign measures that reflect ICP and its effect on the medulla, the hypothalamus, the pons, and the thalamus. Because these structures are very deep, Cushing's triad is usually a late sign of ICP. The signs include an increasing systolic BP with a widening pulse pressure, a bradycardia with a full and bounding pulse, and irregular respirations.

A patient with a head injury has bloody drainage from the ear. To determine whether CSF is present in the drainage, the nurse a. examines the tympanic membrane for a tear b. tests the fluid for a halo sign on a white dressing c. tests the fluid with a glucose identifying strip or stick d. collects 5 mL of fluid in a test tube and sends it to the laboratory for analysis

B. Tests the fluid for a halo sing on a white dressing- Testing clear drainage for CSF in nasal or ear drainage may be done with a Dextrostik or Tes-Tape strip, but if blood is present, the glucose in the blood will produce and unreliable result. To test bloody drainage, the nurse should test the fluid for a halo or ring that occurs when a yellowish ring encircles blood dripped onto a white pad or towel

The nurse plans care for a patient with increased ICP with the knowledge that the best way to position the patient is to a. keep the head of the bed flat b. elevate the head of the bed to 30 degrees c. maintain patient on the left side with the head supported on a pillow d. use a continuous rotation bed to continuously change patient position

B. elevate the head of the bed to 30 degrees

A diagnosis of a ruptured cerebral aneurysm has been made in a patient with manifestations of a stroke. The nurse anticipates that treatment options that would be evaluated for the patient include a. hyperventilation therapy b. surgical clipping of the aneurysm c. administration of hyperosmotic agents d. administration of thrombolytic therapy

B: Surgical clipping of they aneurysm- Surgical management with clipping of an aneurysm to decrease re bleeding and vasospasm is an option for a stroke cause by rupture of a cerebral aneurysm. Placement of coils into the lumens of the aneurysm by intercentional radiologists is increasing in popularity. Hyperventilation therapy would increase vasodilation and the potential for hemorrhage. Thrombolytic therapy would be absolutely contraindicated, and if a vessel is patent, osmotic diuretics may leak into tissue, pulling fluid out of the vessel and increasing edema.

To promote communication during rehabilitation of the patient with aphasia, an appropriate nursing intervention is to a. use gestures, pictures, and music to stimulate patient responses b. talk about activities of daily living (ADLs) that are familiar to the patient c. structure statements so that patient does not have to respond verbally d. use flashcards with simple words and pictures to promote language recall

B: Talk about ADLs that are familiar to the patient- during rehabilitation, the patient with aphasia needs frequent, meaningful verbal stimulation that has relevance for him. Conversation by the nurse and family should address ADLs that are familiar to the patient. Gestures, pictures, and simple statements are more appropriate in the acute phase, when patients may be overwhelmed with verbal stimuli. Flashcards are often perceived by the patient as childish and meaningless.

Damage to which part of the brain is responsible for expressive aphasia? a) Vernicke's b) Broca's c) Parietal lobe d) None of the above

Broca's

Which nursing diagnosis is more appropriate for a patient with advanced Parkinson's disease? A. Urinary urge incontinence related to effects of drug therapy B. Ineffective breathing pattern related to diaphragm fatigue C. Risk for aspiration related to impaired swallowing D. Risk for injury related to limited vision

C

Which neurologic test or procedure requires the nurse to determine whether an informed consent has been obtained from the client before the test or procedure? a. Measurement of sensation using the pinprick method b. Computed tomography of the cranial vault c. Lumbar puncture for cerebrospinal fluid (CSF) sampling d. Venipuncture for autoantibody analysis

C A lumbar puncture is an invasive procedure with many potentially serious complications. The other assessments or tests are considered noninvasive.

The nurse is discussing advanced directives with a client who has amyotrophic lateral sclerosis (ALS). The client states, "I do not want to be placed on a mechanical ventilator." How does the nurse respond? a. "You will need to discuss that with your family and health care provider." b. "Why are you afraid of being placed on a breathing machine?" c. "What would you like to be done if you begin to have difficulty breathing?" d. "You will be on the ventilator only until your muscles get stronger."

C ALS is an adult-onset upper and lower motor neuron disease, characterized by progressive weakness, muscle wasting, and spasticity, eventually leading to paralysis. Once muscles of breathing are involved, the client must include in the advance directives what is to be done when breathing is no longer possible without intervention. The other statements do not address the client's needs.

The patient had an ischemic stroke 5 hours earlier. What treatment do you anticipate? A. Administer nicardipine (Cardene) for the patient's blood pressure of 200/100 mm Hg. B. Administer systemic thrombolytic tissue plasminogen activator (tPA). C. Make patient NPO. D. Administer acetaminophen (Tylenol) prophylactically.

C About 25% of patients worsen in the first 24 to 48 hours after a stroke. Patients should have nothing by mouth (NPO) until the stroke has stabilized to ensure there is no progression to loss of gag reflex and aspiration. Elevated blood pressure is common immediately after a stroke and may be a protective response to maintain cerebral perfusion. A drug is not used to lower blood pressure unless the systolic pressure is more than 220 mm Hg. Systemic tPA must be administered within 3 to 4.5 hours of stroke onset. The patient's temperature is treated, but not prophylactically.

Which statement is true regarding the prevalence of amyotrophic lateral sclerosis (ALS)? A. Death occurs in less than one year B. Survival rate is less than two years C. Rates are higher in males than females D. Rates are highest in the age group of 20 to 30 year olds

C Amyotrophic lateral sclerosis (ALS) is a rare progressive neurologic disorder characterized by loss of motor neurons. ALS usually leads to death within 2 to 6 years after diagnosis, but many patients may survive for more than 10 years. The onset is usually between 40 and 70 years of age. ALS is more common in men than women by a ratio of 2:1. Reference: 1514-1515

Which option is most indicative of a skull fracture after blunt head trauma? A. Facial edema B. Epitasis C. Otorrhea positive for glucose D. Laceration oozing blood

C An indication of a basal fracture is cerebrospinal fluid (CSF) leakage from the ear, which confirms that the fracture has traversed the dura. Periorbital ecchymosis can indicate a skull fracture, but generalized facial edema does not. The head is vascular, and it is not unusual to have a nosebleed; a positive ring sign (halo sign) indicates a skull fracture. A superficial laceration does not indicate a skull fracture.

You explain to the patient with a stroke who is scheduled for angiography that the test is used to determine the A. presence of increased intracranial pressure (ICP). B. site and size of the infarction. C. patency of the cerebral blood vessels. D. presence of blood in the cerebrospinal fluid.

C Angiography provides visualization of cerebral blood vessels, can provide an estimate of perfusion, and can detect filling defects in the cerebral arteries.

A client with Alzheimer's disease is admitted to the hospital. Which psychosocial assessment is most important for the nurse to complete? a. Ability to recall past events b. Ability to perform self-care c. Reaction to a change of environment d. Relationship with close family members

C As the disease progresses, the client experiences changes in emotional and behavioral affect. The nurse should be alert to the client's reaction to a change in environment, such as being hospitalized, because the client may exhibit an exaggerated response, such as aggression, to the event. The other assessments should be completed but are not as important for a client with Alzheimer's disease.

The nurse is caring for a client who has chronic migraine headaches. Which complementary health therapy does the nurse suggest? a. "Place a hot compress on your forehead at the onset of the headache." b. "Wear dark sunglasses when you are in brightly lit spaces." c. "Lie down in a darkened room when you experience a headache." d. "Do not sleep longer than 6 hours at one time."

C At the onset of a migraine attack, the client may be able to alleviate pain by lying down and darkening the room. He or she may want both eyes covered and a cool cloth on the forehead. If the client falls asleep, he or she should remain undisturbed until awakening. The other options are not recognized therapies for migraines.

You would identify which tool as the most useful for diagnosing a tension headache? A. CT scan B. Electromyography (EMG) C. Careful history taking D. Assessment of deep tendon reflexes

C Careful history taking is probably the most important tool for diagnosing tension-type headache. Electromyography (EMG) may be performed. This test may reveal sustained contraction of the neck, scalp, or facial muscles. However, many patients may not show increased muscle tension with this test, even when the test is done during the actual headache. Reference: 1486

What does the pathophysiology of Alzheimer's disease (AD) most commonly involve? A. Presence of presenilin-1 and presenilin-2 genes B. Dissolving of plaques in brain tissue C. Changes in brain structure and function D. Residual inflammation from arboviruses

C Characteristic findings of AD are related to changes in the brain's structure and functions: (1) amyloid plaques (more in certain parts of the brain); (2) neurofibrillary tangles (more plentiful than normally seen); and (3) loss of connections between cells and cell death (and atrophy). Three genes are important in the cause of early-onset AD, but only a small percentage of people younger than 60 years develop AD. All individuals develop plaques in their brain tissue as part of aging, but in AD, there are more plaques in certain parts of the brain. Encephalitis is a treatable cause of dementia.

Why does the health care provider prescribe a cholinesterase inhibitor (e.g., donepezil [Aricept]) for a patient with AD? A. Cure the disease B. Stop any further decline C. Slow the rate of decline D. Treat co-morbidity of depression

C Cholinesterase inhibitors are used to slow the rate of decline of the disease and the worsening of symptoms. There is no cure for AD, which is a chronic, progressive disease. Treating depression that is often associated with AD may improve cognitive ability, and antidepressants are prescribed.

A client who has Alzheimer's disease is being discharged home. What safety instructions does the nurse include in the teaching plan for the client's caregiver? a. "Keep exercise to a minimum." b. "Place a padded throw rug at the bedside." c. "Install deadbolt locks on all outside doors." d. "Keep the lights off in the bedroom at night."

C Clients with Alzheimer's disease have a tendency to wander, especially at night. If possible, alarms should be installed on all outside doors to alert family members if the client leaves. At a minimum, all outside doors should have deadbolt locks installed to prevent the client from going outdoors unsupervised. The client should be allowed to exercise within his or her limits. Throw rugs are a slip and fall hazard and should be removed. The client may need or want lights on in the bedroom at night.

The nurse is assessing a client with an early onset of multiple sclerosis (MS). Which clinical manifestation does the nurse expect to see? a. Hyperresponsive reflexes b. Excessive somnolence c. Nystagmus d. Heat intolerance

C Early signs and symptoms of MS include changes in motor skills, vision, and sensation. The other manifestations are later signs of MS.

The nurse is assessing a client with a history of migraines. Which clinical manifestation is an early sign of a migraine with aura? a. Vertigo b. Lethargy c. Visual disturbances d. Numbness of the tongue

C Early warning of impending migraine with aura usually consists of visual changes, flashing lights, or diplopia. The other manifestations are not associated with an impending migraine with aura.

What is the best preventive measure for West Nile virus encephalitis? A. Yearly vaccination B. Contact isolation C. Avoidance of standing water D. Prophylactic acyclovir (Zovirax)

C Encephalitis is usually caused by a virus that is transmitted by ticks and mosquitoes. To prevent encephalitis, avoid standing water where mosquitoes can breed. There is no vaccination. Contact isolation does not prevent contracting the disease. Acyclovir is used to treat the disease.

A client with multiple sclerosis is being treated with fingolimod (Gilenya). Which clinical manifestation alerts the nurse to an adverse effect of this medication? a. Periorbital edema b. Black tarry stools c. Bradycardia d. Vomiting after meals

C Fingolimod (Gilenya) is an antineoplastic agent that can cause bradycardia, especially within the first 6 hours after administration. The other manifestations are not adverse effects of fingolimod.

The nurse is caring for a client who has low back pain (LBP) from a work-related injury. Which measures does the nurse incorporate into the client's plan of care? a. Apply moist heat continuously to the affected area. b. Use ice packs or ice massage for 1 to 2 hours over the affected area. c. Apply heat packs for 20 to 30 minutes at least four times daily. d. Advise the client to avoid hot baths or showers.

C Heat increases blood flow to the affected area and promotes healing of injured nerves. However, continuous application of moist heat can promote skin breakdown.

During neurologic testing, the patient is able to perceive pain elicited by a pinprick. Based on this finding, which assessment may be omitted? A. Position sense B. Patellar reflexes C. Temperature perception D. Heel-to-shin movements

C If pain sensation is intact, assessment of temperature sensation may be omitted because both sensations are carried by the same ascending pathways.

What sign would make you suspect the cause of increased ICP involves the hypothalamus? A. Contralateral hemiparesis B. Ipsilateral pupil dilation C. Rise in temperature D. Decreased urine output

C If the ICP affects the hypothalamus, there can be a change in the body temperature. Increasing ICP can cause changes in motor ability, with contralateral hemiparesis. Compression of the cranial nerve III causes dilation of the pupil on the side of the mass (ipsilateral). Decreased urine output is not specific for hypothalamic function.

A patient with right-sided hemiplegia and aphasia resulting from a stroke most likely has involvement of the A. brainstem. B. vertebral artery. C. left middle cerebral artery. D. right middle cerebral artery.

C If the middle cerebral artery is involved in a stroke, the expected clinical manifestations include aphasia, motor and sensory deficit, and hemianopsia on the dominant side and include neglect, motor and sensory deficit, and hemianopsia on the nondominant side.

A patient experiencing TIAs is scheduled for a carotid endarterectomy. You explain that this procedure is done to A. decrease cerebral edema. B. reduce the brain damage that occurs during a stroke in evolution. C. prevent a stroke by removing atherosclerotic plaques blocking cerebral blood flow. D. provide a circulatory bypass around thrombotic plaques obstructing cranial circulation.

C In carotid endarterectomy, the atheromatous lesions are removed from the carotid artery to improve blood flow.

Bladder training for a male patient who has urinary incontinence after a stroke includes A. limiting fluid intake. B. keeping a urinal in place at all times. C. assisting the patient to stand to void. D. catheterizing the patient every 4 hours.

C In the acute stage of stroke, the primary urinary problem is poor bladder control and incontinence. Nurses should promote normal bladder function and avoid the use of indwelling catheters. A bladder retraining program consists of (1) adequate fluid intake, with most given between 8:00 AM and 7:00 PM; (2) scheduled toileting every 2 hours using a bedpan, commode, or bathroom; and (3) observing signs of restlessness, which may indicate the need for urination. Intermittent catheterization may be used for urinary retention (not urinary incontinence). During the rehabilitation phase after a stroke, nursing interventions focused on urinary continence include (1) assessment for bladder distention by palpation; (2) offering the bedpan, urinal, commode, or toilet every 2 hours during waking hours and every 3 to 4 hours at night; (3) focusing the patient on the need to urinate with a direct command; (4) assistance with clothing and mobility; (5) scheduling most fluid intake between 7:00 AM and 7:00 PM; and (6) encouraging the usual position for urinating (standing for men and sitting for women).

A patient with increased ICP has mannitol (Osmitrol) prescribed. Which option is the best indication that the drug is achieving the desired therapeutic effects? A. Urine output increases from 30 mL to 50 mL/hour. B. Blood pressure remains less than 150/90 mm Hg. C. The LOC improves. D. No crackles are auscultated in the lung fields.

C LOC is the most sensitive indicator of ICP. Mannitol is an osmotic diuretic that works to decrease the ICP by plasma expansion and an osmotic effect. Although the other options may indicate a therapeutic effect of a diuretic, they are not the main reason this drug is given.

You are answering questions at a class on brain tumors for nursing students. Which information related to brain tumors should you include in this class? A. Brain tumors are not removed unless than they cause headaches or seizures. B. Seizures are an uncommon symptom unless there is metastasis. C. The most common type of brain tumor is from metastasis of cancer outside of the brain. D. Brain tumors commonly metastasize to the lungs because of high vascularity.

C Metastatic brain tumors from a malignant neoplasm elsewhere in the body are the most common type of brain tumors. All brain tumors eventually cause death from increasing tumor volume that leads to increased ICP, and all tumors should be removed. Seizures are common in patients with gliomas and brain metastases. Brain tumors rarely metastasize outside the CNS because they are contained by structural (meninges) and physiologic (blood-brain) barriers.

A male patient with a diagnosis of Parkinson's disease (PD) has been recently admitted to a long-term care facility. Which action should the health care team take in order to promote adequate nutrition for this patient? A. Provide multivitamins with each meal. B. Provide a diet that is low in complex carbohydrates and high in protein. C. Provide small, frequent meals throughout the day that are easy to chew and swallow. D. Provide the patient with a minced or pureed diet that is high in potassium and low in sodium.

C Nutritional support is a priority in the care of individuals with PD. Such patients may benefit from meals that are smaller and more frequent than normal and which are easy to chew and swallow. Multivitamins are not necessary at each meal, and vitamin intake, along with protein intake, must be monitored to prevent contraindications with medications. It is likely premature to introduce a minced or pureed diet and a low carbohydrate diet is not indicated. Reference: 1509

A client with paraplegia is scheduled to participate in a rehabilitation program. The client states, "I do not understand the need for rehabilitation; the paralysis will not go away and it will not get better." How does the nurse respond? a. "If you do not want to participate in the rehabilitation program, I will cancel the order." b. "Your doctor has helped many clients with your injury and has ordered a rehabilitation program to help you." c. "The rehabilitation program will teach you how to maintain the functional ability you have and prevent further disability." d. "When new discoveries are made regarding paraplegia, people in rehabilitation programs will benefit first."

C Participation in rehabilitation programs has many purposes, including prevention of disability, maintenance of functional ability, and restoration of function. The other responses do not meet the client's needs.

What is most important finding for you to act on for a patient who had a craniotomy? A. Sodium: 134 mEq/L B. While blood cell (WBC) count: 11,000/μL C. Urine specific gravity: 1.001 D. Blood urea nitrogen (BUN): 25 mg/dL

C Patients need frequent monitoring for sodium regulation, onset of diabetes insipidus, and severe hypovolemia. Normal specific gravity for urine should not be below 1.003 and this low value is a priority.

The patient had a stroke on the left side of the brain (right hemiplegia). He is eating dinner and suddenly bursts into tears. How do you respond? A. Ask the patient about his feelings. B. Give him a tissue. C. Distract the patient. D. Obtain an antidepressant for patient.

C Patients who have had strokes often exhibit emotional responses that are not appropriate for the situation. The behavior is out of context and often is unrelated to the underlying emotional state of the patient. Initially it is important to just distract the patient.

Which sensory-perceptual deficit is associated with a left-brain stroke? A. Overestimation of physical abilities B. Difficulty judging position and distance C. Slow and possibly fearful performance of tasks D. Impulsivity and impatience at performing tasks

C Patients with a left-brain stroke (right hemiplegia) commonly are slower in organization and performance of tasks and may have a fearful, anxious response to a stroke. Overconfidence, spatial disorientation, and impulsivity are more commonly associated with a right-brain stroke.

Which gastrointestinal complication would you expect in the patient with multiple sclerosis? A. Diarrhea B. Bowel obstruction C. Constipation D. Rectal prolapse

C Problems with defecation usually involve constipation rather than fecal incontinence. Reference: 1502

A 65-year-old woman was just diagnosed with Parkinson's disease. The priority nursing intervention is A. searching the Internet for educational videos. B. evaluating the home for environmental safety. C. promoting physical exercise and a well-balanced diet. D. designing an exercise program to strengthen and stretch specific muscles.

C Promotion of physical exercise and a well-balanced diet are major concerns for nursing care for patients with Parkinson's disease. Reference: 1509

The nurse is reviewing a client's prescription for sumatriptan succinate (Imitrex). Which condition in this client's medical history does the nurse report to the health care provider? a. Bronchial asthma b. Gonorrhea c. Prinzmetal's angina d. Chronic kidney disease

C Sumatriptan succinate effectively reduces pain and other associated symptoms of migraine by binding to serotonin receptors and triggering cranial vasoconstriction. Vasoconstrictive effects are not confined to the cranium and can cause coronary vasospasm in clients with Prinzmetal's angina. The other conditions would not affect the client's treatment.

During the nursing assessment the patient states that she is experiencing headaches bilaterally that are described as pressing or tightening. The headaches last from a few minutes to days and the pain is described as a 5 on a scale of 1 to 10. These signs and symptoms are consistent with which type of headache? A. Sinus B. Cluster C. Tension D. Migraine

C Tension-type headache, the most common type of headache, is characterized by its bilateral location and pressing/tightening quality. Tension-type headaches are usually of mild or moderate intensity and not aggravated by physical activity.

What is the purpose of the blood-brain barrier? A. To protect the brain by cushioning B. To inhibit damage from external trauma C. To keep harmful agents away from brain tissue D. To provide the blood supply to brain tissue

C The blood-brain barrier is a physiologic barrier between capillaries and brain tissue. The structure of the brain's capillaries is different from others, and substances that are harmful are not allowed to enter brain tissue. Lipid-soluble compounds enter the brain easily, but water-soluble and ionized drugs enter slowly. The spinal fluid and meninges help cushion the brain. The skull protects from external trauma. Blood is supplied to the brain from the internal carotid arteries and the vertebral arteries.

The nurse is caring for a client who has a vertebral fracture. Which intervention does the nurse implement to prevent deterioration of the client's neurologic status? a. Reorient the client to time, place, and person. b. Administer the Mini-Mental State Examination. c. Immobilize the affected portion of the spinal column. d. Reposition the client every 2 hours.

C The nurse keeps the client in optimal body alignment at all times, avoiding flexion and extension at the site of vertebral injury, to prevent further cord injury or irritability from bone fragments. A brace, traction, or external fixation may be used for this purpose. The other interventions would not prevent deterioration of the client's neurologic status. Assessments would assist with the recognition of neurologic changes but would not prevent them.

The nurse is planning care for a client who has a spinal cord injury. Which interdisciplinary team member does the nurse consult with to assist the client with activities of daily living? a. Social worker b. Physical therapist c. Occupational therapist d. Case manager

C The occupational therapist instructs the client in the correct use of all adaptive equipment. In collaboration with the therapists, the nurse instructs family members or the caregiver about transfer skills, feeding, bathing, dressing, positioning, and skin care. The other team members are consulted to assist the client with unrelated issues.

You are assigned to four patients on the clinical unit. Which patient should you assess first? A. Patient with a skull fracture whose nose is bleeding B. Elderly patient with a stroke who is confused and whose daughter is present C. Patient with meningitis who is suddenly agitated and reporting a headache of 10 on a 0 to 10 scale D. Patient who had a craniotomy for a brain tumor 3 days earlier and has continued emesis

C The patient with meningitis should be seen first. Patients with meningitis must be observed closely for manifestations of increased ICP, which is thought to be a result of swelling around the dura and increased cerebrospinal fluid (CSF) volume. Sudden change in the level of consciousness or a change in behavior along with a sudden, severe headache may indicate an acute episode of increased ICP. The postoperative cranial surgery patient should be seen second; although nausea and vomiting are common after cranial surgery and can result in increased ICP. Nausea and vomiting should be treated with antiemetics. The patient with a skull fracture needs to be evaluated for CSF leakage occurring with the nosebleed and should be seen third. Confusion after a stroke may be expected; the patient should be safe with a family member present. Reference: 1452-1453

What is the most likely cause of a brain abscess? A. Secondary metastasis from another site B. Infection from a tick or mosquito bite C. Direct extension from a sinus infection D. Spread of Neisseria meningitidis infection

C The primary cause of a brain abscess is direct extension from ear, tooth, mastoid, or sinus infection. Secondary metastasis is the source of brain tumors. Tick or mosquito bites are associated with viral meningitis. N. meningitidis is a cause of bacterial meningitis.

Which nursing diagnosis is likely to be a priority in the care of a patient with myasthenia gravis? A. Acute confusion B. Bowel incontinence C. Activity intolerance D. Disturbed sleep pattern

C The primary feature of MG is fluctuating weakness of skeletal muscle. Bowel incontinence and confusion are unlikely signs of MG, and while sleep disturbance is likely, activity intolerance is usually of primary concern.

Vascular dementia is associated with A. transient ischemic attacks. B. bacterial or viral infection of neuronal tissue. C. cognitive changes resulting from cerebral ischemia. D. abrupt changes in cognitive function that are irreversible

C Vascular dementia is the loss of cognitive function resulting from ischemic, ischemic-hypoxic, or hemorrhagic brain lesions caused by cardiovascular disease. This type of dementia is the result of decreased blood supply from narrowing and blocking of arteries that supply the brain.

Which nursing intervention is most appropriate when communicating with a patient suffering from aphasia after a stroke? A. Present several thoughts at once so that the patient can connect the ideas. B. Ask open-ended questions to provide the patient the opportunity to speak. C. Use simple, short sentences accompanied by visual cues to enhance comprehension. D. Finish the patient's sentences to minimize the frustration associated with slow speech.

C When communicating with a patient with aphasia, you should present one thought or idea at a time; ask questions that can be answered with yes, no, or another simple word; use visual cues; and allot time for the individual to comprehend and respond to the conversation.

Which nursing intervention is most appropriate when caring for patients with dementia? A. Avoid direct eye contact. B. Lovingly call the patient "honey" or "sweetie." C. Give simple directions, focusing on one thing at a time. D. Treat the patient according to their age-related behavior.

C When dealing with patients with dementia, tasks should be simplified, giving directions using gestures or pictures and focusing on one thing at a time. It is best to treat these patients as adults, with respect and dignity, even when their behavior is childlike. You should use gentle touch and direct eye contact. Calling the patient "honey" or "sweetie" can be condescending and does not demonstrate respect.

Nursing management of a patient with a brain tumor includes (select all that apply) A. discussing with the patient methods to control inappropriate behavior. B. using diversion techniques to keep the patient stimulated and motivated. C. assisting and supporting the family in understanding changes in behavior. D. limiting self-care activities until the patient has regained maximum physical functioning. E. planning for seizure precautions and teaching the patient and caregiver about antiseizure drugs.

C,E Nursing interventions should be based on a realistic appraisal of the patient's condition and prognosis after cranial surgery. You should provide support and education to the caregiver and family related to the patient's behavior changes. You should be prepared to manage seizures and teach the caregiver and family about antiseizure medications and how to manage a seizure. The overall goal is to foster patient independence for as long as possible and to the highest degree possible. You should decrease stimuli in the patient's environment to prevent increases in ICP.

While the nurse performs ROM on an unconscious patient with increased ICP, the patient experiences severe decerebrate posturing reflexes. The nurse should a. use restraints to protect the patient from injury b. administer CNS depressants to lightly sedate the patient c. perform the exercises less frequently because posturing can increase ICP d. continue the exercises because they are necessary to maintain musculoskeletal function

C. Perform the exercises less frequently because posturing can increase ICP- If reflex posturing occurs during ROM or positioning of the patient, these activities should be done less frequently until the patient's condition stabilizes, because posturing can case increases in ICP. Neither restraints nor CNS depressants would be indicated.

A 54-year old man is recovering from a skull fracture with a subacute subdural hematoma. He has return of motor control and orientation but appears apathetic and has reduced awareness of his environment. When planning discharge or the patient, the nurse explains to the patient and the family that a. continuous improvement in the patient's condition should occur until he has returned to pre trauma status b. the patient's complete recovery may take years, and the family should plan for his long term dependent care c. the patient is likely to have long term emotional and mental changes that may require continued professional help d. role changes in family members will be necessary because the patient will be dependent on his family for care and support

C. Residual mental and emotional changes of brain trauma with personality changes are often the most incapacitating problems following head injury and are common in patients who have been comatose longer than 6 hours. Families must be prepared for changes in the patient's behavior to avoid family-patient friction and maintain family functioning, and professional assistance may be required. There is no indication he will be dependent on others for care, but he likely will not return to pre trauma status

A thrombus that develops in a cerebral artery does not always cause a loss of neurologic function because a. the body can dissolve the atherosclerotic plaques as they form b. some tissues of the brain do not require constant blood supply to prevent damage c. circulation through the circle of Willis may provide blood supply to the affected area of the brain d. neurologic deficits occur only when major arteries are occluded by thrombus formation around an atherosclerotic plaque

C: Circulation through the circle of Willis may provide blood supply to the affected area of the brain. The communication between cerebral arteries in the circle of Willing provides a collateral circulation, which may maintain circulation to an area of the brain if its original blood supply is obstructed. ALl areas of the brain require constant blood supply, and atherosclerotic plaques are not readily reversed. Neurologic deficits can result from ischemia cause by many factors.

The incidence of ischemic stroke in patients with TIAs and other risk factors is reduced with administration of a. furosemide (Lasix) b. lovastatin (Mevacor) c. daily low dose aspirin d. nimodipine (Nimotop)

C: Daily low dose aspirin- the administration of antiplatelet agents, such as aspirin, dipyridamole (Persantine), and ticlopdipine (Ticlid), reduces the incidence of stroke in those at risk. Anticoagulants are also used for prevention of embolic strokes but increase the risk for hemorrhage. Diuretics are not indicated for stroke prevention other than for their role in controlling BP, and antilipemic agents have bot been found to have a significant effect on stroke prevention. The calcium channel blocker nimodipine is used in patients with subarachnoid hemorrhage to decrease the effects of vasospasm and minimize tissue damage. P.S. I freaking love you and good luck on the final!!

A patient with a stroke has a right sided hemiplegia. The nurse prepares family members to help control behavior changes seen with this type of stroke by teaching them to a. ignore undesirable behaviors manifested by the patient b. provide directions to the patient verbally in small steps c. distract the patient from inappropriate emotional responses d. supervise all activities before allowing the patient to pursue them independently

C: Distract the patient from inappropriate emotional responses- patients with left-sided brain damage from stroke often experience emotional lability, inappropriate emotional responses, mood swings, and uncontrolled tears or laughter disproportionate or out of context with the situation. The behavior is upsetting and embarrassing to both the patient and the family, and the patient should be distracted to minimize its presence. Patients with right-brain damage often have impulsive, rapid behavior that supervision and direction.

In promoting health maintenance for prevention of strokes, the nurse understands that the highest risk for the most common type of stroke is present in a. African Americans b. women who smoke c.individuals with hypertension and diabetes d. those who are obese with high dietary fat intake

C: Individuals with hypertension and diabetes- The highest risk factors for thrombotic stroke are hypertension and diabetes. African Americans have a higher risk for stroke than do white persons but probably because they have a greater incidence of hypertension. Factors such as obesity, diet high in saturated fats and cholesterol, cigarette smoking, and excessive alcohol use are also risk factors but carry less risk than hypertension.

A carotid endarterectomy is being considered as a treatment for a patient who has had several TIAs. The nurse explains to the patient that this surgery a. is used to restore blood to the brain following an obstruction of a cerebral artery b. involves intracranial surgery to join a superficial extracranial artery to an intracranial artery c. involves removing an atherosclerotic plaque in the carotid artery to prevent an impending stroke d. is sued to open a stenosis in a carotid artery with a balloon and stent to restore cerebral circulation

C: Involves removing an atherosclerotic plaque in the carotid artery to prevent an impending stroke- An endarterectomy is a removal of an atherosclerotic plaque, and plaque in the carotid artery may impair circulation enough to cause a stroke. A carotid endarterectomy is performed to prevent a cerebrovascular accident (CVA), as are most other surgical procedures. An extacranial-intracranial bypass involves cranial surgery to bypass a sclerotic intacranial artery. Percutaneous transluminal angioplasty uses a balloon to compress stenotic areas in the carotid and vertebrobasilar arteries and often includes inserting a stent to hold the artery open.

A patient with right hemisphere stroke has a nursing diagnosis of unilateral neglect related to sensory perceptual deficits. During the patient's rehabilitation, it is important for the nurse to a. avoid positioning the patient on the affected side b. place all objects for care on the patient's unaffected side c. teach the patient to care consciously for the affected side d. protect the affected side from injury with pillows and supports

C: Teach the patient to care consciously for the affected side- unilateral neglect, or neglect syndrome, occurs when the patient with a stroke is unaware of the affected side of the body, which puts the patient at risk for injury. During the acute phase, the affected side is cared for by the nurse with positioning and support, during rehabilitation the patient is taught to care consciously for and attend to the affected side of the body to protect it from injury. Patients may be positioned on the affected side for up to 30 minutes.

The neurologic functions that are affected by a stroke are primarily related to a. the amount of tissue area involved b. the rapidity of onset of symptoms c. the brain area perfused by the affected artery d. the presence or absence of collateral circulation

C: The brain area perfused by the affected artery- clinical manifestation of altered neurologic function differ, depending primarily on the specific cerebral artery involved and the area of the brain that is perfused by the artery. The degree of impairment depends on rapidity of onset, the size of the lesion, and the presence of collateral circulation.

An appropriate food for a patient with a stroke who has mild dysphagia is a. fruit juices b. pureed meat c. scrambled eggs d. fortified milkshakes

C: scrambled eggs- soft foods that provide enough texture, flavor, and bulk to stimulate swallowing should be used for the patient with dysphasia. Thin liquids are difficult to swallow, and patients may not be able to control them in the mouth. Pureed foods are often too bland and to smooth, and milk products should be avoided because they tend to increase the viscosity of mucus and increase salivation.

The patient with bacterial meningitis has irritation of cranial nerve (CN) II. What symptom would you expect the patient to have? A. Inability to hear whispered words B. Abnormal pronation and supination test result C. Papilledema D. Dysphagia

CN II is the optic nerve, and when compressed from the increased ICP, papilledema (swelling of the optic disc) is often present and blindness may occur. The acoustic nerve is CN VIII. Assessing abnormal pronation or supination is a coordination/balance test for cerebellar function. Dysphagia can be a symptom of CN IX and CN X abnormalities.

A patient has a nursing diagnosis of disturbed sensory perception related to sleep deprivation. Which action will the nurse include in the plan of care?

Cluster nursing activities so that the patient has uninterrupted rest periods. rationale: Clustering nursing activities and providing uninterrupted rest periods will minimize sleep-cycle disruption. Sedative and opioid medications tend to decrease the amount of rapid eye movement (REM) sleep and can contribute to sleep disturbance and disturbed sensory perception. Silencing the alarms on the cardiac monitors would be unsafe in a critically ill patient, as would discontinuing assessments during the night.

Which finding indicates that rehydration is complete and hypovolemic shock has been successfully treated in a patient? 1. CVP = 8 mm Hg 2. MAP = 45 mm Hg 3. Urinary output of 0.1 mL/kg/hr 4. Hct = 54%

Correct Answer: 1 Rationale 1: A CVP reading of 8 mm Hg is within normal range and rehydration has been restored. Rationale 2: The mean arterial pressure (MAP) should be between 60 to 70 mm Hg as evidence of positive fluid resuscitation efforts. Rationale 3: Urinary output to reflect adequate rehydration begins at 0.5 to 1 mL/kg/hr. Therefore, 0.1 mL is too small and renal insufficiency may be present due to inadequate circulating blood volume. Rationale 4: Hematocrit (Hct) is the percentage of the number of RBCs per fluid volume. The normal range is 35% to 45% for an adult. The higher percentage represents a decreased fluid-to-cell ratio, which implies a fluid deficit and rehydration is not complete. An Hct of 54% is critical and increases the risk of clots, strokes, and other vessel obstruction from potential hemolysis and sluggishness of cellular movements.

A patient is being treated for pericarditis. The nurse will plan interventions to prevent the onset of which type of shock? 1. Obstructive 2. Hypovolemic 3. Distributive 4. Cardiogenic

Correct Answer: 1 Rationale 1: Acute pericarditis and the development of fluid accumulation in the pericardial space can lead to the development of obstructive shock. Rationale 2: Pericarditis is not a risk factor for the development of hypovolemic shock. Rationale 3: Pericarditis is not a risk factor for the development of distributive shock. Rationale 4: Pericarditis is not a risk factor for the development of cardiogenic shock.

Which assessment finding indicates that an infusion of intravenous epinephrine 4 mcg/min is effective in the treatment of a patient with anaphylactic shock? 1. Reduced wheezing 2. Heart rate 55 and regular 3. Blood pressure 98/50 mm Hg 4. Respiratory rate 28

Correct Answer: 1 Rationale 1: An expected action for epinephrine is bronchodilation as evidenced by less wheezing. Rationale 2: Epinephrine increases heart rate. Rationale 3: Epinephrine increases blood pressure. Rationale 4: Epinephrine lowers the respiratory rate. This respiratory rate indicates that epinephrine has not been effective.

A patient is diagnosed with cardiac tamponade. When planning care, the nurse will include interventions to address which type of shock? 1. Obstructive 2. Hypovolemic 3. Distributive 4. Cardiogenic

Correct Answer: 1 Rationale 1: Cardiac tamponade can lead to obstructive shock. Rationale 2: Cardiac tamponade will not lead to hypovolemic shock. Rationale 3: Cardiac tamponade will not lead to distributive shock. Rationale 4: Cardiac tamponade will not lead to cardiogenic shock.

A patient is demonstrating signs of obstructive shock but the cause has yet to be determined. Which finding indicates the patient is experiencing a pulmonary embolism as the cause for obstructive shock? 1. Chest pain 2. Hypotension 3. Tachycardia 4. Oliguria

Correct Answer: 1 Rationale 1: Chest pain is a symptom associated with a massive pulmonary embolus. Rationale 2: Hypotension is seen in other causes of obstructive shock and is not a symptom that differentiates the cause as being from a pulmonary embolus. Rationale 3: Tachycardia is seen in other causes of obstructive shock and is not a symptom that differentiates the cause as being from a pulmonary embolus. Rationale 4: Oliguria is seen in other causes of obstructive shock and is not a symptom that differentiates the cause as being from a pulmonary embolus.

A patient is experiencing an anaphylactic reaction to a medication. The nurse is concerned that the patient will develop distributive shock because: 1. The release of histamine causes vasodilation with plasma leakage. 2. Sympathetic innervation is interrupted. 3. Microorganisms overwhelm the vascular system. 4. Parasympathetic innervation functions are unopposed.

Correct Answer: 1 Rationale 1: In an anaphylactic reaction leading to distributive shock, the release of histamine causes vasodilation with plasma leakage. Vasodilation leads to profound hypotension, hypovolemia from fluid extravasation, reduced reload, and reduced cardiac output. Rationale 2: Sympathetic innervation is not interrupted in an anaphylactic reaction. Rationale 3: Microorganisms do not overwhelm the vascular system in an anaphylactic reaction. Rationale 4: Parasympathetic innervation functioning unopposed is not a characteristic of an anaphylactic reaction.

A patient is receiving phenylephrine 50 mcg/min as treatment for shock. Which assessment finding indicates this medication is effective? 1. Blood pressure 110/68 mm Hg 2. Heart rate 110 3. Respiratory rate 12 and regular 4. Decreased peripheral pulses

Correct Answer: 1 Rationale 1: The expected effect of this medication is an increase in blood pressure. Rationale 2: Phenylephrine does not increase the heart rate. Rationale 3: Phenylephrine does not affect the respiratory rate. Rationale 4: Decreased peripheral pulses is a side/toxic effect of this medication.

The nurse is concerned that a patient is at risk for developing obstructive shock because of which assessment findings? Note: Credit will be given only if all correct choices and no incorrect choices are selected. Standard Text: Select all that apply. 1. Age 80 2. History of atrial fibrillation 3. Bacteremia 4. T3 spinal cord injury 5. Latex allergy

Correct Answer: 1,2 Rationale 1: Advanced age increases the risk for development of pulmonary emboli, which is one cause of obstructive shock. Rationale 2: Atrial fibrillation increases the risk for developing pulmonary emboli, which is one cause of obstructive shock. Rationale 3: Bacteremia increases a patient's risk of developing septic shock and not obstructive shock. Rationale 4: A spinal cord injury increases the risk for developing distributive shock and not obstructive shock. Rationale 5: A latex allergy increases the risk for developing distributive shock and not obstructive shock.

A patient is experiencing acute respiratory distress after eating an item of a known food allergy. What interventions will the nurse implement when providing emergency care to this patient? Note: Credit will be given only if all correct choices and no incorrect choices are selected. Standard Text: Select all that apply. 1. Administer epinephrine 1:1000 intramuscularly. 2. Apply oxygen via face mask as prescribed. 3. Provide diphenhydramine 25 mg intravenous. 4. Administer vasopressin. 5. Prepare to administer antithrombolytic agents as prescribed.

Correct Answer: 1,2,3 Rationale 1: Epinephrine produces bronchodilation, improving the respiratory status. The route of administration is initially intramuscular. Rationale 2: Supplemental oxygen is used in the treatment of anaphylactic shock. Rationale 3: Hydrogen ion blockers such as diphenhydramine may be administered to block the histamine effects. Rationale 4: Vasopressin is not used in the treatment of anaphylactic shock. Rationale 5: Antithrombolytic agents are not used in the treatment of anaphylactic shock.

During an assessment the nurse is concerned that a patient is developing cardiogenic shock. What did the nurse assess in this patient? Note: Credit will be given only if all correct choices and no incorrect choices are selected. Standard Text: Select all that apply. 1. Systolic blood pressure 82 mm Hg 2. Capillary refill 10 seconds 3. Crackles bilateral lung bases 4. Heart rate 55 and regular 5. Warm dry skin

Correct Answer: 1,2,3 Rationale 1: Hypotension is a manifestation of cardiogenic shock. Rationale 2: Delayed capillary refill is a manifestation of cardiogenic shock. Rationale 3: Crackles are a manifestation of cardiogenic shock. Rationale 4: Bradycardia is not a manifestation of cardiogenic shock. Rationale 5: Warm dry skin is not a manifestation of cardiogenic shock.

The nurse is caring for a patient recovering from a spinal cord injury sustained during a motor vehicle crash. What assessment findings indicate that the patient is developing neurogenic shock? Note: Credit will be given only if all correct choices and no incorrect choices are selected. Standard Text: Select all that apply. 1. Hypotension 2. Bradycardia 3. Warm dry skin 4. Abdominal cramps 5. Palpitations

Correct Answer: 1,2,3 Rationale 1: Hypotension is a manifestation of neurogenic shock because of the loss of autonomic reflexes. Rationale 2: Bradycardia occurs because of the loss of sympathetic innervation. Rationale 3: Warm dry skin occurs because of a loss of cutaneous control of sweat glands. Rationale 4: Abdominal cramping is not a manifestation of neurogenic shock. Rationale 5: Palpitations are not seen in neurogenic shock.

A patient, experiencing vasodilation, is diagnosed with distributive shock. The nurse will assess the patient for which etiologies? Note: Credit will be given only if all correct choices and no incorrect choices are selected. Standard Text: Select all that apply. 1. Sepsis 2. Spinal cord injury 3. Anaphylaxis 4. Hemorrhage 5. Pulmonary embolism

Correct Answer: 1,2,3 Rationale 1: One etiology of distributive shock is sepsis. Rationale 2: One etiology of distributive shock is spinal cord injury. Rationale 3: One etiology of distributive shock is anaphylaxis. Rationale 4: Hemorrhage is not an etiology of distributive shock. Rationale 5: Pulmonary embolism is not an etiology of distributive shock.

A patient being treated for cardiogenic shock is being hemodynamically monitored. Which findings are consistent with the patient's diagnosis? Note: Credit will be given only if all correct choices and no incorrect choices are selected. Standard Text: Select all that apply. 1. Elevated pulmonary arterial wedge pressure 2. Elevated central venous pressure 3. Elevated systemic vascular resistance index 4. Elevated mean arterial pressure 5. Elevated stroke volume

Correct Answer: 1,2,3 Rationale 1: This finding is consistent with pulmonary vascular congestion. Rationale 2: This finding is consistent with fluid volume overload. Rationale 3: This finding is consistent with pulmonary vascular congestion. Rationale 4: This finding is not consistent with cardiogenic shock. Rationale 5: This finding is not consistent with cardiogenic shock.

A patient is brought to the emergency department with manifestations of anaphylactic shock. What will the nurse assess as possible causes for this disorder? Note: Credit will be given only if all correct choices and no incorrect choices are selected. Standard Text: Select all that apply. 1. Recent bee sting 2. Ingestion of drugs 3. History of latex allergy 4. Recent diagnostic imaging tests 5. Recent myocardial infarction

Correct Answer: 1,2,3,4 Rationale 1: Venoms such as bee stings can trigger anaphylactic shock. Rationale 2: Drugs can trigger anaphylactic shock. Rationale 3: Latex can trigger anaphylactic shock. Rationale 4: Contrast media for diagnostic tests can trigger anaphylactic shock. Rationale 5: Myocardial infarction is not a trigger for anaphylactic shock.

The nurse is explaining the mechanism of a pulmonary embolism to the family of a patient diagnosed with the disorder. Place in order the steps the nurse will use to instruct the family about this disease process. Standard Text: Click and drag the options below to move them up or down. Choice 1. Blood clot causes backup of blood in the right ventricle. Choice 2. Blood clot blocks blood to the left ventricle. Choice 3. Left ventricle does not get enough blood to pump through the body. Choice 4. Amount of blood the heart has to pump to the body drops. Choice 5. Blood pressure drops. Choice 6. Amount of blood going to the body drops.

Correct Answer: 1,2,3,4,5,6 Rationale 1: The obstruction caused by the pulmonary embolism increases the afterload of the right ventricle, causing right ventricular failure. Rationale 2: The embolus prevents adequate blood flow from the pulmonary circulation to the left ventricle. Rationale 3: Because blood flow from the pulmonary circulation is blocked, left ventricular preload drops. Rationale 4: Because left ventricular preload is decreased, there is not enough blood in the heart to pump, causing decreased cardiac output. Rationale 5: A lack of blood circulating will lead to hypotension. Rationale 6: When the blood is backed up and is not being pumped into the general circulation, tissue perfusion is reduced.

The nurse is preparing medications for a patient being treated for cardiogenic shock. Which medications will the nurse most likely provide to this patient? Note: Credit will be given only if all correct choices and no incorrect choices are selected. Standard Text: Select all that apply. 1. Dopamine 2. Norepinephrine 3. Dobutamine 4. Epinephrine 5. Phenylephrine

Correct Answer: 1,2,3,5 Rationale 1: Dopamine is commonly used in the treatment of cardiogenic shock. Rationale 2: Norepinephrine is commonly used in the treatment of cardiogenic shock. Rationale 3: Dobutamine may be used in the patient with cardiogenic shock who has an adequate blood pressure. Rationale 4: Epinephrine is not used in the treatment of cardiogenic shock. Rationale 5: Phenylephrine is a vasopressor and may be used in the patient with cardiogenic shock who is receiving dobutamine.

Which finding indicates that a patient is experiencing increased peripheral resistance and vasoconstriction? 1. Strong bounding pulse with deep red coloring 2. Pale, cool extremities with decreased pulses 3. Increased venous engorgement with strong pulses 4. Faster than normal capillary refill time

Correct Answer: 2 Rationale 1: An increased blood supply would increase color and bounding pulses as seen with vasodilation (blood engorgement) and is not present with increased peripheral resistance and vasoconstriction. Rationale 2: Increased peripheral resistance causes the blood supply to decrease and results in decreased blood to the tissues, which causes pallor and decreased skin temperatures. The pulses would decrease in intensity with a decreased blood supply. Rationale 3: Venous engorgement would not result from vasoconstriction of the arteries. Strong pulses would not be present with vasoconstriction from increased peripheral resistance. Rationale 4: Capillary refill times are delayed or slowed due to decreased blood flow through the vessels caused by the vasoconstriction from increased peripheral resistance.

A patient is demonstrating pulmonary edema, hypotension, and delayed capillary refill. The nurse suspects the patient is experiencing which type of shock? 1. Hypovolemic 2. Cardiogenic 3. Anaphylactic 4. Obstructive

Correct Answer: 2 Rationale 1: Pulmonary edema would not be present in hypovolemic shock. Rationale 2: In cardiogenic shock, there is a low cardiac output, hypotension, and pulmonary edema. Rationale 3: Pulmonary edema would not be present in anaphylactic shock. Rationale 4: Pulmonary edema would not be present in obstructive shock

Which laboratory finding should cause the nurse to suspect that a patient is developing hypovolemic shock? 1. Serum sodium of 130 mEq/L (130 mmol/L) 2. Metabolic acidosis validated by arterial blood gases 3. Serum lactate of 3 mmol/L 4. SvO2 greater than 80%

Correct Answer: 2 Rationale 1: The sodium level in hypovolemic shock is elevated above the normal values of 135 to 145 mEq/L, not reduced. Rationale 2: Metabolic acidosis is present due to an accumulation of carbonic acid, leaving a bicarbonate deficit from decreased tissue perfusion. Rationale 3: Serum lactate is greater than 4 mmol/L as a result of tissue ischemia, hypoxia, and breakdown from decreased blood flow with hypovolemic shock. Rationale 4: SvO2 (mixed venous oxygen saturation) would be less than 60% due to decreased circulating blood volume or decrease in cells to carry the oxygen. Therefore, O2 is carried less efficiently and decreased, not increased.

Which life-threatening complications would the nurse anticipate developing in the patient being treated for hypovolemic shock? Note: Credit will be given only if all correct choices and no incorrect choices are selected. Standard Text: Select all that apply. 1. Fluid volume overload 2. Renal insufficiency 3. Cerebral ischemia 4. Gastric stress ulcer 5. Pulmonary edema

Correct Answer: 2,3 Rationale 1: Fluid volume overload is not an identified complication of hypovolemic shock. Rationale 2: Renal insufficiency is a serious complication because of the prerenal etiology of hypovolemia. Rationale 3: Early identification and correction of the fluid volume deficit in hypovolemic shock is necessary to prevent cerebral ischemia. Rationale 4: Although physiologic stress can increase the risk for the development of stress ulcers, it is not considered one of the common or life-threatening complications of hypovolemic shock. Rationale 5: Pulmonary edema is not an identified complication of hypovolemic shock.

What will the nurse identify as symptoms of hypovolemic shock in a patient? Note: Credit will be given only if all correct choices and no incorrect choices are selected. Standard Text: Select all that apply. 1. Temperature of 97.6°F (36.4°C) 2. Restlessness 3. Decrease in blood pressure of 20 mm Hg when the patient sits up 4. Capillary refill time greater than 3 seconds 5. Sinus bradycardia of 55 beats per minute

Correct Answer: 2,3,4 Rationale 1: Fever will increase oxygen demands but is unrelated to hypovolemic shock unless prolonged fever has caused severe dehydration, reducing the circulating blood volume. Hypovolemic shock reduces temperatures by peripheral shunting of blood away from the extremities and reducing the core metabolic rate. Rationale 2: Due to decreased blood flow to the brain and peripheral areas when blood is shunted to maintain the vital organs, cerebral hypoxia occurs, leading to a change in mental status. Rationale 3: Orthostatic hypotension is a manifestation of hypovolemic shock. Rationale 4: Due to decreased blood flow to the brain and peripheral areas when blood is shunted to maintain the vital organs, capillary refill time will be reduced. Rationale 5: Bradycardia is not present. The compensatory response is to increase the heart rate to circulate the blood faster to make up for the fluids that are not present in hypovolemic shock.

The nurse recognizes that which patient would be most likely to develop hypovolemic shock? A patient with: 1. Decreased cardiac output 2. Severe constipation, causing watery diarrhea 3. Ascites 4. Syndrome of inappropriate ADH (SIADH)

Correct Answer: 3 Rationale 1: Although ECG changes reflect the effectiveness of the heart's pumping when circulating the blood, it is not a risk factor for hypovolemic shock, which reflects a decreased circulating volume from either blood or fluid losses within the intravascular system. Rationale 2: Severe constipation does not affect the circulating blood volume. Rationale 3: Third spacing shifts move the fluids from the intravascular space into the interstitial space, causing a drop in the circulating blood volume. Therefore, third spacing is a risk factor for the development of hypovolemic shock. Rationale 4: Overhydration does not lead to hypovolemic shock. It leads to fluid overload, which might cause cardiogenic shock, congestive heart failure, and pulmonary edema.

The nurse, caring for a patient in hypovolemic shock, will not utilize a hypotonic solution for fluid resuscitation because hypotonic solutions: 1. Move quickly into the interstitial spaces and can cause third spacing 2. Stay longer to expand the intravascular space but deplete intracellular fluid levels 3. Do not stay in the intravascular space long enough to expand the circulating blood volume 4. Need a smaller bore needle to run at a slower rate to keep the intravascular space low

Correct Answer: 3 Rationale 1: Hypotonic solutions do not cause third spacing. Rationale 2: Hypotonic solutions do not stay in the intravascular space long enough to expand the circulating blood volume. Rationale 3: Hypotonic solutions do not stay in the intravascular space long enough to expand the circulating blood volume. Rationale 4: The bore size of the needle does not affect the displacement or shifting of fluids.

A patient with neurogenic shock is demonstrating bradycardia. What action will the nurse take at this time? 1. Limit patient movement. 2. Prepare to administer crystalloids. 3. Administer phenylephrine as prescribed. 4. Administer atropine as prescribed.

Correct Answer: 4 Rationale 1: Limiting movement will not correct bradycardia in the patient with neurogenic shock. Rationale 2: Crystalloids are used to correct vasodilation. Rationale 3: Phenylephrine is used in the patient with neurogenic shock to correct hypotension. Rationale 4: Bradycardia in neurogenic shock is corrected by the administration of atropine at the dose of 0.5 to 1.0 mg intravenous every 5 minutes to a total dose of 3 mg.

When the nurse applies a painful stimulus to the nailbeds of an unconscious patient, the patient responds with internal rotation, adduction, and flexion of the arms. The nurse documents this as a. decorticate posturing. b. decerebrate posturing. c. localization of pain. d. flexion withdrawal.

Correct Answer: A Rationale: Internal rotation, adduction, and flexion of the arms in an unconscious patient is documented as decorticate posturing. Extension of the arms and legs is decerebrate posturing. Because the flexion is general, it does not indicate localization of pain or flexion withdrawal. Cognitive Level: Comprehension Text Reference: p. 1472 Nursing Process: Assessment NCLEX: Physiological Integrity

When caring for a patient who has had a head injury, which assessment information is of most concern to the nurse? a. The blood pressure increases from 120/54 to 136/62. b. The patient is more difficult to arouse. c. The patient complains of a headache at pain level 5 of a 10-point scale. d. The patient's apical pulse is slightly irregular.

Correct Answer: B Rationale: The change in level of consciousness (LOC) is an indicator of increased ICP and suggests that action by the nurse is needed to prevent complications. The change in BP should be monitored but is not an indicator of a need for immediate nursing action. Headache is not unusual in a patient after a head injury. A slightly irregular apical pulse is not unusual. Cognitive Level: Application Text Reference: p. 1470 Nursing Process: Assessment NCLEX: Physiological Integrity

A patient with possible cerebral edema has a serum sodium level of 115 mEq/L (115 mmol/L), a decreasing level of consciousness (LOC) and complains of a headache. All of the following orders have been received. Which one should the nurse accomplish first? a. Administer acetaminophen (Tylenol) 650 mg orally. b. Administer 5% hypertonic saline intravenously. c. Draw blood for arterial blood gases (ABGs). d. Send patient to radiology for computed tomography (CT) of the head.

Correct Answer: B Rationale: The patient's low sodium indicates that hyponatremia may be causing the cerebral edema, and the nurse's first action should be to correct the low sodium level. Acetaminophen (Tylenol) will have minimal effect on the headache because it is caused by cerebral edema and increased ICP. Drawing ABGs and obtaining a CT scan may add some useful information, but the low sodium level may lead to seizures unless it is addressed quickly. Cognitive Level: Application Text Reference: p. 1470 Nursing Process: Implementation NCLEX: Physiological Integrity

The nurse notes that a patient with a head injury has a clear nasal drainage. The most appropriate nursing action for this finding is to a. obtain a specimen of the fluid and send for culture and sensitivity. b. take the patient's temperature to determine whether a fever is present. c. check the nasal drainage for glucose with a Dextrostik or Testape. d. have the patient to blow the nose and then check the nares for redness.

Correct Answer: C Rationale: If the drainage is cerebrospinal fluid (CSF) leakage from a dural tear, glucose will be present. Fluid leaking from the nose will have normal nasal flora, so culture and sensitivity will not be useful. A dural tear does increase the risk for infections such as meningitis, but the nurse should first determine whether the clear drainage is CSF. Blowing the nose is avoided to prevent CSF leakage. Cognitive Level: Application Text Reference: p. 1481 Nursing Process: Implementation NCLEX: Physiological Integrity

When assessing a patient with a head injury, the nurse recognizes that the earliest indication of increased intracranial pressure (ICP) is a. vomiting. b. headache. c. change in level of consciousness (LOC). d. sluggish pupil response to light.

Correct Answer: C Rationale: LOC is the most sensitive indicator of the patient's neurologic status and possible changes in ICP. Vomiting and sluggish pupil response to light are later signs of increased ICP. A headache can be caused by compression of intracranial structures as the brain swells, but it is not unexpected after a head injury. Cognitive Level: Comprehension Text Reference: p. 1470 Nursing Process: Assessment NCLEX: Physiological Integrity

A patient admitted with a head injury has admission vital signs of temperature 98.6° F (37° C), blood pressure 128/68, pulse 110, and respirations 26. Which of these vital signs, if taken 1 hour after admission, will be of most concern to the nurse? a. Blood pressure 130/72, pulse 90, respirations 32 b. Blood pressure 148/78, pulse 112, respirations 28 c. Blood pressure 156/60, pulse 60, respirations 14 d. Blood pressure 110/70, pulse 120, respirations 30

Correct Answer: C Rationale: Systolic hypertension with widening pulse pressure, bradycardia, and respiratory changes represent Cushing's triad and indicate that the ICP has increased and brain herniation may be imminent unless immediate action is taken to reduce ICP. The other vital signs may indicate the need for changes in treatment, but they are not indicative of an immediately life-threatening process. Cognitive Level: Application Text Reference: p. 1469 Nursing Process: Assessment NCLEX: Physiological Integrity

A patient has a systemic blood pressure (BP) of 120/60 mm Hg and an intracranial pressure of 24 mm Hg. The nurse determines that the cerebral perfusion pressure (CPP) of this patient indicates a. high blood flow to the brain. b. normal intracranial pressure (ICP). c. impaired brain blood flow. d. adequate cerebral perfusion.

Correct Answer: C Rationale: The patient's CPP is 56, below the normal of 70 to 100 mm Hg and approaching the level of ischemia and neuronal death. The patient has low cerebral blood flow/perfusion. Normal ICP is 0 to 15 mm Hg. Cognitive Level: Application Text Reference: p. 1468 Nursing Process: Assessment NCLEX: Physiological Integrity

A patient with a head injury has an arterial blood pressure is 92/50 mm Hg and an intracranial pressure of 18 mm Hg. Which action by the nurse is appropriate? a. Document and continue to monitor the parameters. b. Elevate the head of the patient's bed. c. Notify the health care provider about the assessments. d. Check the patient's pupillary response to light.

Correct Answer: C Rationale: The patient's cerebral perfusion pressure is only 46 mm Hg, which will rapidly lead to cerebral ischemia and neuronal death unless rapid action is taken to reduce ICP and increase arterial BP. Documentation and monitoring are inadequate responses to the patient's problem. Elevating the head of the bed will lower the ICP but may also lower cerebral blood flow and further decrease CPP. Changes in pupil response to light are signs of increased ICP, so the nurse will only take more time doing this without adding any useful information. Cognitive Level: Analysis Text Reference: pp. 1468-1469 Nursing Process: Implementation NCLEX: Physiological Integrity

Mechanical ventilation with a rate and volume to maintain a mild hyperventilation is used for a patient with a head injury. To evaluate the effectiveness of the therapy, the nurse should a. monitor oxygen saturation. b. check arterial blood gases (ABGs). c. monitor intracranial pressure (ICP). d. assess patient breath sounds.

Correct Answer: C Rationale: The purpose of hyperventilation for a patient with a head injury is reduction of ICP, and ICP should be monitored to evaluate whether the therapy is effective. Although oxygen saturation and ABGs are monitored in patient's receiving hyperventilation, they do not provide data about whether the therapy is successful in reducing ICP. Breath sounds are assessed, but they are not helpful in determining whether the hyperventilation is effective. Cognitive Level: Application Text Reference: p. 1475 Nursing Process: Evaluation NCLEX: Physiological Integrity

What information provided by the patient can help differentiate a hemorrhagic stroke from a thrombotic stroke? A. Sensory disturbance B. A history of hypertension C. Presence of motor weakness D. Sudden onset of severe headache

D A hemorrhagic stroke usually causes sudden onset of symptoms, including neurologic deficits, headache, nausea, vomiting, decreased level of consciousness, and hypertension. Ischemic stroke symptoms may progress in the first 72 hours as infarction and cerebral edema increase. Reference: 1464

The nurse is talking to the family of a client who has Parkinson's disease. Which statement indicates that the family has a good understanding of the changes in motor movement associated with this disease? a. "I can never tell what she's thinking. She hides behind a frozen face." b. "She drools all the time so I just can't take her out anywhere." c. "I think this disease makes her nervous. She perspires all the time." d. "She has trouble chewing so I will offer bite-sized portions."

D A masklike face, drooling, and excess perspiration are common in clients with Parkinson's disease. Changes in facial expression or a masklike facies in a Parkinson's disease client can be misinterpreted. Because chewing and swallowing can be problematic, small frequent meals and a supplement are better for meeting the client's nutritional needs. The other statements indicate poor understanding of the disease process.

You know that dietary teaching has been effective when the patient with multiple sclerosis makes which meal choice? A. Hamburger, fries, vanilla shake B. Steak, scalloped potatoes, French toast, iced tea C. Ham sandwich, potato chips, glass of whole milk D. Salad with tomatoes, chicken, bran muffin, strawberries, low- fat milk

D A nutritious, well-balanced diet is essential. Although there is no standard prescribed diet, a high-protein diet with supplementary vitamins is often recommended. A diet high in roughage may help relieve the problem of constipation. Reference: 1504

The patient had a blunt head injury. What is most important for you to do before the patient's discharge? A. Have the patient sign the discharge papers. B. Teach the patient how to perform the Glasgow Coma Scale (GCS). C. Tell the patient to return if he has a headache. D. Ensure there is a responsible adult to check on the patient

D Complications from a head injury can arise 2 to 3 days later, and the discharged patient must have a responsible adult who can stay with or check on the patient. The patient may understand the instructions but without an objective observer, he or she would not be aware whether some of the key symptoms were occurring. A patient would not know how to do the GCS if impaired or confused. A headache is not a concern, but a worsening headache unrelieved by over-the-counter medications needs to be checked.

Which option indicates a sign of Cushing's triad, an indication of increased intracranial pressure (ICP)? A. Heart rate increases from 90 to 110 beats/minute B. Kussmaul respirations C. Temperature over 100.4° F (38° C) D. Heart rate decreases from 75 to 55 beats/minute

D Cushing's triad is systolic hypertension with a widening pulse pressure, bradycardia with a full and bounding pulse, and slowed respirations. The rise in blood pressure is an attempt to maintain cerebral perfusion, and it is a neurologic emergency because decompensation is imminent. The other options are not part of Cushing's triad.

What is a common cause of delirium? A. Prion protein B. Microscopic deposits of Pick bodies C. Abnormal deposits of protein α-synuclein D. Severe, acute illness

D Delirium is often the result of the interaction of the patient's underlying condition with a precipitating event. Common causes in a vulnerable (especially older) adult include fluid and electrolyte abnormalities, drug administration, surgery, acute or worsening significant illness, and withdrawal from drugs or alcohol. Prion protein is found in Creutzfeldt-Jakob disease. Microscopic deposits of Pick bodies are found in frontotemporal dementia (Pick's disease). Abnormal deposits of protein α-synuclein are found in Lewy body dementia. Reference: 1534

Dementia is defined as a A. syndrome that results only in memory loss. B. disease associated with abrupt changes in behavior. C. disease that is always due to reduced blood flow to the brain. D. syndrome characterized by cognitive dysfunction and loss of memory.

D Dementia is a syndrome characterized by dysfunction or loss of memory, orientation, attention, language, judgment, and reasoning. Personality changes and behavioral problems such as agitation, delusions, and hallucinations may result.

A client with new-onset status epilepticus is prescribed phenytoin (Dilantin). After teaching the client about this treatment regimen, the nurse assesses the client's understanding. Which statement indicates that the client understands the teaching? a. "I must drink at least 2 liters of water daily." b. "This will stop me from getting an aura before a seizure." c. "I will not be able to be employed while taking this medication." d. "Even when my seizures stop, I will take this drug."

D Discontinuing antiepileptic drugs can lead to the recurrence of seizures or status epilepticus. The client does not need to drink more water and can continue to work while taking this medication. The medication will not stop an aura before a seizure.

When assessing motor function of a patient admitted with a stroke, you notice mild weakness of the arm demonstrated by downward drifting of the extremity. How would you accurately document this finding? A. Athetosis B. Hypotonia C. Hemiparesis D. Pronator drift

D Downward drifting of the arm or pronation of the palm is identified as pronator drift. Hemiparesis is weakness of one side of the body, hypotonia describes flaccid muscle tone, and athetosis is a slow, writhing, involuntary movement of the extremities

The daughter of a client with Alzheimer's disease asks, "Will the medication my mother is taking improve her dementia?" How does the nurse respond? a. "It will help your mother live independently once more." b. "It is used to halt the advancement of Alzheimer's disease but will not cure it." c. "It will provide a steady improvement in memory but not in problem solving." d. "It will not improve dementia but can help control emotional responses."

D Drug therapy is not effective for treating dementia or halting the advancement of Alzheimer's disease. However, certain drugs may help suppress emotional disturbances and psychiatric manifestations.

The pain reliever of choice for patients experiencing a tension type headache is A. meperidine hydrochloride (Demerol). B. morphine sulfate (Morphine). C. codeine sulfate (Codeine). D. acetaminophen (Tylenol).

D Drug treatment for tension-type headache usually involves a nonopioid analgesic (e.g., aspirin, acetaminophen) used alone or in combination with a sedative, muscle relaxant, tranquilizer, or codeine.

The nurse is caring for a client with Parkinson's disease. Which intervention does the nurse implement to prevent respiratory complications in the client? a. Keep an oral airway at the bedside. b. Ensure fluid intake of at least 3 L/day. c. Teach the client pursed-lip breathing techniques. d. Maintain the head of the bed at 30 degrees or greater.

D Elevation of the back rest will help prevent aspiration. The other options will not prevent aspiration, which is the greatest respiratory complication of Parkinson's disease.

What is a key clinical manifestation of classic bacterial meningitis? A. Temperature B. Nystagmus C. Tachycardia D. Nuchal rigidity

D Fever, severe headache, vomiting, and nuchal rigidity (neck stiffness) are key clinical manifestations of meningitis.

A client who has a lower motor neuron injury experiences a flaccid bowel elimination pattern. Which action does the nurse implement to assist in relieving this client's constipation? a. Pouring warm water over the perineum b. Tapping the abdomen from left to right c. Administering daily tap water enemas d. Implementing a consistent daily time for elimination

D For the client with a lower motor neuron injury, the resulting flaccid bowel may require a bowel program for the client, which includes stool softeners, increased fluid intake, a high-fiber diet, and a consistent elimination time. The other interventions do not assist this client.

Early manifestations of amyotrophic lateral sclerosis (ALS) and multiple sclerosis (MS) are somewhat similar. Which clinical feature of ALS distinguishes it from MS? a. Dysarthria b. Dysphagia c. Muscle weakness d. Impairment of respiratory muscles

D In ALS, progressive muscle atrophy occurs until a flaccid quadriplegia develops. Eventually, the respiratory muscles are involved, and this leads to respiratory compromise.

The nurse is caring for a client with a lower motor neuron lesion who wishes to achieve bladder control. Which intervention does the nurse implement to effectively stimulate the initiation of voiding for this client? a. Stroking the inner aspect of the thigh b. Intermittent catheterization c. Digital anal stimulation d. The Valsalva maneuver

D In clients with lower motor neuron problems, such as spinal cord injury, performing a Valsalva maneuver or tightening the abdominal muscles are interventions that can initiate voiding. The other interventions do not initiate voiding.

The nurse is assessing a client who had a discectomy 6 hours ago. Which client complaint requires priority action by the nurse? a. "I am feeling tired." b. "My mouth is so dry." c. "I can't seem to relax and rest." d. "I am unable to urinate."

D Inability to void may indicate damage to the sacral spinal nerves. The other symptoms require the nurse to provide care but are not the priority or a complication of the procedure.

Treatment of medication overuse headache is A. use of an ice cap. B. skeletal muscle relaxants. C. elimination of caffeinated beverages. D. abrupt withdrawal of all nonopioid medications.

D Medication overuse headache (MOH) is the term used to describe an analgesic rebound headache. Drugs known to cause this problem are acetaminophen, aspirin, NSAIDs (e.g., ibuprofen), butalbital, sumatriptan, and opioids. Treatment involves abrupt withdrawal of the offending drug (except for opioids, which need to be tapered) and initiation of alternative drugs such as amitriptyline.

The nurse is planning to bathe a client diagnosed with meningococcal meningitis. In addition to gloves, what personal protective equipment does the nurse use? a. Particulate respirator b. Isolation gown c. Shoe covers d. Surgical mask

D Meningeal meningitis is spread via saliva and droplets. Caregivers should wear a surgical mask when within 6 feet of the client and should continue to use Standard Precautions. A particulate respirator, an isolation gown, and shoe covers are not necessary for Droplet Precautions.

A client is scheduled for magnetic resonance imaging (MRI). Which action does the nurse implement before the test? a. Ensure that the person does not eat for 8 hours before the procedure. b. Discontinue all neuroactive medications 3 hours before the procedure. c. Make sure that the client has an identification bracelet that cannot be removed. d. Replace the client's gown with metal snaps with one that has cloth ties.

D Metal objects are a hazard because of the magnetic field used in the MRI procedure. The other actions are not necessary for MRI.

A client presents with an acute exacerbation of multiple sclerosis. Which prescribed medication does the nurse prepare to administer? a. Baclofen (Lioresal) b. Interferon beta-1b (Betaseron) c. Dantrolene sodium (Dantrium) d. Methylprednisolone (Medrol)

D Methylprednisolone is the drug of choice for acute exacerbations of the disease. The other medications are not appropriate.

. During the nursing assessment the patient identifies experiencing episodic headaches with pain described as a 10 on a scale of 1 to 10. The patient describes the pain as unilateral and located on the left side, lasting for 2 to 3 days. The patient also experiences nausea. You would determine the patient was experiencing which type of headache? A. Sinus B. Cluster C. Tension D. Migraine

D Migraine headache is a recurring headache characterized by unilateral (sometimes bilateral) throbbing pain, a triggering event or factor, strong family history, and manifestations associated with neurologic and autonomic nervous system dysfunction.

Which of the following patients is at highest risk for a stroke? A. An obese, 45-year-old Native American B. A 35-year-old Asian American woman who smokes C. A 32-year-old, white woman taking oral contraceptives D. A 65-year-old African American man with hypertension

D Nonmodifiable risk factors for stroke include age (>65 years), male gender, ethnicity or race (African Americans > Hispanics, Native Americans/Alaska Natives, and Asian Americans > whites), and family history of stroke or personal history of a transient ischemic attack or stroke. Modifiable risk factors for stroke include hypertension (most important), heart disease (especially atrial fibrillation), smoking, excessive alcohol consumption (causes hypertension), abdominal obesity, sleep apnea, metabolic syndrome, lack of physical exercise, poor diet (high in saturated fat and low in fruits and vegetables), and drug abuse (especially cocaine). Other risk factors for stroke include a diagnosis of diabetes mellitus, increased serum cholesterol, birth control pills (high levels of progestin and estrogen), history of migraine headaches, inflammatory conditions, hyperhomocysteinemia, and sickle cell disease.

A patient being monitored has an ICP pressure of 12 mm Hg. You understand that this pressure reflects A. a severe decrease in cerebral perfusion pressure. B. an alteration in the production of cerebrospinal fluid. C. the loss of autoregulatory control of intracranial pressure. D. a normal balance between brain tissue, blood, and cerebrospinal fluid.

D Normal ICP ranges from 5 to 15 mm Hg. A sustained pressure above the upper limit is considered abnormal.

You assess a patient for signs of meningeal irritation and observe her for nuchal rigidity. Which option indicates the presence of this sign of meningeal irritation? A. Tonic spasms of the legs B. Curling in a fetal position C. Arching of the neck and back D. Resistance to flexion of the nec

D Nuchal rigidity is a clinical manifestation of meningitis. During assessment, the patient resists passive flexion of the neck by the health care provider.

The nurse assesses for which clinical manifestations in the client with suspected encephalitis? a. Fever of 101° F (38.3° C) b. Nausea and vomiting c. Hypoactive deep tendon reflexes d. Pain on flexion of the neck

D Nuchal rigidity is associated with meningeal irritation and is frequently present in clients with encephalitis. The other manifestations are not associated with encephalitis.

What is a common priority nursing need for a patient with Lewy body dementia? A. Dysphasia B. Apraxia C. Visual agnosia D. Dysphagia

D Patients with Lewy body dementia have problems with dysphagia and immobility. Swallowing problems can lead to impaired nutrition. All dementia patients can have problem with comprehending language and oral communication (dysphasia), inability to manipulate objects or perform purposeful acts (apraxia), and inability to recognize objects of sight (visual agnosia). Dysphagia (trouble with swallowing) is a priority.

A wife indicates she has been providing care to her husband, who was diagnosed with bacterial meningococcal meningitis. What is your most important action related to the wife? A. Teach airborne isolation precautions. B. Assess her for respiratory infection. C. Explain the signs of meningitis. D. Provide prophylactic antibiotics.

D Persons who have close contact with anyone who has bacterial meningitis should be given prophylactic antibiotics, and this supersedes all other actions because she has been exposed. Meningococcal meningitis is highly contagious.

Which statement accurately describes mild cognitive impairment (MCI)? A. People with MCI are often unaware that they have any deficiencies. B. Most people with MCI go on to develop dementia. C. You need to emphasize the need to take drug therapy consistently. D. You must monitor the person for disorientation or feeling lost in a familiar place.

D Persons with MCI should be monitored for the 10 signs of Alzheimer's disease (AD) and signs that they are progressing into dementia. The person with MCI is often aware of a significant change in memory, which also is noticed by family members. A person with dementia is unaware of deficits. Having MCI puts an individual at risk for dementia, and an estimated 15% of people with MCI eventually develop dementia. No drugs are approved for the treatment of MCI

Which patient may be at greatest risk for delirium? A. A patient with fibromyalgia whose chronic pain has recently worsened B. An elderly patient whose recent computed tomography study shows brain atrophy C. A patient with a fracture who has spent the night in the emergency department D. An elderly patient who takes multiple medications for various health problems

D Polypharmacy is implicated in many cases of delirium, and this phenomenon is especially common among older adults. Brain atrophy, if associated with cognitive changes, is indicative of dementia. Alterations in sleep and environment and pain may cause delirium, but this is less of a risk than taking multiple medications by an older adult.

What is the most effective measure for rabies after a patient has sustained a bite by an animal thought to be infected? A. Antibiotic administration B. Contact isolation C. Wound irrigation D. Rapid postexposure prophylaxis

D Rabies usually is fatal. Management efforts are directed at preventing the transmission and onset of the disease, and postexposure prophylaxis is administered. It is more effective than the other measures.

Which patient is most at risk for delirium? A. A 50-year-old woman with cholecystitis B. A 19-year-old man with a fractured femur C. A 42-year-old woman having an elective hysterectomy D. A 78-year-old man admitted to the medical unit with complications related to heart failure

D Risk factors that can precipitate delirium include age 65 years or older, male gender, and severe, acute illness (e.g., heart failure). The 78-year-old man has the most risk factors for delirium.

Which statement is true regarding dementia? A. The patient is often the first one to be aware of the problem. B. Onset is usually relatively sudden. C. Initial memory loss consists of long-term memories. D. Thyroid deficiency is ruled out before the diagnosis.

D Screening for cobalamin (vitamin B12) deficiency and hypothyroidism are usually done before making the diagnosis. It is often a family member, particularly the spouse, who reports the patient's declining memory to the heath care provider. The onset is usually insidious and gradual. In dementia, the memory loss initially relates to recent events, with long-term (remote) memories still intact. With time and progression of the dementia, memory loss includes short-term (recent) and long-term (remote) memory.

A client with epilepsy develops stiffening of the muscles of the arms and legs, followed by an immediate loss of consciousness and jerking of all extremities. How does the nurse document this seizure activity? a. Atonic seizure b. Absence seizure c. Myoclonic seizure d. Tonic-clonic seizure

D Seizure activity that begins with stiffening of the arms and legs, followed by loss of consciousness and jerking of all extremities, is characteristic of a tonic-clonic seizure. The other seizures do not manifest in this manner.

Which of the following are characteristic of a tension-type headache? A. The patient experiences an aura. B. They are aggravated by physical activity. C. Nausea and vomiting are present. D. They involve photosensitivity.

D Tension-type headaches are usually of mild or moderate intensity and not aggravated by physical activity. Tension-type headaches are subcategorized as episodic or chronic. Tension-type headaches can last from minutes to days. There is no prodrome (early manifestation of impending disease) in tension-type headache. The headache does not involve nausea or vomiting but may involve sensitivity to light (photophobia) or sound (phonophobia).

Which option is the most sensitive indication of increased ICP? A. Papilledema B. Cushing's triad C. Projectile vomiting D. Change in the level of consciousness (LOC)

D The LOC is the most sensitive and reliable indicator of the patient's neurologic status. Changes in LOC are a result of impaired cerebral brain flow. Papilledema and Cushing's triad are late signs. Projectile vomiting is not a sensitive indicator.

Emergency medical services arrive to the emergency department with a client who has a cervical spinal cord injury. Which priority assessment does the emergency department nurse perform at this time? a. Level of consciousness and orientation b. Heart rate and rhythm c. Muscle strength and reflexes d. Respiratory pattern and airway

D The first priority for a client with a spinal cord injury is assessment of respiratory status and airway patency. Clients with cervical spine injuries are particularly prone to respiratory compromise and may even require intubation. The other assessments should be performed after airway and breathing are assessed.

The nursing student reports to the nurse in charge that the patient experienced a generalized tonic-clonic seizure. This means that the patient is exhibiting A. a momentary loss of consciousness. B. jerking movements throughout the body. C. rigidity for several seconds, then flaccidity. D. rigidity of muscles followed by muscle jerking.

D The most common generalized seizure is the generalized tonic-clonic (formerly known as grand mal) seizure. Tonic-clonic seizure is characterized by loss of consciousness and falling to the ground if the patient is upright, followed by stiffening of the body (tonic phase) for 10 to 20 seconds and subsequent jerking of the extremities (clonic phase) for another 30 to 40 seconds

What is the best patient to assign to a new graduate nurse on her first week of orientation? A. Patient with bacterial meningitis admitted from the emergency department today B. Patient returning from a craniotomy for a pituitary brain tumor C. Patient with head trauma with suspected epidural bleed admitted 3 hours earlier D. Patient with viral meningitis who is being discharged today

D The new nurse should have the patient who is the most stable and has a predictable outcome. That is the patient with the less serious condition of viral meningitis who is being discharged. The other three patients are potentially unstable and need skilled nursing assessments.

The nurse is teaching a client who is newly diagnosed with epilepsy. Which statement by the client indicates a need for further teaching concerning the drug regimen? a. "I will not drink any alcoholic beverages." b. "I will wear a medical alert bracelet." c. "I will let my doctor know about all of my prescriptions." d. "I can skip a couple of pills if they make me ill."

D The nurse must emphasize that antiepileptic drugs must be taken even if seizure activity has stopped. Discontinuing the medication can predispose the client to seizure activity and status epilepticus. The client should not drink alcohol while taking seizure medications. The client should wear a medical alert bracelet and should make the doctor aware of all medications to prevent complications of polypharmacy.

The nurse suspects the presence of an arterial epidural hematoma in the patient who experiences a. failure to regain consciousness following a head injury b. a rapid deterioration of neurologic function within 24 to 48 hours following a head injury c. nonspecific, nonlocalizing progression of alteration in LOC occurring over weeks or months d. unconsciousness at the time of a head injury with a brief period of consciousness followed by a decrease in LOC

D. An arterial epidural hematoma is the most acute neurologic emergency, and the typical symptoms include unconsciousness at the scene, with a brief lucid interval followed by a decrease in LOC. An acute subdural hematoma manifests signs within 48 hours of an injury; a chronic subdural hematoma develops over weeks or months

Skull radiographs and a computed tomography (CT) scan provide evidence of a depressed parietal fracture with a subdural hematoma in a patient admitted to the emergency department following an automobile accident. In planning care for the patient, the nurse anticipates that a. the patient will receive life-support measures until the condition stabilizes b. immediate burr holes will be made to rapidly decompress the intracranial activity c. the patient will be treated conservatively with close monitoring for changes in neurologic condition d. the patient will be taken to surgery for a craniotomy for evacuation of blood and decompression of the cranium

D. When there is a depressed fracture and fractures with loose fragments, a craniotomy is indicated to elevate the depressed bone and remove free fragments. A craniotomy is also indicated in cases of acute subdural and epidural hematomas to remove the blood and control the bleeding. Burr holes may be used in an extreme emergency for rapid decompression, but with a depressed fracture, surgery would be the treatment of choice

A nursing intervention is indicated for the patient with hemiplegia is a. the use of a footboard to prevent plantar flexion b. immobilization of the affected arm against the chest with a sling c. positioning the patient in bed with each joint lower than the joint proximal to it d. having the patient perform passive ROM of the affected limb with the unaffected limb

D: Having the patient perform passive ROM of the affected limb with the unaffected limb- active ROM should be initiated on the unaffected side as soon as possible, and passive ROM of the affected side should be started on the first day. Having the patient actively exercise the unaffected side provides the patient with active and passive ROM as needed. Use of footboards is controversial because they stimulate plantar flexion. The unaffected arm should be supported, but immobilization may precipitate a painful shoulder-hand syndrome. The patient should be positioned with each joint higher than the joint proximal to it to prevent dependent edema.

The nurse can assist the patient and the family in coping with the long term effects of a stroke by a. informing family members that the patient will need assistance with almost all ADLs b. explaining that the patient's prestroke behavior will return as improvement progresses c. encouraging the patient and family members to seek assistance from family therapy or stroke support groups d. helping the patient and family understand the significance of residual stroke damage to promote problem solving and planning

D: Helping the patient and family understand the significance of residual stroke damage to promote problem solving and planning- the patient and family need accurate and complete information about the effects of the stroke to problem solve and make plans for chronic care of the patient. It is uncommon for patients with major strokes to return completely to pre stroke function, behaviors, and role, and both the patient and family will mourn these losses. The patient's specific needs for care must be identified, and rehabilitation efforts should be continued at home. Family therapy and support groups may be helpful for some patients and families.

A patient's wife asks the nurse why her husband did not receive the clot busting medication (tPA) she has been reading about. Her husband is diagnosed with a hemorrhagic stroke. What should the nurse respond? a. He didn't arrive within the time frame for that therapy b. Not every is eligible for this drug. Has he had surgery lately? c. You should discuss the treatment of your husband with your doctor d. The medication you are talking about dissolves clots and could cause more bleeding in your husband's head

D: The medication you are talking about dissolves clots and could cause more bleeding in your husband's head- tPA dissolves clots and increases the risk for bleeding. It is not used with hemorrhagic strokes. If the patient had a thrombotic/embolic stroke the time frame would be important as well as a history of surgery. The nurse should answer the question as accurately as possible and then encourage the individual to talk with the primary care physician if he or she has further questions.

A patient comes to the emergency department immediately after experiencing numbness of the face and an inability to speak, but while the patient awaits examination, the symptoms disappear and the patient request discharge. The nurse stresses that it is important for the patient to be evaluated primarily because a. the patient has probably experienced an asymptomatic lacunar stroke b. the symptoms are likely to return and progress to worsening neurologic deficit in the next 24 hours c. neurologic deficits that are transient occur most often as a result of small hemorrhages that clot off d. the patient has probably experienced a transient ischemic attack (TIA), which is a sign of progressive cerebral vascular disease

D: The patient has probably experienced a transient ischemic attack (TIA), which is a sign of progressive cerebral vascular disease- A TIA is a temporary focal loss of neurologic function caused by ischemia of an area of the brain, usually lasting only about 3 hours. TIAs may be due to microemboli from heart disease or carotid or cerebral thrombi and are a warning of progressive disease. Evaluation is necessary to determine the cause of the neurologic deficit and provide prophylactic treatment if possible.

The family members of a patient who has just been admitted to the intensive care unit (ICU) with multiple traumatic injuries have just arrived in the ICU waiting room. Which action should the nurse take first?

Describe the patient's injuries and the care that is being provided. rationale: Lack of information is a major source of anxiety for family members and should be addressed first. Family members should be prepared for the patient's appearance and the ICU environment before visiting the patient for the first time. ICU visiting should be individualized to each patient and family rather than being dictated by rigid visitation policies. Inviting the family to participate in a multidisciplinary conference is appropriate but should not be the initial action by the nurse.

4.In reviewing laboratory data on a client, the nurse correlates which findings with Guillain-Barré syndrome (GBS)?

Increased cerebral spinal fluid (CSF) protein level without increased cell count A lumbar puncture is performed to evaluate the CSF. An increased CSF protein level without increased cell count is a distinguishing feature of GBS.

a.The nurse prioritizes which nursing diagnosis for the client admitted with Guillain-Barré syndrome?

Ineffective Breathing Pattern related to skeletal muscle weakness The most common cause of death for the client with Guillain-Barré syndrome is complications from respiratory compromise. Airway and breathing problems should receive priority nursing diagnoses.

Which neurologic test or procedure requires the nurse to determine whether an informed consent has been obtained from the client before the test or procedure?

Lumbar puncture for cerebrospinal fluid (CSF) sampling A lumbar puncture is an invasive procedure with many potentially serious complications. The other assessments or tests are considered noninvasive.

dA 28-year-old woman has had multiple sclerosis (MS) for 3 years and wants to have children before her disease worsens. When she asks about the risks associated with pregnancy, the nurse explains that

MS symptoms may be worse after the pregnancy. Rationale: During the postpartum period, women with MS are at greater risk for exacerbation of symptoms. There is no increased risk for congenital defects in infants born of mothers with MS. Symptoms of MS may improve during pregnancy. Pregnancy, labor, and delivery are not affected by MS.

When the ventilator alarm sounds, the nurse finds the patient lying in bed holding the endotracheal tube (ET). Which action should the nurse take first?

Manually ventilate the patient with 100% oxygen. rationale: The nurse should ensure maximal patient oxygenation by manually ventilating with a bag-valve-mask system. Offering reassurance to the patient, notifying the health care provider about the need to reinsert the tube, and activating the rapid response team also are appropriate after the nurse has stabilized the patient's oxygenation.

If a patient is taking selegiline it is important for you to make sure that meperidine is not included in the patient's list of medications. The common reason a patient would be scheduled to have meperidine is: a) hypertension b) pain c) fever d) Parkinson's disease

Pain

A client tells the nurse that he is experiencing some leg stiffness when walking and slowness when performing ADLs. Occasionally he has noted slight tremors in his hands at rest. This information leads the nurse to suspect

Parkinson's disease (PD). Early in PD the client may notice a slight slowing in the ability to perform ADLs. A general feeling of stiffness may be noticed, along with mild, diffuse muscular pain. Tremor is a common early manifestation that usually occurs in one of the upper limbs.

The nurse notes in the patient's medical history that the patient has a positive Romberg test. Which nursing diagnosis is appropriate?

Risk for falls related to dizziness or weakness Rationale: A positive Romberg test indicates that the patient has difficulty maintaining balance with the eyes closed.

d.In planning an inservice program on Huntington's disease, which information does the nurse include?

Risk identification and counseling should precede genetic testing. The risks of people knowing that they have a debilitating, progressive neurologic disorder range from depression to suicide. Before having genetic testing, clients should undergo risk identification and counseling to assist with decision making.

Characteristics of intracerebral hemorrhage

Rupture of atherosclerotic vessels, carries the poorest prognosis, creates mass that compresses the brain

The client with myasthenia gravis in cholinergic crisis has been treated with atropine. Which nursing intervention is a priority for this client?

Suctioning the client Atropine can cause thickening of secretions and formation of mucous plugs. The client is maintained on a ventilator during the crisis. Measures to remove secretions to prevent the buildup of secretions and the possibility of pneumonia are most important.

20.Immediately after undergoing a thymectomy, the nurse monitors for which complication in the client with myasthenia gravis?

Sudden onset of shortness of breath The complication to be alert for is pneumothorax or hemothorax. The nurse monitors the client for chest pain, sudden onset of shortness of breath, diminished chest wall expansion, decreased breath sounds, restlessness, and change in vital signs. The other symptoms are not likely to occur or are not related to the removal of the thymus.

To determine the effectiveness of medications that a patient has received to reduce left ventricular afterload, which hemodynamic parameter will the nurse monitor?

Systemic vascular resistance (SVR) rationale: Systemic vascular resistance reflects the resistance to ventricular ejection, or afterload. The other parameters will be monitored, but do not reflect afterload as directly.

The nurse is caring for a patient receiving a continuous norepinephrine (Levophed) IV infusion. Which patient assessment information indicates that the infusion rate may be too high?

Systemic vascular resistance (SVR) is elevated. rationale: Vasoconstrictors such as norepinephrine (Levophed) will increase SVR, and this will increase the work of the heart and decrease peripheral perfusion. Bradycardia, hypotension, and low PAWP are not associated with norepinephrine infusion.

When the charge nurse is evaluating the care that a new RN staff member provides to a patient receiving mechanical ventilation, which action by the new RN indicates the need for more education?

The RN positions the patient with the head of bed at 10 degrees. rationale: The head of the patient's bed should be positioned at 30 to 45 degrees to prevent ventilator-acquired pneumonia. The other actions by the new RN are appropriate.

When the nursing supervisor is evaluating the performance of a new RN, which action indicates that the new RN is safe in providing care to a patient who is receiving mechanical ventilation with 10 cm of peak end-expiratory pressure (PEEP)?

The RN uses a closed-suction technique to suction the patient. rationale: The closed-suction technique is suggested when patients require high levels of PEEP to prevent the loss of PEEP that occurs when disconnecting the patient from the ventilator. Suctioning should not be scheduled routinely, but it should be done only when patient assessment data indicate the need for suctioning. Taping connections between the ET and the ventilator tubing would restrict the ability of the tubing to swivel in response to patient repositioning. Ventilator tubing changes increase the risk for ventilator-associated pneumonia (VAP) and are not indicated routinely.

You would correctly identify which age group as most often affected by absence seizures? A. Infants B. Children C. Young adults D. Older adults

The absence (petit mal) seizure usually occurs only in children and rarely continues beyond adolescence. This type of seizure may cease altogether as the child matures, or it may evolve into another type of seizure. Reference: 1493

Which factor related to cerebral blood flow most often determines the extent of cerebral damage from a stroke? A. Amount of cardiac output B. Oxygen content of the blood C. Degree of collateral circulation D. Level of carbon dioxide in the blood

The extent of the stroke depends on rapidity of onset, the size of the lesion, and the presence of collateral circulation.

The primary goal for the treatment of restless legs syndrome is to A. reduce episodes. B. improve sleep quality. C. increase exercise tolerance. D. increase iron stores.

The goal of collaborative management is to reduce patient discomfort and distress and to improve sleep quality. When RLS is secondary to renal failure or iron deficiency, correction of these conditions will decrease symptoms. Reference: 1514

When caring for a patient who has an arterial catheter in the radial artery to monitor blood pressure, which information obtained by the nurse is most important to report to the health care provider?

There is redness at the catheter insertion site. rationale: Redness at the catheter insertion site indicates possible infection. The Allen test is performed before arterial line insertion, and a positive test indicates normal ulnar artery perfusion. A MAP of 86 is normal and the dicrotic notch is normally present on the arterial waveform.

The nurse formulates the following nursing diagnosis for a client with MS: Impaired Physical Mobility related to muscle weakness. Useful interventions the nurse could plan include

a. encouraging long naps or rest periods. b. encouraging strengthening exercises for affected muscles every 4 hours. c. having the client perform ROM exercises at least two times daily. d. performing all the activities of daily living (ADLs) for the client. ANS: C Range-of-motion exercises should be performed at least twice daily.

A nurse is caring for a patient with acute respiratory distress syndrome (ARDS) who is receiving mechanical ventilation using synchronized intermittent mandatory ventilation (SIMV). The settings include fraction of inspired oxygen (FIO2) 80%, tidal volume 450, rate 16/minute, and positive end-expiratory pressure (PEEP) 5 cm. Which assessment finding is most important for the nurse to report to the health care provider? a. Oxygen saturation 99% b. Respiratory rate 22 breaths/minute c. Crackles audible at lung bases d. Heart rate 106 beats/minute

a. O2 Sat of 99%

Which patient is most appropriate for the intensive care unit (ICU) charge nurse to assign to a registered nurse (RN) who has floated from the medical unit? a. A 45-year-old receiving IV antibiotics for meningococcal meningitis b. A 25-year-old admitted with a skull fracture and craniotomy the previous day c. A 55-year-old who has increased intracranial pressure (ICP) and is receiving hyperventilation therapy d. A 35-year-old with ICP monitoring after a head injury last week

a. a 45 y.o. receiving IV antibiotics for meningococcal meningitis

A male patient who has possible cerebral edema has a serum sodium level of 116 mEq/L (116 mmol/L) and a decreasing level of consciousness (LOC). He is now complaining of a headache. Which prescribed interventions should the nurse implement first? a. Administer IV 5% hypertonic saline. b. Draw blood for arterial blood gases (ABGs). c. Send patient for computed tomography (CT). d. Administer acetaminophen (Tylenol) 650 mg orally.

a. administer IV 5% hypertonic saline

The nurse reviews the electronic medical record for a patient scheduled for a total hip replacement. Which assessment data shown in the accompanying figure increase the patient's risk for respiratory complications after surgery? a. Albumin level and recent weight loss b. Mild confusion and recent weight loss c. Age and recent arthroscopic procedure. d. Anemia and recent arthroscopic procedure

a. albumin level and recent weight loss

Which action will the nurse include in the plan of care for a 62-year-old patient who is experiencing pain from trigeminal neuralgia? a. Assess fluid and dietary intake. b. Apply ice packs for 20 minutes. c. Teach facial relaxation techniques. d. Spend time talking with the patient.

a. assess fluid and dietary intake

Admission vital signs for a brain-injured patient are blood pressure 128/68, pulse 110, and respirations 26. Which set of vital signs, if taken 1 hour after admission, will be of most concern to the nurse? a. Blood pressure 154/68, pulse 56, respirations 12 b. Blood pressure 134/72, pulse 90, respirations 32 c. Blood pressure 148/78, pulse 112, respirations 28 d. Blood pressure 110/70, pulse 120, respirations 30

a. blood pressure 154/68, pulse 56, respirations 12

Following a cauda equina spinal cord injury, which action will the nurse include in the plan of care? a. Catheterize patient every 3 to 4 hours. b. Assist patient to ambulate several times daily. c. Administer medications to reduce bladder spasm. d. Stabilize the neck when repositioning the patient.

a. catheterize patient every 3 to 4 hours

A 68-year-old male patient is brought to the emergency department (ED) by ambulance after being found unconscious on the bathroom floor by his spouse. Which action will the nurse take first? a. Check oxygen saturation. b. Assess pupil reaction to light. c. Verify Glasgow Coma Scale (GCS) score. d. Palpate the head for hematoma or bony irregularities.

a. check oxygen saturation

A nurse is caring for a patient who is orally intubated and receiving mechanical ventilation. To decrease the risk for ventilator-associated pneumonia, which action will the nurse include in the plan of care? a. Elevate head of bed to 30 to 45 degrees. b. Suction the endotracheal tube every 2 to 4 hours. c. Limit the use of positive end-expiratory pressure. d. Give enteral feedings at no more than 10 mL/hr.

a. elevate head of bed to 30 to 45 degrees

A 42-year-old patient who has bacterial meningitis is disoriented and anxious. Which nursing action will be included in the plan of care? a. Encourage family members to remain at the bedside. b. Apply soft restraints to protect the patient from injury. c. Keep the room well-lighted to improve patient orientation. d. Minimize contact with the patient to decrease sensory input.

a. encourage family members to remain at the beside

A patient is admitted with possible botulism poisoning after eating home-canned green beans. Which intervention ordered by health care provider will the nurse question? a. Encourage oral fluids to 3 L/day b. Document neurologic symptoms c. Position patient lying on the side d. Observe respiratory status closely

a. encourage oral fluids 3L/day

Which action will the public health nurse take to reduce the incidence of epidemic encephalitis in a community? a. Encourage the use of effective insect repellents during mosquito season. b. Remind patients that most cases of viral encephalitis can be cared for at home. c. Teach about the importance of prophylactic antibiotics after exposure to encephalitis. d. Arrange for screening of school-age children for West Nile virus during the school year.

a. encourage the use of effective insect repellents during mosquito season

Which information about a patient who is receiving cisatracurium (Nimbex) to prevent asynchronous breathing with the positive pressure ventilator requires immediate action by the nurse? a. Only continuous IV opioids have been ordered. b. The patient does not respond to verbal stimulation. c. There is no cough or gag when the patient is suctioned. d. The patient's oxygen saturation fluctuates between 90% to 93%.

a. only continuous IV opioids have been ordered

A 32-year-old pregnant patient with Bell's palsy refuses to eat while others are present because of embarrassment about drooling. The best response by the nurse is to a. respect the patient's feelings and arrange for privacy at mealtimes. b. teach the patient to chew food on the unaffected side of the mouth. c. offer the patient liquid nutritional supplements at frequent intervals. d. discuss the patient's concerns with visitors who arrive at mealtimes.

a. respect the patient's feelings and arrange for privacy at mealtimes

A college athlete is seen in the clinic 6 weeks after a concussion. Which assessment information will the nurse collect to determine whether a patient is developing postconcussion syndrome? a. Short-term memory b. Muscle coordination c. Glasgow Coma Scale d. Pupil reaction to light

a. short term memory

A 39-year-old patient is being evaluated for a possible spinal cord tumor. Which finding by the nurse requires the most immediate action? a. The patient has new onset weakness of both legs. b. The patient complains of chronic severe back pain. c. The patient starts to cry and says, "I feel hopeless." d. The patient expresses anxiety about having surgery.

a. the patient has new onset weakness of both legs

When admitting a patient with possible respiratory failure with a high PaCO2, which assessment information should be immediately reported to the health care provider? a. The patient is somnolent. b. The patient complains of weakness. c. The patient's blood pressure is 164/98. d. The patient's oxygen saturation is 90%.

a. the patient is somnolent

When admitting a 42-year-old patient with a possible brain injury after a car accident to the emergency department (ED), the nurse obtains the following information. Which finding is most important to report to the health care provider? a. The patient takes warfarin (Coumadin) daily. b. The patient's blood pressure is 162/94 mm Hg. c. The patient is unable to remember the accident. d. The patient complains of a severe dull headache.

a. the patient takes wafarin daily

The nurse is caring for a 33-year-old patient who arrived in the emergency department with acute respiratory distress. Which assessment finding by the nurse requires the most rapid action? a. The patient's PaO2 is 45 mm Hg. b. The patient's PaCO2 is 33 mm Hg. c. The patient's respirations are shallow. d. The patient's respiratory rate is 32 breaths/minute.

a. the patient's PAO2 is 45 mm Hg

11. A patient with acute respiratory distress syndrome (ARDS) is placed in the prone position. When prone positioning is used, which information obtained by the nurse indicates that the positioning is effective? a. The patient's PaO2 is 89 mm Hg, and the SaO2 is 91%. b. Endotracheal suctioning results in clear mucous return. c. Sputum and blood cultures show no growth after 48 hours. d. The skin on the patient's back is intact and without redness.

a. the patient's PaO2 is 89 mm Hg and the SaO2 is 91%

When teaching the patient with newly diagnosed multiple sclerosis (MS) about the disease, the nurse explains that?

autoimmune processes cause gradual destruction of the myelin sheath of nerves in the brain and spinal cord. Rationale: The primary pathology in MS is an autoimmune process that leads to loss of the myelin sheath and results in decreased nerve transmission. Although MS susceptibility does appear to be inherited, the disease is not congenital because the interaction of multiple factors precipitates MS development. Impulse transmission along nerve fibers is slowed. Antibodies to acetylcholine receptors do not cause MS.

1. Which actions should the nurse initiate to reduce the risk for ventilator-associated pneumonia (VAP) (select all that apply)? a. Obtain arterial blood gases daily. b. Provide a "sedation holiday" daily. c. Elevate the head of the bed to at least 30°. d. Give prescribed pantoprazole (Protonix). e. Provide oral care with chlorhexidine (0.12%) solution daily.

b, c, d, e

Family members of a patient who has a traumatic brain injury ask the nurse about the purpose of the ventriculostomy system being used for intracranial pressure monitoring. Which response by the nurse is best? a. "This type of monitoring system is complex and it is managed by skilled staff." b. "The monitoring system helps show whether blood flow to the brain is adequate." c. "The ventriculostomy monitoring system helps check for alterations in cerebral perfusion pressure." d. "This monitoring system has multiple benefits including facilitation of cerebrospinal fluid drainage."

b. "the monitoring system helps show whether blood flow to the brain is adequate"

A 46-year-old patient with a head injury opens the eyes to verbal stimulation, curses when stimulated, and does not respond to a verbal command to move but attempts to push away a painful stimulus. The nurse records the patient's Glasgow Coma Scale score as a. 9. b. 11. c. 13. d. 15.

b. 11

A patient with acute respiratory distress syndrome (ARDS) and acute kidney injury has the following medications ordered. Which medication should the nurse discuss with the health care provider before giving? a. Pantoprazole (Protonix) 40 mg IV b. Gentamicin (Garamycin) 60 mg IV c. Sucralfate (Carafate) 1 g per nasogastric tube d. Methylprednisolone (Solu-Medrol) 60 mg IV

b. Gentamicin 60 mg IV

Before administering botulinum antitoxin to a patient in the emergency department, it is most important for the nurse to a. obtain the patient's temperature. b. administer an intradermal test dose. c. document the neurologic symptoms. d. ask the patient about an allergy to eggs.

b. administer an intradermal test dose

A construction worker arrives at an urgent care center with a deep puncture wound after an old nail penetrated his boot.. The patient reports having had a tetanus booster 6 years ago. The nurse will anticipate a. IV infusion of tetanus immune globulin (TIG). b. administration of the tetanus-diphtheria (Td) booster. c. intradermal injection of an immune globulin test dose. d. initiation of the tetanus-diphtheria immunization series.

b. administration of tetanus-diptheria booster

An unconscious 39-year-old male patient is admitted to the emergency department (ED) with a head injury. The patient's spouse and teenage children stay at the patient's side and ask many questions about the treatment being given. What action is best for the nurse to take? a. Ask the family to stay in the waiting room until the initial assessment is completed. b. Allow the family to stay with the patient and briefly explain all procedures to them. c. Refer the family members to the hospital counseling service to deal with their anxiety. d. Call the family's pastor or spiritual advisor to take them to the chapel while care is given.

b. allow the family to stay with the patient and briefly explain the procedures to them

To assess for functional deficits, which question will the nurse ask a patient who has been admitted for treatment of a benign occipital lobe tumor? a. "Do you have difficulty in hearing?" b. "Are you experiencing visual problems?" c. "Are you having any trouble with your balance?" d. "Have you developed any weakness on one side?"

b. are you experiencing visual problems?

Which nursing action has the highest priority for a patient who was admitted 16 hours previously with a C5 spinal cord injury? a. Cardiac monitoring for bradycardia b. Assessment of respiratory rate and effort c. Application of pneumatic compression devices to legs d. Administration of methylprednisolone (Solu-Medrol) infusion

b. assessment of respiratory rate and effort

The oxygen saturation (SpO2) for a patient with left lower lobe pneumonia is 90%. The patient has rhonchi, a weak cough effort, and complains of fatigue. Which action is a priority for the nurse to take? a. Position the patient on the left side. b. Assist the patient with staged coughing. c. Place a humidifier in the patient's room. d. Schedule a 2-hour rest period for the patient.

b. assist the patient with staged coughing

A 20-year-old male patient is admitted with a head injury after a collision while playing football. After noting that the patient has developed clear nasal drainage, which action should the nurse take? a. Have the patient gently blow the nose. b. Check the drainage for glucose content. c. Teach the patient that rhinorrhea is expected after a head injury. d. Obtain a specimen of the fluid to send for culture and sensitivity.

b. check the drainage for glucose content

A patient with respiratory failure has a respiratory rate of 6 breaths/minute and an O2 sat of 88%. The patient is increasingly lethargic. Which intervention will the nurse anticipate? a. administration of 100% oxygen by non-rebreather mask b. endotracheal intubation and positive pressure ventilation c. insertion of mini tracheostomy with frequent suctioning d. initiation of CPAP ventilation

b. endotracheal intubation and positive pressure ventilation

After endotracheal suctioning, the nurse notes that the intracranial pressure for a patient with a traumatic head injury has increased from 14 to 17 mm Hg. Which action should the nurse take first? a. Document the increase in intracranial pressure. b. Ensure that the patient's neck is in neutral position. c. Notify the health care provider about the change in pressure. d. Increase the rate of the prescribed propofol (Diprivan) infusion.

b. ensure that the patient's neck is in neutral position

Which statement by a 40-year-old patient who is being discharged from the emergency department (ED) after a concussion indicates a need for intervention by the nurse? a. "I will return if I feel dizzy or nauseated." b. "I am going to drive home and go to bed." c. "I do not even remember being in an accident." d. "I can take acetaminophen (Tylenol) for my headache."

b. i am going to drive home and go to bed

The nurse admitting a patient who has a right frontal lobe tumor would expect the patient may have a. expressive aphasia. b. impaired judgment. c. right-sided weakness. d. difficulty swallowing.

b. impaired judgement

The nurse is admitting a patient with a basal skull fracture. The nurse notes ecchymoses around both eyes and clear drainage from the patient's nose. Which admission order should the nurse question? a. Keep the head of bed elevated. b. Insert nasogastric tube to low suction. c. Turn patient side to side every 2 hours d. Apply cold packs intermittently to face.

b. insert nasogastric tube to low suction

Which action should the nurse take when assessing a patient with trigeminal neuralgia? a. Have the patient clench the jaws. b. Inspect the oral mucosa and teeth. c. Palpate the face to compare skin temperature bilaterally. d. Identify trigger zones by lightly touching the affected side.

b. inspect the oral mucosa and teeth

A patient admitted with acute respiratory failure has a nursing diagnosis of ineffective airway clearance related to thick, secretions. Which action is a priority for the nurse to include in the plan of care? a. Encourage use of the incentive spirometer. b. Offer the patient fluids at frequent intervals. c. Teach the patient the importance of ambulation. d. Titrate oxygen level to keep O2 saturation >93%.

b. offer the patient fluids at frequent intervals

Which finding for a patient who has a head injury should the nurse report immediately to the health care provider? a. Intracranial pressure is 16 mm Hg when patient is turned. b. Pale yellow urine output is 1200 mL over the last 2 hours. c. LICOX brain tissue oxygenation catheter shows PbtO2 of 38 mm Hg. d. Ventriculostomy drained 40 mL of cerebrospinal fluid in the last 2 hours.

b. pale yellow urine output is 1200 mL over the last 2 hours

Which statement by the nurse when explaining the purpose of positive end-expiratory pressure (PEEP) to the family members of a patient with ARDS is accurate? a. "PEEP will push more air into the lungs during inhalation." b. "PEEP prevents the lung air sacs from collapsing during exhalation." c. "PEEP will prevent lung damage while the patient is on the ventilator." d. "PEEP allows the breathing machine to deliver 100% oxygen to the lungs."

b. peep prevents the lung air sacs from collapsing during exhalation

A 23-year-old patient who is suspected of having an epidural hematoma is admitted to the emergency department. Which action will the nurse plan to take? a. Administer IV furosemide (Lasix). b. Prepare the patient for craniotomy. c. Initiate high-dose barbiturate therapy. d. Type and crossmatch for blood transfusion.

b. prepare the patient for craniotomy

Which action will the nurse take when caring for a 46-year-old patient who develops tetanus from an injectable substance use? a. Avoid use of sedatives. b. Provide a quiet environment. c. Check pupil reaction to light every 4 hours. d. Provide range-of-motion exercises several times daily.

b. provide a quiet environment

Which action will the emergency department nurse anticipate for a patient diagnosed with a concussion who did not lose consciousness? a. Coordinate the transfer of the patient to the operating room. b. Provide discharge instructions about monitoring neurologic status. c. Transport the patient to radiology for magnetic resonance imaging (MRI). d. Arrange to admit the patient to the neurologic unit for 24 hours of observation.

b. provide discharge instructions about monitoring neurologic status

When the nurse is developing a rehabilitation plan for a 30-year-old patient with a C6 spinal cord injury, an appropriate goal is that the patient will be able to a. drive a car with powered hand controls. b. push a manual wheelchair on a flat surface. c. turn and reposition independently when in bed. d. transfer independently to and from a wheelchair.

b. push a manual wheelchair on a flat surface

The nurse is caring for a patient who is intubated and receiving positive pressure ventilation to treat acute respiratory distress syndrome (ARDS). Which finding is most important to report to the health care provider? a. Blood urea nitrogen (BUN) level 32 mg/dL b. Red-brown drainage from orogastric tube c. Scattered coarse crackles heard throughout lungs d. Arterial blood gases: pH 7.31, PaCO2 50, PaO2 68

b. red-brown drainage from orogastric tube

A patient admitted with a diffuse axonal injury has a systemic blood pressure (BP) of 106/52 mm Hg and an intracranial pressure (ICP) of 14 mm Hg. Which action should the nurse take first? a. Document the BP and ICP in the patient's record. b. Report the BP and ICP to the health care provider. c. Elevate the head of the patient's bed to 60 degrees. d. Continue to monitor the patient's vital signs and ICP

b. resport the bp and icp to the health care provider

patient with possible viral meningitis is admitted to the nursing unit after lumbar puncture was performed in the emergency department. Which action prescribed by the health care provider should the nurse question? a. Elevate the head of the bed 20 degrees. b. Restrict oral fluids to 1000 mL daily. c. Administer ceftriaxone (Rocephin) 1 g IV every 12 hours. d. Give ibuprofen (Motrin) 400 mg every 6 hours as needed for headache.

b. restrict oral fluids to 1000 mL daily

After evacuation of an epidural hematoma, a patient's intracranial pressure (ICP) is being monitored with an intraventricular catheter. Which information obtained by the nurse is most important to communicate to the health care provider? a. Pulse 102 beats/min b. Temperature 101.6° F c. Intracranial pressure 15 mm Hg d. Mean arterial pressure 90 mm Hg

b. temperature 101.6

A nurse is caring for a patient with ARDS who is being treated with mechanical ventilation and high levels of positive end-expiratory pressure (PEEP). Which assessment finding by the nurse indicates that the PEEP may need to be reduced? a. The patient's PaO2 is 50 mm Hg and the SaO2 is 88%. b. The patient has subcutaneous emphysema on the upper thorax. c. The patient has bronchial breath sounds in both the lung fields. d. The patient has a first-degree atrioventricular heart block with a rate of 58.

b. the patient has subcutaneous emphysema on the upper thorax

Which assessment data for a patient who has Guillain-Barré syndrome will require the nurse's most immediate action? a. The patient's triceps reflexes are absent. b. The patient is continuously drooling saliva. c. The patient complains of severe pain in the feet. d. The patient's blood pressure (BP) is 150/82 mm Hg.

b. the patient is continuously droolin saliva

The nurse assessing a 54-year-old female patient with newly diagnosed trigeminal neuralgia will ask the patient about a. visual problems caused by ptosis. b. triggers leading to facial discomfort. c. poor appetite caused by loss of taste. d. weakness on the affected side of the face.

b. triggers leading to facial discomfort

A patient with increased ICP is being monitored in the intensive care unit (ICU) with a fiberoptic catheter. Which order is a priority for you? A. Perform hourly neurologic checks. B. Take a complete set of vital signs. C. Administer the prescribed mannitol (Osmitrol). D. Give an H2-receptor blocker.

c he priority is to treat the known existing problem, and mannitol is the only thing that can do that. Because the patient is having the current pressure measured with objective numbers, treating the known problem is a priority over additional assessments. H2-blockers are given when corticosteroids are administered to help prevent gastrointestinal bleeding, but they are not a priority compared with the treatment of ICP.

A patient admitted with dermal ulcers who has a history of a T3 spinal cord injury tells the nurse, "I have a pounding headache and I feel sick to my stomach." Which action should the nurse take first? a. Check for a fecal impaction. b. Give the prescribed analgesic. c. Assess the blood pressure (BP). d. Notify the health care provider.

c. assess the blood pressure

A patient has increased intracranial pressure and a ventriculostomy after a head injury. Which action can the nurse delegate to unlicensed assistive personnel (UAP) who regularly work in the intensive care unit? a. Document intracranial pressure every hour. b. Turn and reposition the patient every 2 hours. c. Check capillary blood glucose level every 6 hours. d. Monitor cerebrospinal fluid color and volume hourly.

c. check capillary blood glucose level every 6 hours

Which finding in a patient with a spinal cord tumor is most important for the nurse to report to the health care provider? a. Back pain that increases with coughing b. Depression about the diagnosis of a tumor c. Decreasing sensation and ability to move the legs d. Anxiety about scheduled surgery to remove the tumor

c. decreasing sensation and ability to move the legs

A 38-year-old patient has returned home following rehabilitation for a spinal cord injury. The home care nurse notes that the spouse is performing many of the activities that the patient had been managing unassisted during rehabilitation. The most appropriate action by the nurse at this time is to a. remind the patient about the importance of independence in daily activities. b. tell the spouse to stop because the patient is able to perform activities independently. c. develop a plan to increase the patient's independence in consultation with the patient and the spouse. d. recognize that it is important for the spouse to be involved in the patient's care and encourage that participation.

c. develop a plan to increase the patient's independence in consultation with the patient and the spouse

The nurse is admitting a patient with a neck fracture at the C6 level to the intensive care unit. Which assessment finding(s) indicate(s) neurogenic shock? a. Hyperactive reflex activity below the level of injury b. Involuntary, spastic movements of the arms and legs c. Hypotension, bradycardia, and warm, pink extremities d. Lack of sensation or movement below the level of injury

c. hypotension bradycardia, and warm, pink extremities

The public health nurse is planning a program to decrease the incidence of meningitis in adolescents and young adults. Which action is most important? a. Encourage adolescents and young adults to avoid crowds in the winter. b. Vaccinate 11- and 12-year-old children against Haemophilus influenzae. c. Immunize adolescents and college freshman against Neisseria meningitides. d. Emphasize the importance of hand washing to prevent the spread of infection.

c. immunize adolescents and college freshmen against Neisseria meningtides

The nurse has administered prescribed IV mannitol (Osmitrol) to an unconscious patient. Which parameter should the nurse monitor to determine the medication's effectiveness? a. Blood pressure b. Oxygen saturation c. Intracranial pressure d. Hemoglobin and hematocrit

c. intracranial pressure

A 41-year-old patient who is unconscious has a nursing diagnosis of ineffective cerebral tissue perfusion related to cerebral tissue swelling. Which nursing intervention will be included in the plan of care? a. Encourage coughing and deep breathing. b. Position the patient with knees and hips flexed. c. Keep the head of the bed elevated to 30 degrees. d. Cluster nursing interventions to provide rest periods.

c. keep the head of the bed elevated to 30 degrees

Which information about a 30-year-old patient who is hospitalized after a traumatic brain injury requires the most rapid action by the nurse? a. Intracranial pressure of 15 mm Hg b. Cerebrospinal fluid (CSF) drainage of 25 mL/hour c. Pressure of oxygen in brain tissue (PbtO2) is 14 mm Hg d. Cardiac monitor shows sinus tachycardia at 128 beats/minute

c. pressure of oxygen in brain tissue is 14 mm Hg

A patient who had a C7 spinal cord injury a week ago has a weak cough effort and audible rhonchi. The initial intervention by the nurse should be to a. administer humidified oxygen by mask. b. suction the patient's mouth and nasopharynx. c. push upward on the epigastric area as the patient coughs. d. encourage incentive spirometry every 2 hours during the day.

c. push upward on the epigastric area as the patient coughs

The nurse is caring for a patient who has a head injury and fractured right arm after being assaulted. Which assessment information requires the most rapid action by the nurse? a. The apical pulse is slightly irregular. b. The patient complains of a headache. c. The patient is more difficult to arouse. d. The blood pressure (BP) increases to 140/62 mm Hg.

c. the patient is more difficult arouse

A patient with chronic obstructive pulmonary disease (COPD) arrives in the emergency department complaining of shortness of breath and dyspnea on minimal exertion. Which assessment finding by the nurse is most important to report to the health care provider? a. The patient has bibasilar lung crackles. b. The patient is sitting in the tripod position. c. The patient's respirations have decreased from 30 to 10 breaths/minute. d. The patient's pulse oximetry indicates an O2 saturation of 91%.

c. the patient's respirations have decreased from 30 to 10 breaths/minute

The charge nurse observes an inexperienced staff nurse caring for a patient who has had a craniotomy for resection of a brain tumor. Which action by the inexperienced nurse requires the charge nurse to intervene? a. The staff nurse assesses neurologic status every hour. b. The staff nurse elevates the head of the bed to 30 degrees. c. The staff nurse suctions the patient routinely every 2 hours. d. The staff nurse administers an analgesic before turning the patient.

c. the staff nurse suctions the patient routinely every 2 hours

During change-of-shift report on a medical unit, the nurse learns that a patient with aspiration pneumonia who was admitted with respiratory distress has become increasingly agitated. Which action should the nurse take first? a. Give the prescribed PRN sedative drug. b. Offer reassurance and reorient the patient. c. Use pulse oximetry to check the oxygen saturation. d. Notify the health care provider about the patient's status.

c. use pulse oximetry to check the O2 Sat

After the emergency department nurse has received a status report on the following patients who have been admitted with head injuries, which patient should the nurse assess first? a. A 20-year-old patient whose cranial x-ray shows a linear skull fracture b. A 30-year-old patient who has an initial Glasgow Coma Scale score of 13 c. A 40-year-old patient who lost consciousness for a few seconds after a fall d. A 50-year-old patient whose right pupil is 10 mm and unresponsive to light

d. a 50 y.o. patient whose right pupil is 10 mm and unresponsive to light

After receiving change-of-shift report on a medical unit, which patient should the nurse assess first? a. A patient with cystic fibrosis who has thick, green-colored sputum b. A patient with pneumonia who has crackles bilaterally in the lung bases c. A patient with emphysema who has an oxygen saturation of 90% to 92% d. A patient with septicemia who has intercostal and suprasternal retractions

d. a patient with septicemia who has intercostal and suprasternal retractions

When assessing a patient with chronic obstructive pulmonary disease (COPD), the nurse finds a new onset of agitation and confusion. Which action should the nurse take first? a. Notify the health care provider. b. Check pupils for reaction to light. c. Attempt to calm and reorient the patient. d. Assess oxygenation using pulse oximetry.

d. assess oxygenation using pulse oximetry

The nurse identifies a patient with type 1 diabetes and a history of herpes simplex infection as being at risk for Bell's palsy. Which information should the nurse include in teaching the patient? a. "You may be able to prevent Bell's palsy by doing facial exercises regularly." b. "Prophylactic treatment of herpes with antiviral agents prevents Bell's palsy." c. "Medications to treat Bell's palsy work only if started before paralysis onset." d. "Call the doctor if you experience pain or develop herpes lesions near the ear."

d. call the doctor if you experience pain or develop herpes lesions near the ear

The nurse is caring for a 78-year-old patient who was hospitalized 2 days earlier with community-acquired pneumonia. Which assessment information is most important to communicate to the health care provider? a. Scattered crackles bilaterally in the posterior lung bases. b. Persistent cough that is productive of blood-tinged sputum. c. Temperature of 101.5° F (38.6° C) after 2 days of IV antibiotic therapy. d. Decreased oxygen saturation to 90% with 100% O2 by non-rebreather mask.

d. decreased O2 Sat to 90% with 100% O2 by non-rebreather mask

A patient with acute respiratory distress syndrome (ARDS) who is intubated and receiving mechanical ventilation develops a right pneumothorax. Which action will the nurse anticipate taking next? a. Increase the tidal volume and respiratory rate. b. Increase the fraction of inspired oxygen (FIO2). c. Perform endotracheal suctioning more frequently. d. Lower the positive end-expiratory pressure (PEEP).

d. lower the PEEP

Which of these nursing actions for a 64-year-old patient with Guillain-Barré syndrome is most appropriate for the nurse to delegate to an experienced unlicensed assistive personnel (UAP)? a. Nasogastric tube feeding q4hr b. Artificial tear administration q2hr c. Assessment for bladder distention q2hr d. Passive range of motion to extremities q4hr

d. passive range of motion to extremities q4h

After having a craniectomy and left anterior fossae incision, a 64-year-old patient has a nursing diagnosis of impaired physical mobility related to decreased level of consciousness and weakness. An appropriate nursing intervention is to a. cluster nursing activities to allow longer rest periods. b. turn and reposition the patient side to side every 2 hours. c. position the bed flat and log roll to reposition the patient. d. perform range-of-motion (ROM) exercises every 4 hours.

d. perform ROM exercises every 4 hours

A patient being admitted with bacterial meningitis has a temperature of 102.5° F (39.2° C) and a severe headache. Which order for collaborative intervention should the nurse implement first? a. Administer ceftizoxime (Cefizox) 1 g IV. b. Give acetaminophen (Tylenol) 650 mg PO. c. Use a cooling blanket to lower temperature. d. Swab the nasopharyngeal mucosa for cultures.

d. swab the nasopharyngeal mucosa for cultures

The nurse is caring for a patient who was admitted the previous day with a basilar skull fracture after a motor vehicle crash. Which assessment finding is most important to report to the health care provider? a. Complaint of severe headache b. Large contusion behind left ear c. Bilateral periorbital ecchymosis d. Temperature of 101.4° F (38.6° C)

d. temperature of 101.4

When assessing a 53-year-old patient with bacterial meningitis, the nurse obtains the following data. Which finding should be reported immediately to the health care provider? a. The patient exhibits nuchal rigidity. b. The patient has a positive Kernig's sign. c. The patient's temperature is 101° F (38.3° C). d. The patient's blood pressure is 88/42 mm Hg.

d. the patient's blood pressure is 88/42 mm Hg

Four hours after mechanical ventilation is initiated for a patient with chronic obstructive pulmonary disease (COPD), the patient's arterial blood gas (ABG) results include a pH of 7.50, PaO2 of 80 mm Hg, PaCO2 of 29 mm Hg, and HCO3- of 23 mEq/L (23 mmol/L). The nurse will anticipate the need to ____________

decrease the respiratory rate. rationale: The patient's PaCO2 and pH indicate respiratory alkalosis caused by too high a respiratory rate. The PaO2 is appropriate for a patient with COPD, increasing the tidal volume would further lower the PaCO2, and the PaCO2 and pH indicate a need to make the ventilator changes.

While assessing a patient with a central venous catheter, the nurse notes the catheter insertion site is red and tender and the patient's temperature is 101.8° F. The nurse will plan to _____________

discontinue the catheter and culture the tip. rationale: The information indicates that the patient has a local and systemic infection caused by the catheter and the catheter should be discontinued. Changing the flush system, administration of analgesics, and continued monitoring will not help prevent or treat the infection. Administration of antibiotics is appropriate, but the line should still be discontinued to avoid further complications such as endocarditis.

In which order will the nurse take these actions when assisting with oral intubation of a patient who is having respiratory distress?

e) Oxygenate the patient with a bag-valve-mask system for several minutes. b) Place the patient in the supine position. c) Inflate the cuff of the d) Attach an end-tidal CO2 detector to the endotracheal a) Obtain a portable chest-x-ray.tube.endotracheal tube. rationale: The patient is pre-oxygenated with a bag-valve-mask system for 3 to 5 minutes before intubation and then placed in a supine position. Following the intubation, the cuff on the endotracheal tube is inflated to occlude and protect the airway. Tube placement is assessed first with an end-tidal CO2 sensor, then with a chest x-ray.

The nurse formulates the following nursing diagnosis for a client with MS: Impaired Physical Mobility related to muscle weakness. Useful interventions the nurse could plan include

having the client perform ROM exercises at least two times daily. Range-of-motion exercises should be performed at least twice daily.

A patient with multiple sclerosis (MS) is to begin treatment with glatiramer acetate (Copaxone). In planning the patient teaching necessary with the use of the drug, the nurse recognizes that the patient will need to be taught

how to draw up and administer injections of the medication. Rationale: Copaxone is administered by self-injection. Oral contraceptives are an appropriate choice for birth control. No laboratory monitoring is needed. The purpose of the medication is to modify the MS disease process.

A patient with a subarachnoid hemorrhage is intubated and placed on a mechanical ventilator. When monitoring the patient, the nurse will need to notify the health care provider if the patient develops _______________

increased jugular vein distention (JVD). rationale: Increases in JVD in a patient with a subarachnoid hemorrhage may indicate an increase in intra-cranial pressure (ICP) and that the PEEP setting is too high for this patient. A respiratory rate of 18, O2 saturation of 94%, and green nasogastric tube drainage are normal.

An elderly patient who has stabilized after being in the intensive care unit (ICU) for a week is preparing for transfer to the step down unit when the nurse notices that the patient has new onset confusion. The nurse will plan to ________________

inform the receiving nurse and then transfer the patient. rationale: The patient's history and symptoms most likely indicate delirium associated with the sleep deprivation and sensory overload in the ICU environment, and informing the receiving nurse and transferring the patient is appropriate. Postponing the transfer is likely to prolong the delirium. Benzodiazepines and restraints contribute to delirium and agitation.

To inflate the cuff of an endotracheal tube (ET) when the patient is on mechanical ventilation, the nurse ______________

injects air into the cuff until a slight leak is heard only at peak inflation. rationale: The minimal occluding volume technique involves injecting air into the cuff until an air leak is present only at peak inflation. The volume to inflate the cuff varies with the ET and the patient's size. Cuff pressure should be maintained at 20 to 25 mm Hg. An accurate assessment of cuff pressure cannot be obtained by palpating the pilot balloon.

A client with MS is being taught self-care measures to prevent constipation. The nurse would realize goals for teaching had been met when the client states he/she will avoid

laxatives. A high-fiber diet, bulk formers, and stool softeners are useful for maintaining stool consistency. Explain that laxatives and enemas should be avoided because they lead to dependence.

When assisting with insertion of a pulmonary artery (PA) catheter, the nurse identifies that the catheter is correctly placed when the _______________

monitor shows a typical PAWP tracing. rationale: The purpose of a PA line is to measure PAWP, so the catheter is floated through the pulmonary artery until the dilated balloon wedges in a distal branch of the pulmonary artery, and the PAWP readings are available. After insertion, the balloon is deflated and the PA waveform will be observed. Systemic arterial pressures are obtained using an arterial line. The length of catheter needed for insertion will vary with patient size.

The most helpful intervention by the nurse for a client experiencing a parkinsonian crisis would be to

place the client in a nonstimulating environment. Occasionally, clients with PD experience a parkinsonian crisis as a result of emotional trauma or sudden or inadvertent withdrawal of anti-parkinsonian medication. Severe exacerbation of tremor, rigidity, and bradykinesia, accompanied by acute anxiety, sweating, tachycardia, and hyperpnea occur. The client should be placed in a quiet room with subdued lighting. Medical treatment may include barbiturates in addition to anti-parkinsonian drugs.

To verify the correct placement of an endotracheal tube (ET) after insertion, the best initial action by the nurse is to _______________

use an end-tidal CO2 monitor to check for placement in the trachea. rationale: End-tidal CO2 monitors are currently recommended for rapid verification of ET placement. Auscultation for bilateral breath sounds and checking chest expansion also are used, but they are not as accurate as end-tidal CO2 monitoring. A chest x-ray confirms the placement but is done after the tube is secured.

Most common artery involved

Internal Carotid Artery

The nurse is assessing a client's coping strategies after suffering a traumatic spinal cord injury. Which information related to this assessment is important for the nurse to obtain? (Select all that apply.) a. Spiritual or religious beliefs b. Level of pain c. Family support d. Level of independence e. Annual income f. Previous coping strategies

A, C, D, F

The nurse is assessing the results of diagnostic tests on a client's cerebrospinal fluid (CSF). Which values and observations does the nurse correlate as most indicative of viral meningitis? (Select all that apply.) a. Clear b. Cloudy c. Normal protein level d. Increased protein level e. Normal glucose level f. Decreased glucose level

A, D, E

Which of the following is the best treatment for acute ischemic stroke? a. heparin b. LMWH c. Alteplase d. Eptifibatie e. Warfarin

C: Alteplase

Characteristics of embolic stroke

Onset unrelated to activity, quick onset and resolution, associated with endocardial disorders

6.A client with Guillain-Barré syndrome is undergoing plasmapheresis. The nurse determines shunt patency through which parameter?

The presence of a bruit Nursing care of the client undergoing plasmapheresis includes care of the shunt. The nurse checks for bruits every 2 to 4 hours for patency.

Which assessment finding is most important for you to follow-up in a newly admitted adult patient diagnosed with viral encephalitis? A. Positive Babinski sign B. Negative Kernig sign C. Doll's-eye reflex D. Deep tendon reflex 2+

A Adults have a negative Babinski sign (toes curl downward, plantar reflex). A positive sign in an adult can indicate disease of the brain or spinal cord. The other signs are normal findings.

When planning care for a patient with MS who has a nursing diagnosis of risk for activity intolerance related to extremity weakness secondary to stress, the most appropriate patient goal is

"The patient will complete ADLs without fatigue." Rationale: Because the nurse has identified the patient's problem as activity intolerance, a patient goal that indicates improvement in activity tolerance, such as ability to accomplish ADLs without fatigue, is most appropriate. The other goals are appropriate for nursing diagnoses such as ineffective coping, impaired physical mobility, and inadequate nutritional intake.

the client is diagnosed with a closed head injury and is in a coma. The nurse writes the client problem as high risk for immobility complications. Which intervention would be included in the plan of care? 1. Position the client with the head of the bed elevated at intervals. 2. Performed active range of motion exercises every 4 hours. 3. Turn the client every shift and massage bony prominences. 4. Explain all procedures to the client before performing them.

1. The head of the clients bed should be elevated to help the lungs expand and prevent stasis of secretions that could lead to pneumonia, a complication of immobility.

4 major causes of hemorrhagic stroke

1)deep hypertensive intracerebral hemorrhages, 2)ruptured saccular aneurysms, 3)arteriovenous malformation,4)spontaneous lobar hemorrhages

Secondary prevention of stroke (3)

1. Anticoagulants 2. Antiplatelets 3. Surgery

Arterial sources of stroke (3)

1. Intracranial vascular disease 2. Carotid vascular disease 3. Aortic arch

the 29-year-old client that was employed as a forklift operator sustains a traumatic brain injury secondary to a motor vehicle accident. The client is being discharged from the rehabilitation unit after 3 months and has cognitive deficits. Which goal would be more realistic for this client? 1. The client will return to work within 6 months. 2. The client is able to focus and stay on task for 10 minutes. 3. The client will be able to dress self without assistance. 4. The client will regain power and bladder control.

2 . Cognitive pertains to mental processes of comprehension, judgment, memory, and reasoning.

A 50 year old patient is admitted to the ED with a scalp laceration that was bleeding profusely and was bandaged at the scene by emergency personnel. The patient is currently stable. The nurse is aware that before wound closure is initiated, which of the following activities is a priority? A complete neurological assessment to rule out cerebral contusion A CAT scan of the head to rule out skull fracture A cerebral arteriogram to identify which vessel has been ruptured A skull series to determine the presence of hematomas

2. A CAT scan of the head to rule out skull fracture Rationale: 1. A complete neurological assessment may indicate symptoms suggestive of a cerebral contusion, but a CAT scan is the diagnostic tool. 2. A CAT scan of the head to rule out skull fracture must be performed before wound closure is undertaken. 3. A cerebral arteriogram is not appropriate in this case because the vessel is outside of the skull. 4. A skull series is used to identify bone placement and continuity and, if a hematoma was suspected, a CAT scan would be the appropriate diagnostic test.

A patient with Alzheimer's disease is being care for at home by her adult daughter. Which of the following should the nurse instruct the daughter to do to support her health promotion needs? Discuss advance directives for management of advanced stages of the disease Encourage getting involved in clinical research studies Instruct to avoid extreme temperatures and exposure to infections Refer for psychological and spiritual counseling

2. Encourage talking about advance directives Rationale: 1. The physical and mental health of the primary family caregiver is critical to the care of the Alzheimer's patient. One health promotional activity that can optimize both patient and caregiver health is to encourage talking about advance directives for management of advanced stages of AD. 2. Involvement in clinical research studies is often done by patients as a way of advancing science while managing their condition with the latest, most promising treatments, but does not promote the health of the patient's daughter. 3. Physical and emotional stress can exacerbate the patient's condition, so patients should be instructed to avoid extreme temperatures and exposure to infections. This does not address the health promotion for the caregiver. 4. Referral for psychological and/or spiritual counseling may help the patient and family come to terms with the disease and its prognosis, but the need is not indicated in the scenario. Cognitive level: Analysis

The nurse is preparing a discharge plan of care for a patient with Parkinson's disease. Which of the following should be included to help the patient with constipation? Restrict fluids Increase dietary fiber Take a laxative every evening Limit protein intake

2. Increase dietary fiber Rationale: 1. Restriction of fluids will increase the risk of constipation. 2. Interventions to assist a patient with Parkinson's disease who is experiencing constipation include increasing dietary fluid and fiber.. 3. Use of laxatives to treat the constipation associated with Parkinson's disease is not recommended. 4. Limiting protein is not recommended to help with constipation. Because protein can interfere with levodopa absorption in a small percentage of patients, patients might be instructed to take levodopa at least 30 minutes before eating or 1 hour after eating.

the client has sustained a severe closed head injury and the neurosurgeon is determining if the client is brain dead. Which data support That the client is brain dead?1. The clients head is turned to the right, the eyes turn to the righT. 2. the EEG has identifiable waveforms. 3. There is no Eye activity when the cold caloric test is performed. 4 the client assumes decorticate posturing when painful stimuli are applied.

3. The cold caloric test, also called the ocular vestibular test, is used to determine if the brain is intact or dead. No Eye activity indicates brain death. If the client eyes moved, that would indicate that the brainstem is intact.

A patient asks the nurse about the differences between a benign and malignant brain tumor. Which of the following statements reflects the characteristics of benign brain tumors that the nurse may include in her explanation? These tumors are nonmalignant and are easily removed by surgery The tumors are self-limiting and do not influence the function of surrounding tissues Benign tumors may be surgically difficult to remove and may compress vital structures These tumors do not progress into malignant tumors

3. Benign tumors may be surgically difficult to remove and may compress vital structures. Rationale: 1. These tumors may be difficult to remove, depending upon structure and position. 2. Tumors may not invade surrounding tissue, but their growth will compress them and may interfere with their function. 3. If the tumor is deep within the brain structure, it may be extremely difficult or impossible to remove surgically. 4. Benign tumors can progress into malignant tumors.

The nurse is monitoring a patient's intracranial pressure for a potential increase. The nurse is aware that the body may initially try to compensate for pressure changes in any of the components of the cranium by which of the following mechanisms? Increasing production of CSF Expansion of brain tissue Blocking the autoregulation of cerebral blood flow Increasing absorption of CSF

4. Increasing absorption of CSF Rationale: 1. Increasing production of CSF would increase the ICP. 2. Brain tissue does not expand; it is compressed. 3. Autoregulation ensures that the brain receives required sufficient oxygen and glucose to meet metabolic needs; this may be inhibited, but it is not blocked as a normal compensatory mechanism. 4. Increased CSF absorption is one mechanism to maintain a constant relationship between the three brain components.

The nurse is devising a plan of care for a patient with multiple sclerosis. Which of the following interventions should the nurse include to help reduce the incidence and frequency of fatigue? Discuss voiding patterns Plan for a daily hot shower Minimize the use of assistive devices Instruct to avoid prolonged exposure to excessive heat

4. Instruct to avoid prolonged exposure to excessive heat. Rationale: 1. Voiding patterns will not affect a patient's level of fatigue. 2. Hot showers and baths can increase fatigue. 3. The use of assistive devices will decrease fatigue. 4. Fatigue is exacerbated with exposure to hot temperatures, including showers, baths, and environmental temperatures.

the client is admitted to the medical floor with a diagnosis of closed head injury. Which nursing intervention has priority? 1. Assess nurological status. 2. Monitor pulse, respiration, and blood pressure. 3. Initiate an intravenous access. 4. Maintain an adequate airway.

4. The most important nursing goal in the management of a client with a head injury is to establish and maintain adequate airway.

the nurse is enjoying a day out at the lake and witnesses a water skier hit the boat ramp. The water skier is in the water not responding to verbal stimuli. The nurse is the first health care provider to respond to the accident. Which intervention should be implemented first? 1. Assess the clients loc. 2. Organize onlookers to remove the client from the lake. 3. Perform a head to toe assessment to determine injuries. 4. Stabilize the clients cervical spine.

4. The nurse should always assume that the client with traumatic head injury may have sustained spinal cord injury. Moving the client could further injure the spinal cord and cause paralysis.Therefore the nurse should stabilize the cervical spinal cordas best as possible prior to removing the client from the water.

Normal ICP ranges from:

5 to 15 mm Hg

The nurse is assessing a client with a history of absence seizures. Which clinical manifestation does the nurse assess for? a. Automatisms b. Intermittent rigidity c. Sudden loss of muscle tone d. Brief jerking of the extremities

A Automatisms are characteristic of absence seizures. These behaviors consist of lip smacking, patting, and picking at clothing. The other manifestations do not correlate with absence seizures.

The patient is admitted with a diagnosis of bacterial meningitis. The patient has a temperature of 101° F and a headache rated as an 8. Which prescription has a priority for you to administer? A. IV cefuroxime (Ceftin) B. Vital signs C. PO acetaminophen (Tylenol) D. Neurologic check

A Bacterial meningitis is a medical emergency, and treating the cause is a priority over treating the symptoms or further assessing effects of the disease process. The antibiotic may be given after cultures are obtained but before the diagnosis is confirmed.

The following patients are in the emergency department. Which one is a priority for treatment? A. A college student with suspected bacterial meningitis B. A teenager who had a partial seizure an hour earlier and is now awake C. Patient with a history of stroke last year with residual hemiparesis D. Patient with suspected trigeminal neuralgia (tic douloureux)

A Bacterial meningitis is considered a medical emergency. The other patients are currently stable.

The nurse is monitoring a patient for increased ICP following a head injury. Which of the following manifestations indicate an increased ICP (select all that apply) a. fever b. oriented to name only c. narrowing pulse pressure d. dilated right pupil > left pupil e. decorticate posturing to painful stimulus

A, B, D, E- The first sign of increased ICP is a change in LOC. Other manifestations are dilated ipsilateral pupil, changes in motor response such as posturing, and fever, which may indicate pressure on the hypothalamus. Changes in vital signs would be an increased systolic BP with widened pulse pressure and bradycardia

What nursing actions can help to minimize the risk of delirium in an older, hospitalized patient (select all that apply)? A. Identification of high risk patients B. Providing patient's glasses and hearing aid C. Consistent nursing staff near the nursing station D. Having a clock and calendar available E. Administering anticholinergic drugs prophylactic

A,B,C,D Identify high-risk patients, including those with neurologic disorders, sensory impairment, or advanced age. Other risk factors include hospitalization in an intensive care unit, lack of a watch or calendar, absence of reading glasses, and untreated pain. Precipitating factors are eliminated. Assess for drug and alcohol withdrawal, fluid and electrolyte imbalance, nutritional deficiencies, and infection. Care includes protecting from harm, increasing familiarity with the environment, and reorientation and behavioral interventions. Polypharmacy is a common cause; drugs are not used prophylactically for this problem.

When a patient is admitted to the emergency department following a head injury, the nurse's first priority in management of the patient once a patent airway is confirmed is a. maintaining cervical spine precautions b. determining the presence of increased ICP c. monitoring for changes in neurologic status d. establishing IV access with a large-bore catheter

A. In addition to monitoring for a patent airway during emergency care of the patient with a head injury, the nurse must always assume that a patient with a head injury may have a cervical spine injury. Maintaining cervical spine precautions in all assessment and treatment activities with the patient is essential to prevent additional neurologic damage.

During admission of a patient with a severe head injury to the ED, the nurse places highest priority on assessment for a. patency of of airway b. presence of a neck injury c. neurologic status with Glascow Coma Scale d. CSF leakage from ears and nose

A. Patency of airway is the #1 priority with all head injuries

11. Norepinephrine (Levophed) has been prescribed for a patient who was admitted with dehydration and hypotension. Which patient information indicates that the nurse should consult with the health care provider before administration of the norepinephrine? a. The patient's central venous pressure is 3 mm Hg. b. The patient is receiving low dose dopamine (Intropin). c. The patient is in sinus tachycardia at 100 to 110 beats/min. d. The patient has had no urine output since being admitted.

ANS: A Adequate fluid administration is essential before administration of vasopressors to patients with hypovolemic shock. The patient's low central venous pressure indicates a need for more volume replacement. The other patient data are not contraindications to norepinephrine administration. DIF: Cognitive Level: Application REF: 1733-1735 | 1736 TOP: Nursing Process: Implementation MSC: NCLEX: Physiological Integrity

A patient with acute respiratory distress syndrome (ARDS) who is intubated and receiving mechanical ventilation develops a pneumothorax. Which action will the nurse anticipate taking? a. Lower the positive end-expiratory pressure (PEEP). b. Increase the fraction of inspired oxygen (FIO2). c. Suction more frequently. d. Increase the tidal volume.

ANS: A Because barotrauma is associated with high airway pressures, the level of PEEP should be decreased. The other actions will not decrease the risk for pneumothorax. DIF: Cognitive Level: Application REF: 1760-1761 TOP: Nursing Process: Planning MSC: NCLEX: Physiological Integrity

15. A patient with septic shock has a BP of 70/46 mm Hg, pulse 136, respirations 32, temperature 104° F, and blood glucose 246 mg/dL. Which of these prescribed interventions will the nurse implement first? a. Give normal saline IV at 500 mL/hr. b. Infuse drotrecogin- (Xigris) 24 mcg/kg. c. Start insulin drip to maintain blood glucose at 110 to 150 mg/dL. d. Titrate norepinephrine (Levophed) to keep mean arterial pressure (MAP) at 65 to 70 mm Hg.

ANS: A Because of the low systemic vascular resistance (SVR) associated with septic shock, fluid resuscitation is the initial therapy. The other actions also are appropriate and should be initiated quickly as well. DIF: Cognitive Level: Application REF: 1735-1737 OBJ: Special Questions: Prioritization TOP: Nursing Process: Implementation MSC: NCLEX: Physiological Integrity

Nursing activities for a client with ALS and family include helping them a. decide on an acceptable level of care early in the course of the disease. b. determine if they want to share the diagnosis to allow genetic testing. c. incorporate nonpharmacologic pain control techniques in the plan of care. d. plan for extensive rehabilitation after exacerbations.

ANS: A Disease management in ALS includes topics such as tube feedings and mechanical ventilation. Planning for an acceptable level of care should begin early in the disease, before a crisis occurs. Of course, decisions should be re-evaluated occasionally as the client's wishes may changes with their experiences with the disease. ALS is not a genetically-acquired disorder. Pain control is usually not an issue in the disease, and as the disease is relentlessly progressive (rather than characterized by remissions and exacerbations), extensive rehabilitation is not utilized.

A patient develops increasing dyspnea and hypoxemia 2 days after having cardiac surgery. To determine whether the patient has acute respiratory distress syndrome (ARDS) or pulmonary edema caused by left ventricular failure, the nurse will anticipate assisting with a. inserting a pulmonary artery catheter. b. obtaining a ventilation-perfusion scan. c. drawing blood for arterial blood gases. d. positioning the patient for a chest radiograph.

ANS: A Pulmonary artery wedge pressures are normal in the patient with ARDS because the fluid in the alveoli is caused by increased permeability of the alveolar-capillary membrane rather than by the backup of fluid from the lungs (as occurs in cardiogenic pulmonary edema). The other tests will not help in differentiating cardiogenic from noncardiogenic pulmonary edema. DIF: Cognitive Level: Application REF: 1753-1754 TOP: Nursing Process: Implementation MSC: NCLEX: Physiological Integrity

1. A patient with neurogenic shock has just arrived in the emergency department after a diving accident. He has a cervical collar in place. Which of the following actions should the nurse take (select all that apply)? a. Prepare to administer atropine IV. b. Obtain baseline body temperature. c. Prepare for intubation and mechanical ventilation. d. Administer large volumes of lactated Ringer's solution. e. Administer high-flow oxygen (100%) by non-rebreather mask.

ANS: A, B, C, E All of the actions are appropriate except to give large volumes of lactated Ringer's solution. The patient with neurogenic shock usually has a normal blood volume, and it is important not to volume overload the patient. In addition, lactated Ringer's solution is used cautiously in all shock situations because the failing liver cannot convert lactate to bicarbonate. DIF: Cognitive Level: Application REF: 1736-1737 OBJ: Special Questions: Alternate Item Format TOP: Nursing Process: Implementation MSC: NCLEX: Physiological Integrity

he nurse cautions clients with ALS and their families to be aware that (Select all that apply) a. activities should be spaced throughout the day. b. clients experience incontinence, an early cause of falling. c. cognition will usually decline late in the disease. d. muscle weakness may cause a risk for injury.

ANS: A, D Safety is a prime concern with ALS (and with any degenerative neurologic disorder). Muscle weakness is progressive, leading to increased risk of falls. Some interventions to prevent this include spacing activities throughout the day, conserving energy, avoiding extremes of hot and cold, and using assistive devices such as canes or wheelchairs. Clients with ALS usually do not experience incontinence and cognition remains intact for the duration of the disorder.

9. Which of these findings is the best indicator that the fluid resuscitation for a patient with hypovolemic shock has been successful? a. Hemoglobin is within normal limits. b. Urine output is 60 mL over the last hour. c. Pulmonary artery wedge pressure (PAWP) is normal. d. Mean arterial pressure (MAP) is 65 mm Hg.

ANS: B Assessment of end organ perfusion, such as an adequate urine output, is the best indicator that fluid resuscitation has been successful. The hemoglobin level, PAWP, and MAP are useful in determining the effects of fluid administration, but they are not as useful as data indicating good organ perfusion. DIF: Cognitive Level: Application REF: 1733-1735 TOP: Nursing Process: Evaluation MSC: NCLEX: Physiological Integrity

A patient with acute respiratory distress syndrome (ARDS) and acute renal failure has the following medications prescribed. Which medication should the nurse discuss with the health care provider before administration? a. ranitidine (Zantac) 50 mg IV b. gentamicin (Garamycin) 60 mg IV c. sucralfate (Carafate) 1 g per nasogastric tube d. methylprednisolone (Solu-Medrol) 40 mg IV

ANS: B Gentamicin, which is one of the aminoglycoside antibiotics, is potentially nephrotoxic, and the nurse should clarify the drug and dosage with the health care provider before administration. The other medications are appropriate for the patient with ARDS. DIF: Cognitive Level: Application REF: 1761-1762 TOP: Nursing Process: Implementation MSC: NCLEX: Physiological Integrity

Which of these nursing actions included in the care of a mechanically ventilated patient with acute respiratory distress syndrome (ARDS) can the RN delegate to an experienced LPN/LVN working in the intensive care unit? a. Assess breath sounds b. Insert a retention catheter c. Place patient in the prone position d. Monitor pulmonary artery pressures

ANS: B Insertion of retention catheters is included in LPN/LVN education and scope of practice and can be safely delegated to an LPN/LVN who is experienced in caring for critically ill patients. Placing a patient who is on a ventilator in the prone position requires multiple staff and should be supervised by an RN. Assessment of breath sounds and obtaining pulmonary artery pressures require advanced assessment skills and should be done by the RN caring for a critically ill patient. DIF: Cognitive Level: Application REF: 1761-1763 OBJ: Special Questions: Delegation TOP: Nursing Process: Planning MSC: NCLEX: Safe and Effective Care Environment

17. When caring for a patient who has septic shock, which assessment finding is most important for the nurse to report to the health care provider? a. BP 92/56 mm Hg b. Skin cool and clammy c. Apical pulse 118 beats/min d. Arterial oxygen saturation 91%

ANS: B Since patients in the early stage of septic shock have warm and dry skin, the patient's cool and clammy skin indicates that shock is progressing. The other information also will be reported, but does not indicate deterioration of the patient's status. DIF: Cognitive Level: Application REF: 1723 OBJ: Special Questions: Prioritization TOP: Nursing Process: Assessment MSC: NCLEX: Physiological Integrity

A patient with respiratory failure has a respiratory rate of 8 and an SpO2 of 89%. The patient is increasingly lethargic. The nurse will anticipate assisting with a. administration of 100% oxygen by non-rebreather mask. b. endotracheal intubation and positive pressure ventilation. c. insertion of a mini-tracheostomy with frequent suctioning. d. initiation of bilevel positive pressure ventilation (BiPAP).

ANS: B The patient's lethargy, low respiratory rate, and SpO2 indicate the need for mechanical ventilation with ventilator-controlled respiratory rate. Administration of high flow oxygen will not be helpful because the patient's respiratory rate is so low. Insertion of a mini-tracheostomy will facilitate removal of secretions, but it will not improve the patient's respiratory rate or oxygenation. BiPAP requires that the patient initiate an adequate respiratory rate to allow adequate gas exchange. DIF: Cognitive Level: Application REF: 1754-1756 TOP: Nursing Process: Planning MSC: NCLEX: Physiological Integrity

When a client is admitted to the hospital with Guillain-Barré syndrome (GBS), the most important assessment the nurse should make is for a. decreasing alertness. b. respiratory difficulty. c. seizure activity. d. urinary retention.

ANS: B The two most dangerous features of GBS are respiratory muscle weakness and autonomic neuropathy involving both the sympathetic and the parasympathetic systems.

12. When the nurse is assessing a patient who is receiving a nitroprusside (Nipride) infusion to treat cardiogenic shock, which finding indicates that the medication is effective? a. No heart murmur is audible. b. Skin is warm, pink, and dry. c. Troponin level is decreased. d. Blood pressure is 90/40 mm Hg.

ANS: B Warm, pink, and dry skin indicates that perfusion to tissues is improved. Since nitroprusside is a vasodilator, the blood pressure may be low even if the medication is effective. Absence of a heart murmur and a decrease in troponin level are not indicators of improvement in shock. DIF: Cognitive Level: Application REF: 1721 | 1723 | 1733-1735 TOP: Nursing Process: Evaluation MSC: NCLEX: Physiological Integrity

12. A client with MG began to experience a sudden worsening of her condition with difficulty in breathing. The nurse explains that this complication of MG is usually initially treated with a. admission and administration of IV corticosteroids. b. an increased dose of anticholinesterase drugs. c. bolus doses of atropine titrated to effect. d. rest and increased sleep.

ANS: B With myasthenic crisis, if an increase in the dosage of the anticholinesterase drug does not improve the weakness, endotracheal intubation and mechanical ventilation may be required. None of the other options is used to treat a myasthenic crisis.

A patient with chronic obstructive pulmonary disease (COPD) arrives in the emergency department complaining of shortness of breath and dyspnea. Which assessment finding by the nurse is most important to report to the health care provider? a. The patient has bibasilar lung crackles. b. The patient is sitting in the tripod position. c. The patient's respiratory rate has decreased from 30 to 10 breaths/min. d. The patient's pulse oximetry indicates an O2 saturation of 91%.

ANS: C A decrease in respiratory rate in a patient with respiratory distress suggests the onset of fatigue and a high risk for respiratory arrest. Therefore immediate action such as positive pressure ventilation is needed. Patients who are experiencing respiratory distress frequently sit in the tripod position because it decreases the work of breathing. Crackles in the lung bases may be the baseline for a patient with COPD. An oxygen saturation of 91% is common in patients with COPD and will provide adequate gas exchange and tissue oxygenation. DIF: Cognitive Level: Application REF: 1751-1752 OBJ: Special Questions: Prioritization TOP: Nursing Process: Assessment MSC: NCLEX: Physiological Integrity

8. The emergency department (ED) receives notification that a patient who has just been in an automobile accident is being transported to your facility with anticipated arrival in 1 minute. In preparation for the patient's arrival, the nurse will obtain a. 500 mL of 5% albumin. b. lactated Ringer's solution. c. two 14-gauge IV catheters. d. dopamine (Intropin) infusion.

ANS: C A patient with multiple trauma may require fluid resuscitation to prevent or treat hypovolemic shock, so the nurse will anticipate the need for 2 large bore IV lines to administer normal saline. Lactated Ringer's solution should be used cautiously and will not be ordered until the patient has been assessed for possible liver abnormalities. Although colloids may sometimes be used for volume expansion, crystalloids should be used as the initial therapy for fluid resuscitation. Vasopressor infusion is not used as the initial therapy for hypovolemic shock. DIF: Cognitive Level: Application REF: 1731 | 1732 | 1733 TOP: Nursing Process: Planning MSC: NCLEX: Physiological Integrity

3. A patient with massive trauma and possible spinal cord injury is admitted to the emergency department (ED). Which finding by the nurse will help confirm a diagnosis of neurogenic shock? a. Cool, clammy skin b. Inspiratory crackles c. Apical heart rate 48 beats/min d. Temperature 101.2° F (38.4° C)

ANS: C Neurogenic shock is characterized by hypotension and bradycardia. The other findings would be more consistent with other types of shock.

The pulse oximetry for a patient with right lower lobe pneumonia indicates an oxygen saturation of 90%. The patient has rhonchi, a weak cough effort, and complains of fatigue. Which action is best for the nurse to take? a. Position the patient on the right side. b. Place a humidifier in the patient's room. c. Assist the patient with staged coughing. d. Schedule a 2-hour rest period for the patient.

ANS: C The patient's assessment indicates that assisted coughing is needed to help remove secretions, which will improve oxygenation. A 2-hour rest period at this time may allow the oxygen saturation to drop further. Humidification will not be helpful unless the secretions can be mobilized. Positioning on the right side may cause a further decrease in oxygen saturation because perfusion will be directed more toward the more poorly ventilated lung. DIF: Cognitive Level: Application REF: 1754-1755 TOP: Nursing Process: Implementation MSC: NCLEX: Physiological Integrity

The nurse obtains the vital signs for a patient admitted 2 days ago with gram-negative sepsis: temperature 101.2° F, blood pressure 90/56 mm Hg, pulse 92, respirations 34. Which action should the nurse take next? a. Administer the scheduled IV antibiotic. b. Give the PRN acetaminophen (Tylenol) 650 mg. c. Obtain oxygen saturation using pulse oximetry. d. Notify the health care provider of the patient's vital signs.

ANS: C The patient's increased respiratory rate in combination with the admission diagnosis of gram-negative sepsis indicates that acute respiratory distress syndrome (ARDS) may be developing. The nurse should check for hypoxemia, a hallmark of ARDS. The health care provider should be notified after further assessment of the patient. Administration of the scheduled antibiotic and administration of Tylenol also will be done, but they are not the highest priority for a patient who may be developing ARDS. DIF: Cognitive Level: Application REF: 1758 | 1760 TOP: Nursing Process: Implementation MSC: NCLEX: Physiological Integrity

13. Which assessment is most important for the nurse to make in order to evaluate whether treatment of a patient with anaphylactic shock has been effective? a. Pulse rate b. Orientation c. Blood pressure d. Oxygen saturation

ANS: D Because the airway edema that is associated with anaphylaxis can affect airway and breathing, the oxygen saturation is the most critical assessment. Improvements in the other assessments also will be expected with effective treatment of anaphylactic shock. DIF: Cognitive Level: Application REF: 1724-1725 | 1732 TOP: Nursing Process: Evaluation MSC: NCLEX: Physiological Integrity

The nurse is positioning the female client with increased intracranial pressure. Which of the following positions would the nurse avoid? a. Head mildline b. Head turned to the side c. Neck in neutral position d. Head of bed elevated 30 to 45 degrees

Answer B. The head of the client with increased intracranial pressure should be positioned so the head is in a neutral midline position. The nurse should avoid flexing or extending the client's neck or turning the head side to side. The head of the bed should be raised to 30 to 45 degrees. Use of proper positions promotes venous drainage from the cranium to keep intracranial pressure down.

A male client is having a tonic-clonic seizures. What should the nurse do first? a. Elevate the head of the bed. b. Restrain the client's arms and legs. c. Place a tongue blade in the client's mouth. d. Take measures to prevent injury.

Answer D. Protecting the client from injury is the immediate priority during a seizure. Elevating the head of the bed would have no effect on the client's condition or safety. Restraining the client's arms and legs could cause injury. Placing a tongue blade or other object in the client's mouth could damage the teeth

A male client is having a lumbar puncture performed. The nurse would plan to place the client in which position a. Side-lying, with a pillow under the hip b. Prone, with a pillow under the abdomen c. Prone, in slight-Trendelenburg's position d. Side-lying, with the legs pulled up and head bent down onto chest.

Answer D. The client undergoing lumbar puncture is positioned lying on the side, with the legs pulled up to the abdomen and the head bent down onto the chest. This position helps open the spaces between the vertebrae.

A priority goal for the patient with restless legs syndrome is to A. increase exercise and activity throughout the day time. B. improve sleep quality. C. promote weight loss. D. increase nighttime consumption warm beverages.

B he goal of collaborative management is to reduce patient discomfort and distress and to improve sleep quality. Reference: 1514

The nurse is teaching a client with chronic headaches about headache triggers. Which statements does the nurse include in the client's teaching plan? (Select all that apply.) a. "Increase your intake of caffeinated beverages." b. "Increase your intake of fruits and vegetables." c. "Avoid all alcoholic beverages." d. "Avoid drinking red wine." e. "Incorporate physical exercise into your daily routine." f. "Incorporate an occasional fast into your plan."

B, D, E

The nurse is teaching a client who has an unstable thoracic vertebral fracture and is being treated with immobilization before surgery. Which statement does the nurse include in the client's teaching? a. "You will need to apply an immobilizing brace snugly around your waist when out of bed." b. "You will remain strapped to the transport back board until the surgical room is ready." c. "Keep your spine in alignment by not sitting up, arching your back, or twisting in bed." d. "An incentive spirometer will prevent you from having atelectasis and pneumonia after surgery."

C The client with a thoracic vertebral fracture is at risk for spinal cord injury, especially with flexion, extension, or rotation of the trunk. The client will be moved to a more comfortable bed to wait for surgery and will remain on bedrest. Although teaching about how to use an incentive spirometer is important for surgical clients, the incentive spirometer alone does not prevent atelectasis and pneumonia; it only assists the client to breathe deeply.

You would expect the patient with restless legs syndrome to identify the discomfort or pain to present A. within the first 30 minutes after getting out of bed in the morning. B. after vigorous exercise. C. when sedentary. D. during a shower or bath.

C The discomfort occurs when the patient is sedentary and is most common in the evening or at night. Reference: 1514

What is a typical description of a person diagnosed with MCI? A. Does not remember knowing a person B. Forgets what an item is used for C. Slow to recall people's names D. Loses sense of time and day

C The person with MCI frequently forgets people's names and is slow to recall them. The other options describe typical behaviors seen in AD.

Treatment of status epilepticus requires initiation of a rapid-acting antiseizure drug that can be given intravenously. You would anticipate which drugs to be administered (select all that apply)? A. phenytoin (Dilantin) B. phenobarbital C. lorazepam (Ativan) D. diazepam (Valium) E. carbemazepine (Tegretol)

C,D Treatment of status epilepticus requires initiation of a rapid-acting antiseizure drug that can be given intravenously. The drugs most commonly used are lorazepam (Ativan) and diazepam (Valium).

The nurse on the clinical unit is assigned to four patients. Which patient should she assess first? a. patient with a skull fracture whose nose is bleeding b. elderly patient with a stroke who is confused and whose daughter is present c. patient with meningitis who is suddenly agitated and reporting a HA of 10 on a 0 to 10 scale d. patient who had a craniotomy for a brain tumor who is now 3 days postoperative and has had continued emesis

C. patient with meningitis who is suddenly agitated and reporting a HA of 10 on a 0 to 10 scale

A patient with Parkinson's disease is admitted to the hospital for treatment of an acute infection. Which nursing interventions will be included in the plan of care? (Select all that apply.)

Cut patient's food into small pieces. Place an arm chair at the patient's bedside. Use an elevated toilet seat.

The clinical diagnosis of dementia is based on A. computed tomography (CT) or magnetic resonance spectroscopy (MRS). B. brain biopsy. C. electroencephalography. D. patient history and cognitive assessment.

D The diagnosis of dementia focuses on determining the cause. A thorough physical examination is performed to rule out other potential medical conditions. Cognitive testing (e.g., Mini-Mental State Examination) is used to evaluate memory, ability to calculate, language, visual-spatial skills, and degree of alertness. A diagnosis of dementia related to vascular causes is based on cognitive loss, vascular brain lesions demonstrated by neuroimaging techniques, and exclusion of other causes of dementia. Structural neuroimaging with CT or magnetic resonance imaging (MRI) is used to evaluate patients with dementia. A psychologic evaluation is indicated to assess for depression.

The nurse is providing discharge teaching to a client after a lumbar laminectomy. For which complication does the nurse instruct the client to return to the hospital? a. Pain at the incision site b. Decreased appetite c. Slight redness and itching at the incision site d. Clear drainage from the incision site

D The finding of clear fluid on the dressing after a laminectomy strongly suggests a cerebrospinal fluid leak, which constitutes an emergency. The client has in increased risk of meningitis with a spinal fluid leak. Pain, redness, and itching at the site are normal. The client should be encouraged to eat a healthy diet but does not need to return to the hospital for a decreased appetite.

Which statement is true regarding the prevalence of Huntington's disease? A. Death occurs in less than one year. B. Survival rate is less than two years. C. Rates are higher in males than females. D. Onset occurs in the 30 to 50 year old age group.

D The onset of HD is usually between 30 and 50 years of age. HD is a genetically transmitted, autosomal dominant disorder that affects both men and women of all races. Death usually occurs 10 to 20 years after the onset of symptoms. Reference: 1515

A client diagnosed with the Huntington gene but who has no symptoms asks for options related to family planning. Which is the nurse's best response? a. "Most clients with the Huntington gene do not pass on Huntington disease to their children." b. "I understand that they can diagnose this disease in embryos. Therefore you could select a healthy embryo from your fertilized eggs for implantation to avoid passing on Huntington disease." c. "The need for family planning is limited because one of the hallmarks of Huntington disease is infertility." d. "Tell me more specifically what information you need about family planning so that I can direct you to the right information or health care provider."

D The presence of the Huntington gene means that the trait will be passed on to all offspring of the affected person. Understanding options for contraception and conception (e.g., surrogate mother options) and implications for children may require the expertise of a genetic counselor or a reproductive specialist. The other options are not accurate.

What action should you take as part of care for a patient who had a craniotomy? A. Use promethazine (Phenergan) for nausea. B. Position the patient on the operative side if a bone flap was removed. C. Administer phenytoin (Dilantin) by rapid intravenous push (IVP) every 6 hours. D. Keep the head in alignment with the trunk.

D The primary goal of care after cranial surgery is prevention of increased intracranial pressure (ICP), which includes keeping the body in alignment. Use of promethazine is discouraged because it can increase somnolence and alter the accuracy of a neurologic assessment. The patient is not positioned on the operative side if a bone flap was removed (craniectomy). Dilantin is administered slowly, no faster than 25 to 50 mg/min. Reference: 1450

You assess that an 87-year-old woman with Alzheimer's disease is continually rubbing, flexing, and kicking out her legs throughout the day. The night shift reports that this same behavior escalates at night, preventing her from obtaining her required sleep. The next step the nurse should take is to A. ask the physician for a daytime sedative for the patient. B. request soft restraints to prevent her from falling out of her bed. C. ask the physician for a nighttime sleep medication for the patient. D. assess the patient more closely, suspecting a disorder such as restless legs syndrome

D The severity of restless legs syndrome (RLS) sensory symptoms ranges from infrequent minor discomfort (paresthesias, including numbness, tingling, and "pins and needles" sensation) to severe pain. The discomfort occurs when the patient is sedentary and is most common in the evening or at night. The pain at night can produce sleep disruptions and is often relieved by physical activity such as walking, stretching, rocking, or kicking. In the most severe cases, patients sleep only a few hours at night, resulting in daytime fatigue and disruption of the daily routine. The motor abnormalities associated with RLS consist of voluntary restlessness and stereotyped, periodic, involuntary movements. The involuntary movements usually occur during sleep. Symptoms are aggravated by fatigue. Reference: 1514

A patient with ICP monitoring has pressure of 12 mm Hg. The nurse understand that this pressure reflects a. a severe decrease in cerebral perfusion pressure b. an alteration in the production of CSF c. the loss of autoregulatory control of ICP d. a normal balance between brain tissue, blood, and CSF

D. A normal balance between brain tissue, blood, and CSF- normal is 10- 15 mm Hg

Successful achievement of patient outcomes for the patient with cranial surgery would be best indicated by the a. ability to return home in 6 days b. ability to meet all self-care needs c. acceptance of residual neurologic deficits d. absence of signs and symptoms of increased ICP

D. Absence of signs and symptoms of increased ICP- The primary goal after cranial surgery is prevention of increased ICP, and interventions to prevent ICP and infection postoperatively are nursing priorities. The residual deficits, rehabilitation potential, and ultimate function of the patient depend on the reason for surgery, the postoperative course, and the patient's general state of health

Metabolic and nutritional needs of the patient with increased ICP are best met with a. enteral feedings that are low in sodium b. the simple glucose available in D5W IV solutions c. a fluid restriction that promotes a moderate dehydration d. balanced, essential nutrition in a form that the patient can tolerate

D. Balanced, essential nutrition in a form that the patient can tolerate= A patient with increased ICP is in a hypermetabolic and hypercatabolic state and needs adequate glucose to maintain fuel for the brain and other nutrients to meet metabolic needs. Malnutrition promotes cerebral edema, and if a patient cannot take oral nutrition, other means of providing nutrition should be used, such as tube feedings or parenteral nutrition. Glucose alone is not adequate to meet nutritional requirements, and 5% dextrose solutions may increase cerebral edema by lowering serum osmolarity. Patients should remain in a normovolemic fluid state with close monitoring of clinical factors such as urine output, fluid intake, serum and urine osmolality, serum electrolytes, and insensible losses.

When assessing the body function of a patient with increased ICP, the nurse should initially assess a. corneal reflex testing b. extremity strength testing c. pupillary reaction to light d. circulatory and respiratory status

D. Circulatory and respiratory status- Of the body functions that should be assessed in an unconscious patient, cardiopulmonary status is the most vital function and gives priorities to the ABCs (airway, breathing, and circulation)

The priority intervention in the emergency department for the patient with a stroke is a. intravenous fluid replacement b. administration of osmotic diuretics to reduce cerebral edema c. initiation of hypothermia to decrease the oxygen needs of the brain d. maintenance of respiratory function with a patent airway and oxygen administration

D: Maintenance of respiratory function with a patent airway and oxygen administration- the first priority in acute management of the patient with a stroke is preservation of life. Because the patient with a stroke may be unconscious or have a reduced gag reflex, it is most important to maintain a patent airway for the patient and provide oxygen if respiratory effort is impaired. IV fluid replacement, treatment with osmotic diuretics, and perhaps hypothermia may be used for further treatment.

John Smith is an 85 year olfd male who began experiencing right sided weakness and difficulty speaking 30 min ago. His BP is 160/90. He is not on anticoagulants. Which of the following are true: a) He is a candidate for tPa b) He is not a candidate for tPa c) He needs a CT and thorough history to determine eligibility for tPa d) He is too old to receive tPa

He is a candidate for tPa

Which conditions or factors in an adult woman diagnosed with MS are most likely to have contributed to this health problem?

Heritability or genetic factors Having a first-degree relative with MS increases the individual's risk of developing the disease. There is a higher prevalence of certain genes in populations with higher rates of MS.

Characteristics of subarachnoid hemorrhage

High initial mortality, symptoms of meningeal irritation, caused by rupture of intracranial aneurysm, associated with sudden, severe headache

The main rationale for the priority nursing diagnosis in spinal cord injuries being risk for constipation is: a) medication side effects b) hyperreflexia c) decreased mobility d) poor nutrition

Hyperreflexia

A client is being treated in the clinic for an exacerbation of multiple sclerosis. The nurse would anticipate administering which drug?

Interferon 1b (Betaseron) Drugs used to treat exacerbations in ambulatory clients include Interferon 1b, Interferon 1a (Avonex), and glatiramer acetate (Copaxone). Diazepam and lioresal could be used to treat spasticity, while steroids are used for acute relapses.

7.The nurse monitors for which complication in the client with Guillain-Barré syndrome who is undergoing plasmapheresis?

Hypovolemia The client undergoing plasmapheresis is at risk of hypovolemia. The nurse monitors fluid status, assesses vital signs, and administers replacement fluid, as indicated.

A patient who has numbness and weakness of both feet is hospitalized with Guillain-Barré syndrome. The nurse will anticipate that collaborative interventions at this time will include

IV infusion of immunoglobulin (Sandoglobulin). Rationale: Because the Guillain-Barré syndrome is in the earliest stages (as evidenced by the symptoms), use of high-dose immunoglobulin is appropriate to reduce the extent and length of symptoms. Mechanical ventilation and tube feedings may be used later in the progression of the syndrome but are not needed now. Corticosteroid use is not helpful in reducing the duration or symptoms of the syndrome.

Permissive hypertension

If you drop blood pressure then more green tissue (ischemic) turns red (infarcted) (only treat BP if >200)

d.35.The early manifestations of amyotrophic lateral sclerosis (ALS) and MS are somewhat similar. Which clinical feature of ALS distinguishes it from MS?

Impairment of respiratory muscles In ALS, there is progressive muscle atrophy until a flaccid quadriplegia develops. Eventually, there is involvement of the respiratory muscles, which leads to respiratory compromise.

Following surgery, a patient's central venous pressure (CVP) monitor indicates low pressures. Which action will the nurse anticipate taking?

Increase the IV fluid infusion rate. rationale: A low CVP indicates hypovolemia and a need for an increase in the infusion rate. Diuretic administration will contribute to hypovolemia and elevation of the head may decrease cerebral perfusion. Documentation and continued monitoring is an inadequate response to the low CVP.

1st thing you do when you suspect a stroke

Non-contrast CT scan (determines hemorrhagic vs ischemic)

When caring for the patient with a pulmonary artery pressure catheter, the nurse notes that the PA waveform indicates that the catheter is in the wedged position. Which action should the nurse take?

Notify the health care provider. rationale: When the catheter is in the wedge position, blood flow past the catheter is obstructed, placing the patient at risk for pulmonary infarction. A health care provider or specially trained nurse should be called to reposition the catheter. The other actions will not correct the wedging of the PA catheter.

When the nurse is weaning a patient who has chronic obstructive pulmonary disease (COPD) from mechanical ventilation, which patient assessment indicates that the weaning protocol should be discontinued?

The patient respiratory rate is 32 breaths/min. rationale: Tachypnea is a sign that the patient's work of breathing is too high to allow weaning to proceed. The patient's heart rate is within normal limits, although the nurse should continue to monitor it. An oxygen saturation of 93% is acceptable for a patient with COPD. A spontaneous tidal volume of 500 mL is within the acceptable range.

Characteristics of a thrombotic stroke

Type most often signaled by TIAs, commonly occurs during or after sleep, strong association with hypertension

A patient who is receiving mechanical ventilation is anxious and is "fighting" the ventilator. Which action should the nurse take first?

Verbally coach the patient to breathe with the ventilator. rationale: The initial response by the nurse should be to try to decrease the patient's anxiety by coaching the patient about how to coordinate respirations with the ventilator. The other actions also may be helpful if the verbal coaching is ineffective in reducing the patient's anxiety.

A client is being treated in the clinic for an exacerbation of multiple sclerosis. The nurse would anticipate administering which drug?

a. Diazepam (Valium) b. Interferon β1b (Betaseron) c. Lioresal (Baclofen) d. Methylprednisolone (Solu-Cortef) ANS: B Drugs used to treat exacerbations in ambulatory clients include Interferon β1b, Interferon β1a (Avonex), and glatiramer acetate (Copaxone). Diazepam and lioresal could be used to treat spasticity, while steroids are used for acute relapses.

A client with MS is being taught self-care measures to prevent constipation. The nurse would realize goals for teaching had been met when the client states he/she will avoid

a. a high-fiber diet. b. citrus fruits. c. laxatives. d. stool softeners. ANS: C A high-fiber diet, bulk formers, and stool softeners are useful for maintaining stool consistency. Explain that laxatives and enemas should be avoided because they lead to dependence.

The most helpful intervention by the nurse for a client experiencing a parkinsonian crisis would be to

a. administer oxygen by nasal catheter. b. give the client IV fluids that contain potassium. c. place the client in a nonstimulating environment. d. provide the client with foods high in calcium. ANS: C Occasionally, clients with PD experience a parkinsonian crisis as a result of emotional trauma or sudden or inadvertent withdrawal of anti-parkinsonian medication. Severe exacerbation of tremor, rigidity, and bradykinesia, accompanied by acute anxiety, sweating, tachycardia, and hyperpnea occur. The client should be placed in a quiet room with subdued lighting. Medical treatment may include barbiturates in addition to anti-parkinsonian drugs.

A client tells the nurse that he is experiencing some leg stiffness when walking and slowness when performing ADLs. Occasionally he has noted slight tremors in his hands at rest. This information leads the nurse to suspect

a. amyotrophic lateral sclerosis (ALS). b. Huntington's disease. c. myasthenia gravis (MG). d. Parkinson's disease (PD). ANS: D Early in PD the client may notice a slight slowing in the ability to perform ADLs. A general feeling of stiffness may be noticed, along with mild, diffuse muscular pain. Tremor is a common early manifestation that usually occurs in one of the upper limbs.

The nurse reminds a group of students about the major component of pathophysiology in multiple sclerosis (MS), which is

a. damage occurs primarily to the dendrites and oligodendrites. b. once damaged, myelin cannot regenerate at all. c. plaques occur anywhere in the white matter of the central nervous system (CNS). d. Schwann cells are destroyed slowly but relentlessly. ANS: C Although plaques may occur anywhere in the white matter of the CNS, the areas most commonly involved are the optic nerves, cerebrum, and cervical spinal cord.

the client diagnosed with a mild concussion is being discharged from the emergency department. which discharge instruction should the nurse teach the clients significant other? 1. awake in the client every 2 hours. 2. monitor for increased intracranial pressure. 3. observe frequently for hypervigilance. 4. offer the client food every 3 to 4 hours.

answer 1. Awakening the client every 2 hours allows the identification of headache, dizziness, lethargy, irritability, and anxiety, all signs a post concussion syndrome ,which would warrant a return to the emergency department.

A patiA patient is seen in the health clinic with symptoms of a stooped posture, shuffling gait, and pill rolling-type tremor. The nurse will anticipate teaching the patient about

antiparkinsonian drugs.

When preparing to assist with the insertion of a pulmonary artery catheter, the nurse will plan to ______________

attach cardiac monitoring leads before the procedure. rationale: Dysrhythmias can occur as the catheter is floated through the right atrium and ventricle, and it is important for the nurse to monitor for these during insertion. Pulmonary artery catheter insertion does not require anesthesia, and the patient will not need to be NPO. Changes in cardiac enzymes or heart sounds are not expected during pulmonary artery catheter insertion.

A 42-A 42 year-old patient who was adopted at birth is diagnosed with early Huntington's disease (HD). When teaching the patient, spouse, and children about this disorder, the nurse will provide information about the

availability of genetic testing to determine the HD risk for the patient's children.

To evaluate the effectiveness of ordered interventions for a patient with ventilatory failure, which diagnostic test will be most useful to the nurse? a. chest x ray b. oxygen saturation c. ABG analysis d. CVP monitoring

c. ABG analysis

the nurse is caring for a client diagnosed with epidural hematoma. Which nursing interventions should the nurse implement? Select all that apply. 1. Maintain the head of the bed at 60 degrees. 2. Administer stool softeners daily. 3. Ensure that pulse oximeter reading is higher than 93 percent. 4. Perform deep Nasal suction every 2 hours. 5. Administer mild sedative.

correct answers 2, 3,5. Stool softeners are initiated to prevent the Bell sell the maneuver which increaseS ICP. oxygen saturation higher the 93 percent ensures oxygenation of the brain tissues. decreasing oxygen levels increase cerebral edema.mild sedative will reduce the clans agitation. Strong narcotics would not be administered because they decrease the clients loc.

Which statement indicates that the family has a good understanding of the changes in motor movement associated with Parkinson's disease?

d. "I can offer smaller meals with bite-size portions and a liquid supplement." A masklike face, drooling, and excess perspiration are common in clients with Parkinson's disease. Changes in facial expression or a masklike facies in a Parkinson's disease client can be misinterpreted. Because chewing and swallowing can be problematic, small frequent meals and a supplement are better for meeting the client's nutritional needs.

A patient has been admitted with meningococcal meningitis. Which observation by the nurse requires action? a. The bedrails at the head and foot of the bed are both elevated. b. The patient receives a regular diet from the dietary department. c. The lights in the patient's room are turned off and the blinds are shut. d. Unlicensed assistive personnel enter the patient's room without a mask.

d. UAP assistive personnel enter the patient's room without a mask

A 68-year-old patient hospitalized with a new diagnosis of Guillain-Barré syndrome has numbness and weakness of both feet. The nurse will anticipate teaching the patient about a. intubation and mechanical ventilation. b. administration of corticosteroid drugs. c. insertion of a nasogastric (NG) feeding tube. d. infusion of immunoglobulin (Sandoglobulin).

d. infusion of immunoglobulin (Sandoglobulin)

Which nursing interventions included in the care of a mechanically ventilated patient with acute respiratory failure can the registered nurse (RN) delegate to an experienced licensed practical/vocational nurse (LPN/LVN) working in the intensive care unit? a. Assess breath sounds every hour. b. Monitor central venous pressures. c. Place patient in the prone position. d. Insert an indwelling urinary catheter.

d. insert an indwelling urinary catheter

When monitoring for the effectiveness of treatment for a patient with left ventricular failure, the most important information for the nurse to obtain is ______________

pulmonary artery wedge pressure (PAWP). rationale: PAWP reflects left ventricular end diastolic pressure (or left ventricular preload). Because the patient in left ventricular failure will have a high PAWP, a decrease in this value will be the best indicator of patient improvement. The other values would also provide useful information, but the most definitive measurement of improvement is a drop in PAWP.

When obtaining a health history and physical assessment for a patient with possible multiple sclerosis (MS), the nurse should

question the patient about any leg weakness or spasm.

A patient with myasthenia gravis (MG) is admitted to the hospital with severe weakness and acute respiratory insufficiency. The health care provider performs a Tensilon test to distinguish between myasthenic crisis and cholinergic crisis. During the test, it will be most important to monitor the patient's

respiratory function.Rationale: Because the patient's respiratory insufficiency is life threatening, it will be most important to monitor respiratory function during the Tensilon test. Pupillary size and muscle strength may also be affected by the test but are not as important to monitor. LOC is not typically affected by MG, although the LOC may be affected by oxygenation in this patient.


Kaugnay na mga set ng pag-aaral

The Point Chapter 44 Nursing Care of a Child with am Alteration in Mobility / Neuromuscular or Musculoskeletal Disorder

View Set

C.6: Electrochemistry, rechargeable batteries and fuel cells (AHL)

View Set

CHEM: ADVANCED EQUILIBRIUM DYNAMIC STUDY MODULE

View Set

Exam 4: Chapter 16 (Multiple Choice)

View Set